You are on page 1of 138

I Hyrje

I HYRJE

Çdo disiplinë shkencore, qoftë ajo matematike apo jo, merret me


studimin e një numëri të caktuar të kuptimeve që kanë për qëllim të
pasqyrojnë një pjesë të realitetit. Disiplinën në fjalë, pra, mund ta
konsiderojmë si bashkësi pohimesh me të cilët karakterizohen kuptimet e
saj. Varësisht nga natyra e disiplinës ndryshon edhe metoda e përfitimit të
dijeve. Në disa disiplina përparësi ka metoda induktive (metoda e provës,
vërejtjes, eksperimentit ), ku njohuritë e reja janë konkluzione të
përgjithshme të shume vërejtjeve dhe provave të bëra. Në disiplinat
matematike rol të veçantë ka metoda deduktive (nxjerrëse). Bazat e
gjeometrisë është disiplinë matematike që për objekt studimi ka bazimin
(ndërtimin deduktiv) të gjeometrisë.

1.1. Bazimi i një disipline matematike

Do të sqarojmë më poshtë se në çka konsiston ndërtimi deduktiv


i cilës do nje disipline matematike.
Të ndërtohet në rrugë deduktive një disiplinë matematike do të
thotë që bashkësia e të gjitha kuptimeve dhe pohimeve të saj të renditet në
një varg logjik, ashtu që çdo kuptim duhet të përkufizohet dhe çdo pohim
duhet të vërtetohet. Përmbajtja e një kuptimi përcaktohet me ndihmën e
kuptimeve të tjera, dhe pokështu, edhe përmbajtjen e këtyre të fundit, duke
mos hyrë në përkufizime vicioze (rrethore), duhet ta përcaktojmë me
kuptime te reja. Me qenë se asnjë kuptim nuk mund të përkufizohet pa
kuptime të njohura, patjetër proçesin e përkufizimit duhet ta fillojmë me
kuptime te papërkufizuara. Kuptimet e papërkufizuara quhen edhe
kuptime themelore, kuptime fillestare, kuptime të para apo kuptime
primitive. Çdo kuptim tjetër te disiplinës në fjalë, përmbajtja e të cilit, në
mënyrë të drejtpërdrejtë apo tërthorazi, përkufizohet me anë të kuptimeve
fillestare quhet kuptim i përkufizuar, kuptim i nxjerrë, kuptim i
deduktuar apo kuptim i derivuar. Përkufizimet janë thënje me anë të të
cilave përcaktohet përmbajtja e kuptimeve të dedukuar.
Saktësinë e një pohimi e mbështesim në pohime të tjera paraprake
dhe po kështu saktesia e këtyre të fundit duhet të arsyetohet me pohime të
reja. Me qenë se asnjë pohim nuk mund të vërtetohet pa pohime te njohura,
duke mos hyrë në arsyetime ciklike, patjetër, procesin e vërtetimit të
pohimeve duhet ta mbështesim në pohime të pavërtetuara. Pohimet e
tilla quhen pohime të para, pohime themelore, pohime fillestare apo
aksioma. Të gjitha pohimet te tjera të disiplinës, saktësia e të cilëve, direkt
2 I Hyrje

apo indirekt, vërtetohet me anë të aksiomave quhen pohime të deduktuara,


pohime te derivuara apo teorema. Vete proçesi i arsyetimit të një pohimi
nga aksiomat apo teoremat paraprake quhet vërtetim.
Që të ndërtohet në rrugë deduktive një disiplinë matematike T duhet në
fillim, përpara çdo përkufizimi dhe përpara çdo vërtetimi:
1. Të marrim papërkufizim një numër të caktuar kuptimesh të saj;
2. Të marrim pavërtetim një numër të caktuar pohimesh të saj;
3. Çdo kuptim tjetër të T e përkufizojmë, drejtpërdrejt apo tërthorazi
me anë të kutimeve themelore;
4. Çdo pohim tjetër të T, në mënyrë të drejtpërdrejtë apo tërrthorazi e
vërtetojmë me anë të pohimeve themelore.

Po ta shënojmë me K bashkësinë e kuptimeve të papërkufizuara, kurse


me A bashkësinë e aksiomave, atëherë dyshen S=(K,A) e quajmë sistem
aksiomatik apo bërthamë të disiplinës T. Me qenë se çdo kuptim tjetër i
T përkufizohet me kuptime te K dhe çdo pohim i T vërtetohet me pohime
të A, atëherë disiplina T do të paraqitet si bashkësi e kuptimeve dhe
pohimeve që varen nga sistemi aksiomatik S. Këtë varësi simbolikisht do
ta shënojmë T=T(S). Për disiplinën të ndërtuar në këtë mënyrë themi se
është ndërtuar me metodën deduktive apo metodën aksiomatike. Fuqia
e metodës aksiomatike qëndron në studimin e vetive të përbashkëta të
objekteve të ndryshme. Disiplina matematike e ndërtuar me metodën
deduktive quhet teori deduktive, teori aksiomatike ose vetëm teori
matematike.
Gjatë ndërtimit aksiomatik të një teorie T, zakonisht shfrytëzojmë edhe
teori të tjera aksiomatike. Teoritë e tilla quhen teori të supozuara për
teorinë T. Logjika matematike dhe teoria e bashkësive janë teori deduktive
te supozuara për të gjitha teorite matematike. Të gjitha disiplinat
matematike janë teori matematike. Gjuha me të cilën shtjellohen ato bëhet
më elegante dhe më precise, në qoftë se shfrytëzohet Logjika matematike.
Prandaj me të drejtë thuhet se Logjika matematike e përbën bazën e çdo
teorie matematike.
Kuptimet e logjikës përdoren gjatë formulimit të përkufizimeve,
aksiomave dhe teoremave, kurse pohimet e logjikës zbatohen gjatë
vërtetimit të teoremave. Nga aspekti formal do të thoshim se teoria
matematike T=T(S) është bashkësi e kuptimeve dhe pohime që janë
deduksione logjike të sistemit aksiomatik S.
I Hyrje 3

Për ndërtimin deduktiv të Gjeometrisë në këtë tekst, përveç logjikës


dhe teorisë së bashkësive, ne do të përdorim edhe teorine e numërave reale.
Le të jetë T=T(S), S=(K,A) teori matematike e ndërtuar me anë të
sistemit aksiomatik S. Po ti zëvendësojmë kuptimet fillestare me variablat
k1,…,km , aksiomat (dhe të gjithë pohimet te tjera ) të teorisë T do të
shëndrohen në formula predikative A1(k1,…,km), …, An(k1,…, km) që i
përmbajnë k1,…, km si variabla të lira. Për kuptime konkrete K1,…, Km
të zgjedhura sipas dëshirës, mund te konstatojmë se i plotësojnë apo nuk i
plotësojnë kërkesat e të gjitha aksiomave të disiplinës tonë, me fjalë të tjera
mund të konstatojmë se formulat predikative A1(K1,…, Km),…, An(K1,…,
Km) janë të sakta apo jo të sakta. Në rastin e parë themi se objektet K1,…,
Km janë model(interpretim, realizim) konkret për teorinë T. Në qoftë se
objektet K1,…, Km që i plotësojnë kërkesat e aksomave janë objekte të
ndonje teorie tjeter matematike T’, atëherë themi se kemi gjetur model të
teorisë T në teorinë T’.
Që të konstruktohet një model i teorisë deduktive T, sëpari duhet të
zgjidhet disiplina T’ ku do ta ndërtojmë modelin. Kuptimeve(objekteve
dhe relacioneve) themelore k1,…,km te teorise T ju shoqërojmë objekte
dhe relacione te disiplinës T’, që s’është e thënë të jenë kuptime themelore
në T’. Duke i përkthyer në gjuhën e T’ të gjitha aksiomat A1,…, An të
teorisë T do të fitojmë thënjet A1’,…, An’ të teorisë T’. Në qoftë se A1’,…,
An’ janë thënje të sakta në T’, atëherë themi se kemi ndërtuar një model
(realizim) të teorisë T në teorinë T’. Me qenë se kuptimet dhe pohimet e
teorisë T janë rrjedhime të të sistemit S, atëherë kuptimet dhe pohimet
përkatëse të tyre ne model do të jenë rrjedhime logjike të dyshes S’=(K’,
A’), ku K’={ K1,…, Km }, A’={ A’1,…, A’m}. Modeli M i teorisë T në
teorinë T’ do të jetë bashkësia kuptimeve dhe pohimeve që jane rrjedhime
logjike të S’. Kjo do të thotë se modeli M s’është gjë tjetër pos teoria T e
interpretuar me gjuhën e teorisë T’. Kështu themi se teorinë T e kemi
zhytur në teorinë T’.
Zgjedhjen e kuptimeve dhe pohimeve fillestare për ndërtimin
bazimin e një teorie matematike e bëjmë sipas dëshirës, kështu që të njëjtën
teori mund ta ndërtojmë me anë të sistemeve të ndryshme aksiomatike.
Për dy sisteme aksiomatike S dhe S’ të teorisë matematike T themi
se janë ekuivalente në qoftë se:
a) Çdo kuptim i njërit prej atyre dy sistemeve mund të
përkufizohet me kuptimet e sistemit tjetër
4 I Hyrje

b) Çdo pohim i njërit prej atyre dy sistemeve mund të vërtetohet


me pohimet e sistemit tjetër.
Se cili sistem aksiomatik do të zgjidhet gjatë ndërtimit të teorisë varet
nga shumë faktorë si të natyrës teorike ashtu edhe te natyrës praktike,
didaktike dhe estetike.
1.2 Parimet themelore të aksiomatikës
Me gjithë zgjedhjen e lirë të kuptimeve dhe pohimeve themelore për
ndërtimin e një teorie matematike, prap së prap kjo zgjedhje duhet ti
nenshtrohet disa kerkesave të rëndësishme për bazimin e një disipline
shkencore. Sistemi aksiomatik duhet të jetë : jokontradiktor, i plotë dhe
i pavarur.
a) Jokontradiksioni është kërkesa e parë dhe e domosdoshme
gjatë zgjedhjes të një sistemi aksiomatik. Për një sistem aksiomatik S
themi se është jokontradiktor (jokundërthënës, i qëndrueshëm, consistent)
në qoftë se në bashkësinë T(S) të rrjedhimeve të tij nuk ekziston asnjë çif
pohimesh që kundërshtojnë njëri tjetrin. Me fjalë të tjera, prej çdo dy
pohimeve që kundërshtojnë njëri tjetrin të paktën njëri nuk mund të
vërtetohet. Zgjidhjen e këti problemi e bëjmë me ndërtimin e modelit të
sistemit aksiomatik në një disiplinë tjetër jokontradiktore. Mund ta
vërtetojmë këtë teoremë:
Teorema1. Sistemi aksiomatik S i teorisë matematike T=T(S) është
jokontradiktor, në qoftë se ekziston model i tij në një disiplinë tjetër
jokontradiktore T’.
Vërtetim. Sikur S të ishte sistem kontradiktor, atëherë në teorinë T(S) do
të ekzitonte një çift pohimesh që do te kundërshtonin njëri tjetrin. Këta
pohime të përkthyer në gjuhën e modelit, do të na epnin dy pohime të
teorisë T’ që do të kundërshtonin njëri tjetrin, gjë që s është e mundur,
sepse në T’ nuk kemi kontradiksion. □
________________
(Supozimi i teoremës është pohimi p: ekziston modeli M i sistemit
aksiomatik S në disiplinën jokontradiktore T’, kurse përfundimi i saj
pohimi q: S është system jokontradiktor. Vërtetimin mund ta bëjmë edhe
me metodën e kontrapozicionit : Në vend të implikacionit p  q do të
vërtetojmë implikacionin ┐q  ┐p , sepse gjykimi (pq) (┐q┐p)
është tautologji. ┐q: Le të jetë S system kontradiktor, atëherë ekziston një
çift pohimesh r,s nga T(S) që kundërshtojnë njëri tjetrin. Por atëherë do të
ekzistojë edhe çifti i pohimeve r’,s’ në modelin M  T’ që kundërshtojnë
I Hyrje 5

njëri tjetrin. Pra, në T’ ekziston një çift pohimesh që kundërshtojnë njëri


tjetrin, që do të thotë se T’ është kontradiktore. Kështu e vërtetuam ┐p.)
_______________________________________________
Nga teorema e më sipërme shohim se zgjidhja e këti problemi është
me karakter relativ, sepse kontradiksioni e teorisë T është sjellë në varësi
te kontradiksionit të teorisë T’. Ne vetëm konstatojmë se disiplina T është
po aq e lirë nga kontradiksioni sa edhe disiplina T’. Ky problem vetëm
është çvendosur nga teoria T në teorinë T’, qe nuk është objekt i studimit
tonë.

b) Si kërkesë e dytë që duhet të plotësojë një teori deduktive është


plotshmëria e sistemit aksiomatik te saj. Për sistemin aksiomatik S të
teorisë matematike T themi se është i plotë, në qoftë se për çdo pohim të
formuluar me termat e asaj teorie mund të vërtetohet saktësia e tij apo
saktësia e negacionit të tij. Në rastin e parë themi se ai pohim është i vërtetë
(ka vend) në T, kurse në të dytin se pohimi është jo i vërtetë (nuk ka vend)
në T. Pra, sitemi aksiomatik është i plotë, në qoftë se për çdo pohim të saj
mund të konstatojmë se ka apo nuk ka vend në atë teori. Në qoftë se
ekziston ndonjë pohim p i formuluar me termat e T i tillë që edhe p edhe
┐p nuk janë rrjedhime të S, atëherë themi se sistemi aksiomatik S është jo
i plotë.
Me gjithë rëndësinë që ka, plotshmëria e sistemit aksiomatik nuk është
kriter i domosdoshëm për ndërtimin deduktiv të një teorie. Sistemet e
qëndrueshme aksiomatike janë të levërdishme edhe kur nuk janë të plotë.
Mangësia e sistemeve të tilla aksiomatike qëndron në faktin se në teori të
tilla disa pohime as mund të vëretetohen as mund të hidhen si të pavërteta.
Por sistemi jo i plotë S mund të plotësohet me aksioma të reja që do të
mundësonin vërtetimin e atyre pohimeve të disiplinës që janë të pavarura
nga S. Prandaj plotshmërine e një sistemi aksiomatik mund ta
përkufizojmë në këtë mënyrë:
Përkufizimi1. Për sistemin aksiomatik S themi se është i plotë, në qoftë
se nuk ekziston asnjë pohim p që mund të formulohet me termat e tij i tillë
që p dhe ┐p nuk janë rrjedhime të S dhe sistemet S1=S{p}, S2=S{┐p}
janë jokontradiktorë.
Me qenë se S1, S2 e përmbajnë sistemin S, atëherë çdo model i S1, S2
është model edhe për S.
6 I Hyrje

Zgjidhja e problemit të plotshmërisë bëhet me anë të izomorfizmit të


modeleve.
Përkufizimi2. Për dy modele M1, M2 të sistemit aksiomatik S themi se
janë izomorfë dhe i shënojmë M1M2 , në qoftë se ekziston bijeksioni f:
M1M2 i tillë që për çdo dy objekte a,bM1 dhe çdo relacion  nga ab
rrjedh f(a)f(b).
Nga përkufizimi i fundit shihet se izomorfizmi i ruan relacionet dhe
për rrjedhim çdo pohim i njërit model është pohim edhe i modelit izomorf
me te. Kriterin e plotshmërisë të sistemit aksiomatik do ta japim me këtë
teoremë:
Teorema2. Sistemi aksiomatik S është i plotë, në qoftë se çdo dy
modele të tij janë izomorfë.
Vërtetim. Sikur sistemi S te ishte jo i plotë, do të ekzistonte pohimi p i
tillë që p dhe ┐p nuk varen nga S dhe sistemet S1=S{p}, S2=S{┐p}
janë jokontradiktorë. Le të jenë M1, M2 modele përkatësisht të sistemeve
S1, S2. Me qenë se M1, M2 janë modele edhe të sistemit S, nga supozimi i
teoremës kemi M1M2 . Nga se pohimi p ka vend në M1 dhe M1M2
atëherë p ka vend edhe në M2. Pra, në M2 ka vend edhe pohimi p edhe
pohimi ┐p, që nuk është e mundur, sepse S2 është system jokontradiktor.

c) Pavarësia e sistemit aksiomatik është kërkesë e
rëndësishme për sistemin, sepse e liron atë nga aksiomat e tepërta.
Përkufizim3. Për sitemin aksiomatik S themi se është i pavarur, në
qoftë se asnjë nga aksiomat e tij nuk është rrjedhim i aksiomave të tjera
dhe asnjë nga kuptimet fillestare nuk mund të përkufizohet nga kuptimet
të tjera fillestare.
Pavarësia e sistemit aksiomatik mund të trajtohet në dy aspekte:
(i) Në aspektin e renditjes, kur secila nga aksiomat nuk
është rrjedhim i aksiomave që janë para saj;
(ii) Në kuptimin absolut, kur secila nga aksiomat nuk është
rrjedhim i të gjitha aksiomave të tjera.
Me qenë se prej pavarësisë absolute rrjedh ajo relative, ne do ta
trajtojmë këtë të dytën.
Për të konstatuar se një aksiomë A është e pavarur nga aksiomat te tjera
veprojmë në këtë mënyrë: Në sistemin S aksiomën A e zëvendësojmë me
I Hyrje 7

mohimin e saj A’. Shënojmë me S’={S-{A}}{A’}, kurse me T’=T(S’)


bashkësinë e të gjitha rrjedhimeve të sistemit S’. Pavarësine e aksiomës A
tani e vërtetojmë me këtë teoremë:
Teorema 3. Aksioma A është e pavarur, në qoftë se sistemi
aksiomatik S’ është jokontradiktor.
Vërtetim. Sikur aksioma A të varej nga aksiomat e tjera të S ajo do të
ishte rrjedhim edhe i sistemit S’, rrjedhimisht në teorinë T(S’) do të kishin
vend të dy pohimet A dhe A’, që s’është e mundur sepse sistemi S’ është
jokontradiktor. □
Nga teorema e mësipërme shohim se për të vërtetuar pavarsinë e një
aksiome A duhet të vërtetojmë qëndrueshmërinë e sistemit te ri aksiomatik
S’, ku aksioma A është zëvendësuar me negacionin e saj A’. Kështu , për
të vërtetuar pavarsine e sistemit S duhet të vërtetohet qëndrueshmëria e n
sistemeve të tjerë aksiomatikë. Sjellja ne minimum e numrit të aksiomave
te pavarura të sistemit shpeshherë ka për pasojë ndërlikimin dhe
paqartësinë e vërtetimit, sidomos në fillim të teorisë. Kështu që, për tu
larguar nga kjo paqartësi nganjëherë nuk insistohet që numri i aksiomave
të jetë minimal, por lejohet që sistemi aksomatik të jetë i varur. Kështu
veprohet me studimet fillestare dhe tekstet shkolore.

1.3. Zhvillimi historik i gjeometrisë. Elementet e Euklidit.

Gjeometria si një nga disiplinat më të vjetra, jo vetëm matematike, e


ka historinë e saj të gjatë dhe mjaft të pasur. Për këtë arsye është vështirë
të konstatohet me saktësi si ku dhe kur paraqitet për herë të para zanafilla
e saj. Njohuritë e para gjeometrike zënë fill me qytetërimet e para te disa
popujve të vjetër siç janë egjiptasit, babilonasit, asirët dhe sumerët.
Dokumentet historike me përmbajtje matematike janë të pakta për kah
numëri (Papirusi iMoskës dhe ai i Londrës) dhe në bazë të tyre vështirë
mund të konstatohet niveli i njohurive matematike të atyre kohrave.
Gjeometria,ashtu si edhe shumë disiplina të tjera shkencore ka lindur nga
jeta praktike dhe shekuj me rradhë është zhvilluar shkencë empirike.
Njohuritë gjeometrike janë fituar si konkluzione të përgjithshme pas
shumë vërejtjeve dhe provave të bëra. Sipas Herodotit (historian grek i
shekullit të Vërtetim., p.e.s.) njohuritë e para gjeometrike i gjemë në
Egjipt, si rezultat i matjes së tokës pas vërshimeve të shpeshta të lumit Nil.
Këtë e dëshmon edhe vetë emri i saj Geometria, që në greqisht do të thotë
8 I Hyrje

dhematje. Njohuritë e fituara matematike me shekuj janë përcjellë brezave


të ardhshëm.
Një pjesë e madhe e këtyre njohurive kishte të bënte me rregullat për
njehsimin e syprinave të sipërfaqeve (kënddrejtit, paralelogramit,
trekëndëshit,etj.) dhe vëllimit të disa trupave (kuadrit, prizmës dhe
piramidës). Edhepse formulat për llogaritje janë dhënë pa arsyetime nuk
do të thotë se ato janë fituar thjesh me metodën empirike. Ky grumbull
dijesh matematike i fituar me mundin e sa e sa gjeneratave, që gjithnjë po
shtohej duhej të sistemohej. Ndonjë përpjektje konkrete për sistematizimin
e materialit të tillë te këta popuj nuk është e njohur.
Nga gjysma e mileniumit të parë p.e.s. në gadishullin ballkanik lindi
dhe filloi të zhvillohet qytetërimi i grekëve të vjetër. Grekët e vjetër filluan
të organizohen në të ashtuquajtur qytete-shtete dhe të hynë në marrdhenje
të ngushta ekonomike me popujt fqinjë. Kur roli udhëheqës në shkencë
dhe kulturë kalon te grekët e vjetër, për një kohë relativisht të shkurtë (tre-
katër qind vjet), jovetëm që fitohet një numër i madh dijesh të reja
gjeometrike, por gjeometria merr një kahje te re të zhvillimit të saj. Metoda
empirike u zëvendësua me metoda të rrepta logjike të njohjes që
mundësonin përfitimin e njohurive të reja matematike drejtpërdrejt, pa iu
referuar realitetit. Kështu, duke filluar që nga Talesi (624-547 p.e.s.),
Pitagora (580-500 p.e.s.), Hipokrati (470-410 p.e.s.), Demokriti (450-370
p.e.s.), Platoni (427-347 p.e.s.), Aristoteli (384-322 p.e.s.) kemi një pleadë
të tërë mendimtarësh, filozofësh dhe matematicientësh që dhanë një
kontribut të çmuar në këtë drejtim1. Edhe pse përpjekjet e tyre, si material
i shkruar nuk, kanë arritë në ditët tona rëndësia e tyre është tepër e madhe.
Themelues i metodës deductive në shkencë konsiderohet Pitagora.
Ai e kishte të qartë se sistemimi i materialit shkencor nuk mund të kryhet
në mënyrë konsekuente pa i patur faktet e para. Pitagora pajtohet me faktin
se njohuritë gjeometrike mund të rradhiten duke u nisur prej fakteve më të
qarta, më të evidente, por se cilat janë ato nuk e dimë sepse nuk ka mbetur
asgjë e shkruar prej tij. Procesi i ndërtimit aksiomatik në gjeometri zë fill
në Akademinë e Platonit. Platoni duke e ndier mangësinë në përfundimin
sipas intuitës e apostrofon rëndësinë e rreptësisë logjike dhe deduksionit
në proçesin e njohjes. Platoni theksonte se matematika është kusht i
domosdoshëm për njohjen e realitetit. Se ç’rëndësi i kushtonte ai
gjeometrisë dëshmon edhe fakti se në hyrje të akademisë së tij shkruante:
Këtu të mos hyjë asnjë që nuk e njeh gjeometrinë.
Bazat teorike të metodës deductive në formë më të përgjithshme i
vuri Aristoteli. Kuptimet i konsideron ai si abstraksione të dukurive,

1
Shiko: Mjeshër të mëdhej të matematikës greke.
I Hyrje 9

rrënjët e të cilave ndodhen në botën reale. Çdo kuptim duhet përkufizuar


me kuptime paraprake dhe se ekzistojnë kuptime që nuk i përkufizojmë.
Pokështu, edhe çdo pohim duhet ta arsyetojmë me pohime paraprake dhe
se ekzistojnë pohime fillestare që nuk i arsyetojmë. Pohimet fillestare
Aristoteli i ndan në postulate dhe aksioma. Postulatet janë pohime
fillestare te disiplinës konkrete, ndërsa aksiomat janë me karakter të
përgjithshëm. Me kuptimet dhe pohimet themelore, si të domozdoshme
për njohjen, Aristoteli hyn në thelb të përfundimit deduktiv dhe trajtimit
aksiomatik,duke i dalluar ndërmjet tyre përkufizimet, postulatet, aksiomat
dhe teoremat.
Vepra më e plotë, ku në rrugë aksiomatike janë përmbledhur të
gjitha faktet gjeometrike të antikitetit dhe që si material i shkruar ka arritur
edhe në ditët tona është Elementet e matematikanit të greqisë antike
Euklidit (365-270 p.e.s.). Për jetën e Euklidit sot fare pak dihet. Nuk dihet
bile as data e saktë e lindjes dhe vdekjes, as kombësia dhe shënime të tjera
biografike.Dihet vetëm se është shkolluar në Atinë te nxënësit e Platonit
dhe se themeluar shkollën e matematikës në Muzeinonin e Aleksandrisë.
Euklidi, në Elementet e tij,duke u bazuar në doktrinën e Aristotelit dhe
punën e paraardhsëve te tij (Hipokratit, Demokritit, etj. veprat e të cilëve
nuk dihen), i vë vehtes për detyrë që, duke u nisur nga kuptimet dhe
pohimet fillestare gjeometrike, çdo kuptim tjetër ta përkufizojë dhe çdo
pohim ta arsyetoje me pohimet e mëparshme. Vepra Elementet e Euklidit
përbëhet prej 13 librave dhe në to janë përmbledhur të gjitha njohuritë e
gjeometrisë elementare të njohura në atë kohë. Në librat 1-4 përpunohet
planimetria, në librat 5 dhe 6 trajtohet teoria e proprcioneve, librat 7-10
kanë më tepër karakter arithmetik, në to përpunohet teoria gjeometrike e
numrave. Në tri librat e fundit shtjellohet gjeometria në hapësirë.
Librin e parë Euklidi e fillon me 23 përkufizime, me të cilat jepen
kuptimet elementare si pika, drejtëza, rrafshi, këndi, rrethi, etj. Në
përkthim të lirë po i përmendim këtu të gjitha përkufizimet e para të këti
libri, për të vënë në dukje lidhjen e këtyre kuptimeve me kuptimet e natyrës
fizike si pjesë, kufi, gjatësi, gjerësi etj.
1. Pikë është ajo që nuk ka pjesë.
2. Vija është gjatësi pa gjerësi.
3. Kufijtë e vijës janë pika.
4. Drejtëza është vija që është njëlloj e vendosur ndaj të
gjitha pikave të saj.
5. Sipërfaqe është ajo që ka vetëm gjatësi dhe gjerësi.
6. Anët e sipërfaqes janë vija.
7. Sipërfaqe e rrafshtë është ajo që është njëlloj e vendosur bdaj të gjitha
drejtëzave te saj.
10 I Hyrje

8. Këndi në rrafsh është pjerrtësia e ndërsjellë e dy vijave që priten dhe që


nuk i takojnë një drejtëze.
9. Në qoftë se vijat që e formojnë këndin janë drejtëza, këndi
quhet drejtvizor.
10. Në qoftë se një drejtëz që qëndron mbi një drejtëz tjetër formon me të dy
kënde të përbrinjëshëm të barabartë, atëherë secili prej tyre është i drejtë,
kurse drejtëza e ngritur quhet normale e asaj mbi të cilën qëndron.
11. Kënd i gjerë është ai që është më i madh se këndi i drejtë.
12. Kënd i ngushtë është ai që është më i vogël se këndi i drejtë.
13. Kufi është ai që është fundi i çdo gjëje.
14. Figurëë ajo që është e kufizuar me një ose me mëshumë kufij.
15. Rreth quhet figura e kufizuar me një vije të tillë (që quhet periferi), ashtu
që të gjitha drejtëzat që kalojnë nëpëer një pikë që ndodhet brenda figurës
janë të njëjta ndaj asaj vije.
16. Kjo pikë quhet qendër e rrethit.
17. Diametër i rrethit është çdo drejtez që kalon nëpër qendër dhe në të dy anët
është e kufizuar me periferinë e rrethit. Diametri e përgjysmon rrethin.
18. Gjysmërreth quhet figura e kufizuar prej diametrit dhe periferisë se rrethit
të ndarë prej tij. Qendra e gjysmërrethit është e njëjtë me qendrën e rrethit.
19. Figura drejtvizore janë ato që kufizohen me drejtëza; trekëndëshat janë të
kufizuar me tri, katërkëndëshat me katër dhe shumëkëndëshat me shumë
drejtëza.
20. Nga figurat trekëndëshe, trekëndshi barabrinjës ka tri brinjë të barabarta,
trekëndëshi barakrahësh ka dy brinjë të barabarta, kurse trekëndëshi
brinjëndryshëm i ka të gjitha brinjët të ndryshme.
21. Mandej, prej figurave trekëndëshe, trekëndësh kënddrejtë është ai që ka
kënd të drejtë, trekëndësh këndgjerë është ai që ka kënd të gjerë, kurse
trekëndësh këndngushtë është i ka të tre këndet të ngushtë.
22. Prej figurave katërkëndëshe katërori është barabrinjës dhe i ka të gjithë
këndet të drejtë, kënddrejti është me katër kënde të drejtë por nuk është
barabrinjës, rrombi me brinjë të barabarta por jo me kënde të drejtë,
romboidi me brinjë përballë të barabarta por nuk është barabrinjës as me
kënde të drejtë.
23. Paralele janë ato drejtëza që shtrihen në një rrafsh dhe që duke u zgjatur
ne te dy anët nuk takohen askund.

Siç vihet re, përkufizimet e Euklidit janë sqarime të shkurtra të


kuptimeve më elementare gjeometrike dhe se vetë Euklidi fare nuik thirret
në to më vonë. Duke e mbështetur arsyetimin e pohimeve në faktet
fillestare (postulatet dhe aksiomat) nuk është e qartë pse Euklidi mundohet
të përkufizojë çdo gjë. Kuptimet më elelementare pikë, vije, drejtëz i
I Hyrje 11

përkufizon me kuptimet pjesë, kufi, gjatësi, gjerësi të cilat as nuk i


përkufizon as i mer si fillestare. Ndoshta shkaku qëndron në faktin se
gjeometria e asaj kohe s’ka qenë tjetër pos dhematje dhe se konceptet
pjesë, kufi, gjatësi, etj. kanë qenë evidente.

Postulatet e Euklidit janë fakte fillestare gjeometrike dhe në përkthim


të lirë formulohen kështu:

Kërkohet që :
1. Nga çdo pikë në tjetrën të tërhiqet një drejtëz.
2. Dhe që çdo drejtëz të mund të vazhdohet në pambarim.
3. Dhe që me çdo qendër dhe çdo rreze të ndërtohet rrethi.
4. Dhe se të gjithë këndet e drejtë janë të barabartë.
5. Dhe që dy drejtëza, të prera nga një e tretë, e që formojnë me të kënde të
brendshme të njëanshme, shuma e të cilëve është më e vogël se dy kënde
të drejtë, të priten pikërisht nga ajo anë nga është kjo shumë më e vogël
nga dy kënde të drejtë.
Tre postulatet e para, siç shihet, kanë karakter ndërtimi., kurse formulimi
i postulatit të pestë është mjaft i gjerë dhe i ndërlikuar.

Aksiomat e Euklidit janë pohime fillestare që dallohen nga postulatet me


karakterin e të përgjithshëm (me përjashtim të aksiomave 7 dhe 9). Ato
formulohen kështu:

1. Të barabartët me një të tretë janë të barabartë edhe midis veti.


2. Në qoftë se (objekteve) të barabartë u shtohen (objekte) të barabartë, edhe
tërësitë janë të barabarta.
3.Në qoftë se prej (objekteve) të barabartë zbriten (objekte) të
barabartë,edhe mbetjet do të jenë të barabarta.
3. Në qoftë se (objekteve) jo të barabarta u shtohen (objekte) të
barabartë, tërësitë do të jenë jo të barabarta.
4. Në qoftë se (objektet) të barabartë dyfishohen, do fitohen (objekte) të
barabartë.
5. Në qoftë se (objektet) e barabarta përgjysmohen, fitohen (objekte) të
barabartë.
6. Objektet (gjeometrike) që mund të përputhen janë të barabartë midis
tyre.
7. Pjesa është më e vogël se tërësia.
8. Dy drejtëza nuk përcaktojnë zonë.
12 I Hyrje

Nga pikpamja e aksiomatikës së sotme Elementet e Euklidit kanë mjaft të


meta. Si konccepte themelore merren gjatësia, gjerësia, kufiri, pjesa, etj.
me të cilat përkufizohen kuptimet të tjera. Kjo gjë nuk është e rastit po te
kihet parasysh se gjeometria lindi si nevoje e matjes. Euklidi në formë të
hapur apo të mbuluar i përdor cilësitë e figurave që nuk janë përfshirë në
pohimet fillestare.Kështu, psh. ai e cakton kulmin e tretë të trekëndëshit
barabrinjës në prerjen e harqeve rrethore me qendër në dy kulmet e tjera
duke pranuar nga intuita vazhdueshmërinë e vijës rrethore. Euklidi e
përdor kuptimin “ndërmjet” dhe kuptimin e lëvizjes gjeometrike duke u
bazuar ne evidencën e vizatimit pa i përmendur ato fare në listën e
përkufizimeve dhe aksiomave. Largimet e këtilla nga rreptësia logjike në
veprën e Euklidit nuk janë të rralla. Këto kushtëzohen me nivelin e
njohurive matematike në kohen kur ai jetoi (shumë kuptime si lëvizja,
vazhdueshmëria, etj. ende nuk ishin përpunuar) dhe me pjekurinë e
mendimit matematik. Megjithë të metat e shumta të kësaj vepre, rëndësia
e tyre është tepër e madhe. Prej saj, një kohë të gjatë (afër 20 shekuj) jo
vetëm që është mësuar gjeometria, por ajo u ka shërbyer si model brezave
të ardhshëm për mënyrën deductive të sistemimit në shkencë. Në historinë
e matematikës janë të rralla veprat si Elementet e Euklidit që kanë patur
ndikim aq të madh në zhvillimin e matematikës, dhe jovetëm asaj. Ajo
është përkthyer pothuaj se në të gjitha gjuhët e botës.

1.4. Postulati i pestë i Euklidit dhe rëndësia e tij ne zhvillimin e


gjeometrisë

Shumë kohë përpara, që në kohën antike u vu re se sistemi aksiomatik i


Euklidit nuk ishte i plotë, prej tij nuk mund të nxirreshin në rrugë logjike
të gjitha faktet e njohura gjeometrike. Përpjekjet e para në këtë drejtim i
bëri Arhimedi (287-212 p.e.s.). Në veprën e tij “Mbi rruzullin dhe
cilindrin” Arhimedi paraqiti edhe pesë aksioma, prej të cilave,
aksioma e pestë:
Prej dy vijave jo të barabarta, dy sipërfaqeve jo të
barabarta, apo dy trupave jo të barabartë,
më e madhja do të jetë më e vogël prej asaj madhësije që fitohet
kur të voglën e shumëfi-
shojmë sipas dëshirës
edhe sot, e formuluar pak më ndryshe, merret si pohim themelor në gjeometri
(Aksioma e Arhimedit).
Pas Arhimedit shekuj me rradhë u bënë përpjekje të shumta për
përmirësimin e aksiomatikës të Euklidit, megjithse ato, gjer në shekullin
I Hyrje 13

19, nuk kishin ndonjë ndryshim esencial nga aksiomatika e Euklidit dhe
Arhimedit.
Rol të rëndësishëm në këtë drejtim pati postulati i pestë i Euklidit.
Formulimi tepër i gjatë i këtij postulati si dhe fakti se ai nuk gjen zbatim
të menjëhershëm në Elementet, qenë motiv që shumë matematikanë i
shtynë të dyshojnë se mos vallë ky postulat është rrjedhim i postulateve të
tjerë të Euklidit. Që nga koha e Euklidit gjer nga gjysma e shekullit 19,
pothuaj se çdo matematikan në zë është marrë me vërtetimin e postulatit
të pestë. Disa prej tyre, bile, menduan se ia arritën qëllimit, duke mos qenë
të vetëdijshëm se gjatë vërtetimit, në formë të mbuluar, janë thirrur në
postulatin e pestë. Më të rëndësishme në këtë drejtim janë, sidomos,
punimet e këtyre tree matematikanëve: Matematikanit italian G.Saccheri
(1667-1733), zvicëranit Lambert (1728-1777) dhe matematikanit francez
Legendre (1752-1833). Që të tre në vend të postulatit të Vërtetim.
Mundohen të vërtetojnë ndonjë pohim ekuivalent me te.
Saccheri, në punimin e tij: “Euklidi i pastruar nga çdo njollë”, botuar në
Milano, 1773 merr në shqyrtim një katërkëndësh me dy kënde të drejtë
praën bazës dhe me brinjë anësore të barabarta. Katërkëndëshi i tillë sot
njihet me emërin katërkëndësh i Saccherit. Pasi vërteton se edhe dy
këndet e tjerë të këti katërkëndëshi janë të barabartë, ai për ta supozon se
janë: të gjerë, të drejtë apo të ngushtë. Duke e ditur se hipoteza e këndit
të drejtë është ekuivalente me postulatin e Vërtetim., Saccheri përpiqet që
ti hedhë si të pavërteta dy hipotezat e tjera. Por deri sa hipotezën e këndit
të gjerë ai e shumë shpejt e sjell në kundërshtim me pohimet e mëparshme,
me hipotezën e këndit të ngushtë Saccheri nxjerr një varg pohimesh që nuk
janë në kundërshtim me asnjë nga pohimet e mëparshme. Në pohimin e
33-të ai vuri re se po të vlejë hipoteza e këndit të ngushtë, në rrafsh
ekzistojnë dy drjtëza joprerëse të cilat, në njërën anë i largohen
pambarimisht njëra tjrtrës, kurse në anën tjetër asimptotikisht i afrohen
njëra tjetrës. Një natyrë e këtillë, “e çuditshme” e dy drejtëzave joprerëse
të një rrafshi, e shtyri Saccherin të mos vazhdojë më tutje me rrjedhimet e
drejta dhe të logjikshme të kësaj hipoteze, po të vijë në këtë përfundim: Në
qoftë se vlen hipoteza e këndit të ngushtë, atëherë do të ekzistojnë dy
drejtëza të ndryshme në rrafsh që në pikën pambarimisht të largët do të
kenë normale të përbashkët, që nuk është e mundur. Saccheri nuk mundi
të vërejë gabmin që bëri në këtë rast duke i shtrirë vetitë e figurave të
kufizuara në hapësira të pafundme. Megjithë bindjen e tij se e vërtetoi
postulatin e Vërtetim., Saccherivuri re ndryshimin e madh që ekzistonte
ndërmjet këtyre hipotezave. Deri sa e para e sillte atë në kontradiksionin e
dëshiruar, e dyta nuk i siguronte një gjë të tillë. Ai nuk e kuptoi se me
14 I Hyrje

rrjedhimet e hipotezës së dytë vinte themelet e një gjeometrie të re, që sot


njihet me emrin gjeometri hiperbolike.
Lamberti, në punimin e tij “Teoria e vijave paralele (1761) merr në
shqyrtim një katërkëndësh me tre kënde te drejtë (gjysmën e
katërkëndëshit të Saccherit). Për këndin e katërt i shikoi tri hipotezat.
Hipotezën e këndit të gjerë e solli në kontradiksion me pohimet e
mëparshme, kurse hipoteza e kendit të drejtë i jepte postulatin e V. Me
hipotezën e këndit të ngushtë Lamberti i nxorri po ato rrjedhime sikur edhe
Saccheri, por për ndryshim prej tij,Lamberti nuk e hodhi këtë hipotezë. Me
këtë supozim Lamberti vërtetoi se shuma e këndeve në çdo trkëndësh është
më e vogëlse dy kënde të drejtë dhe se syprina e trekëndëshit është
proporcionale me defektin këndor të tij. Ai ardhi në përfundim se
rrjedhimet e hipotezës së këndit të ngushtë mund të gjejnë zbatim në
sipërfaqen sferike me rreze imagjinare dhe kështu qe më afër prej të gjithë
matematicientëve të shek.18 për zgjidhjen e drejtë të problemit të
postulatit të V. (Siç dihet, formulat e trigonometrisë hiperbolike mund të
fitohen nga formulat e trigonometrisë sferike, në qoftë se rrezja r e sferës
zëvevdësohet me rrezen imagjinare ri, ku i 2 = 1).
Për vërtetimin e postulatit të V Legendre-i i shqyrtoi tri mundësitë për
shumën S( ∆ ) të trekëndëshit ∆. S( ∆ ) mund të jetë më e madhe, e
barabartë apo më e vogël nga shuma 2d e dy këndeve të drejtë. Supozimin
S( ∆ )>2d Legendre-i e hodhi si të pamundshëm, kurse S( ∆ ) = 2d ishte
ekuivalente me postulatin e V. Supozimin S( ∆ ) < 2d e solli në
kontradiksion me pohimet e gjeometrisë absolute, duke përdorur gjatë
vërtetimit pohimin: Të gjitha normalet në njërin krah të këndit të ngushtë
e presin krahun tjetër, pohim ky ekuivalent me postulatin eV.
Edhepse Saccheri, Lamberti, Legendre-i, si edhe shumë të tjerëpara tyre
nuk mundën ta “vërtetojnë” postulatin e Vërtetim. Të Euklidit, punimet e
tyre janë kontribut i çmuar drejt zgjidhjes së drejtë të këti problemi,
problem ky numër një i kohës që dominoi mëshumë se dy mijë vjet.
Mundimet e sa e sa brezave përpara u kurorëzuan në shekullin 20
me punimet e Gausit, Boljait dhe Llobaçevskit, që pavarësisht nga njëri
tjetri, duke hedhur temelet e një gjeometrie të re, bënë edhe zgjidhjen e
plotë të këtij problemi: Postulati i V nuk është rrjedhim i postulateve
të tjerë te Euklidit.
Matematikani gjerman K.F.Gauss (1777-1833) u muar një kohë të gjatë
me postulatin e V. Në fillim edhe Gaussi u mundua ta vërtetoje postulatin
e Vërtetim., por më vonë duke u bindur në pamundësine e vërtetimit, në
korepodencat që mbante me shkët e tij shkronte: Hipoteza që shuma e
këndeve në trekëndësh është më e vogël se 180o të shpie në një gjeometri
e ndryshme nga ajo e jona. Unë e kam zhvilluar këtë gjeometri deri në atë
I Hyrje 15

masë, sa që në të mund të zgjidh çdo problem, me përjashtim të percaktimit


të një konstante C. Ka mundësi që unë ti përpunoj studimet e mia të gjera
rreth kësaj çështje ashtu që ato të botohen, siç është e mundur gjithashtu
që unë për këtë çështje të mos vendos gjatë gjithë jetës sime. Vërtet Gaussi
në gjallje të tij nuk botoi asgjë për këtë çështje.
Inxhinjeri hungarez Janosh Boljai (1802-1860) punimet e tij për
problemin e paraleleve i botoi si shtojcë (apendix) në librin e babait të tij
…, prandaj punimi i tij në literaturë njuhet me emrin Apendix. Për vlerën
e apendiksit i ati i Janoshit kërkonte mendimin e Gaussit. Në përgjigje të
letrës Gaussi, ndërmjet të tjerave, shkronte: Vepren e djalit tuaj nuk mund
ta lavdëroj, sepse të lavdërosh këtë vepër do të thotë të lavdërosh
vetvehten. E tërë përmbajtja e kësaj vepre dhe rezultatet e saj përputhen
me mendimet e mia të filluara shumë vjet më parë. Duke mos e patur
përkrahjen e Gaussit, Janoshi hoqi dorë nga çdo studim tjetër matematik.

Nikolai Llobaçevski(1793-1856) e kishte të qartë se postulati i V i


Euklidit është është ekuivalent me pohimin: Nëpër pikën joincidente me
drejtëzën, në rrafshin e tyre kalon vetëm një drejtëz joprerëse me drejtëzën
e dhënë.Edhe ky në fillim u mundua ta vërtetojë postulati i V , por më vonë
problemin e shtroi nga e kundërta: Çdo të ndodhë në qoftë se postulati i V
zëvendësohet me pohimin: Nëpër pikën joincidente me drejtëzën e dhënë,
në rrafshin e tyre, kalojnë të paktën dy drejtëza që nuk e presin
drejtëzën e dhënë. Me këtë supozim, Llobaçevski e zhvilloi gjeometrinë
gjer në atë masë sa edhe me postulatin e V dhe duke mos hasur asnjëherë
në kontradiksion vjen në këtë përfundimi: Ekziston gjeometria ku nuk
ka vend postulati i V i Eyklidit, por negacioni i tij. Duke qenë i bindur
në saktësinë logjike të kësaj teorije, Llobaçevski qe i pari që pati guxim
punimin e tij ta publikojë dhe për nder të tij, më vonë, kjo teori u quajt
Gjeometria joeuklidiane e Llobaçevskit. Gjetja e modeleve konkrete të
kësaj gjeometrienë gjeometrinë euklidiane tregoi, jovetëm se kjo gjeometri
është po aq e që ndrueshme sa edhe gjeometria euklidiane, por edhe zgjidhi
definitivisht problemin shekullor të postulatit të V , duke vërtetuar kështu
se postulati i V i Euklidit është i pavarur nga gjeometria absolute. Jovetëm
kaq, teoria e qëndrueshme e Gaussit, Janoshit dhe Llobaçevskit e zgjeroi
edhe vetë botëkuptimin e gjeometrisë si teori e botës fizike. Njëherë u
konstatua se aksiomat nuk janë dogma të patundshme me të cilat ndërtohet
gjeometria, pos se ato mund të ju nënshtrohen ndryshimeve.
Kjo arritje e madhe për ndërtimin e gjeometrisë duke qenë i pavarur
nga bota reale fizike bëri të mundur lindjen e gjeometrive të reja
joeuklidiane. Matematikani gjerman Bernard Riemann (1826-1866)
shqyrtoi teorine ku aksioma e paraleleve zëvendësohet me pohimin: Të
16 I Hyrje

gjitha drejtzat të një rrafshi që kalojnë nëpër një pikë joincidente me një
drejtëz të dhënë e presin atë drejëtz. Me këtë supozim Riemanni ndërtoi
një teori, që më vonë Kleini e quejti Gjeometri joeuklidiane e
Riemannit, e cila me një prafrim më të madh e pasqyron botën reale fizike,
gjeometria e s cilës vetëm në dukje të parë është euklidiane.
Me lindjen e gjeometrive joeuklidiane lindi nevoja për gjetjen e një
sistemi aksiomatik që do ti përfshinte të gjitha këto gjeometri.Në këtë
drejtim kontribut të çmuar dhanë matematicientët: Peano, Moris Pasch,
Veroneze, idetë e të cilëve i kurorëzoi matematikani gjerman David
Hilbert (1862-1943), me veprën e tij “Bazat e gjeometrisë” që u botua më
1899 dhe që më 1903 e fitoi çmimin ndërkombtar të Llobaçevskit. Hilberti
ndërtoi për herë të parë një sietem aksiomatik të qëndrueshëm, të plotë
dhe të pavarur, me të cilin mund të bazohet jovetëm gjeometria euklidiane,
por edhe gjeometritë joeuklidiane (e Llobaçevskit dhe Riemannit). Për
dallim nga Euklidi, Hilberti nuk i përkufizon kuptimet themelore. Në fillim
të librit të tij thuhet:
“Konsiderojmë tri kategori të ndryshme objektesh: Objektet e
kategorisë të parë i quajmë pika dhe i shënojmë me germa A,B,C,…;
objektet e kategorisë së dytë i quajmë drejtëza dhe i shënojmë me a,b,c,…;
objektet e kategorisë së tretë i quajmë rrafshe dhe i shënojmë me , ,
,…; pikat quhen elemente te gjeometrisë lineare, pikat dhe drejtëzat
elemente të gjeometrisë në rrafsh, ndërsa pikat, drejtëzat dhe rrafshet
quhen elemente të gjeometrisë në hapësirë. Pikat, drejtëzat dhe rrafshet
ndërjet tyre ndodhen në relacione të ndryshme që do ti shprehim me fjalët:
“i takon”, “ndërmjet”, “kongruent”, “paralel”, “i vazhdueshëm”;
përkufizimi i plotë matematik i këtyre relacioneve arrihet me anë të
aksiomave, që do ti ndajmë në pesë grupe:
I 1-8 aksiomat e lidhjes,
II 1-4 aksiomat e renditjes,
III 1-5 aksiomat e kongruencës,
IV aksioma e paraleleve ,
V 1-2 aksiomat e vazhdueshmërisë.”

1.5. Metodat kryesore për ndërtimin e gjeometrisë

Edhe gjeometria bazohet në të njëjtën mënyrë si edhe çdo disiplinë tjetër


matematike. Ekzistojnë tri metoda kryesore për ndërtimin e saj:
(i) Ndërtimi i gjeometrisë me anë të hapësirës vektoriale;
I Hyrje 17

(ii) Ndërtimi i gjeometrisë me anë të grupit;


(iii) Ndëtimi sintetik i gjeometrisë.

Po i themin nga disa fjalë për secilën prej këtyre metodave.

(i) Nërtimi i gjeometrisë me anë të hapësirës vektoriale


Themelues i kësaj metode është matematikani gjerman Hermann Ëeyl
(1885-1955). Për ndërtimi i gjeometrisë sipas kësaj metode merren dy
bashkësi joboshe P dhe X. Elementet e P quhen pika, ndërsa elementet e
X quhen vektorë. Bashkësia X pasurohet me strukturën algjebrike të
hapësirës vektoriale mbi ndonjë fushë të renditur F, që zakonisht merret
fusha R e numrave realë. Objekte themelore, pra, janë pika dhe vektori,
kurse relacione themelore: mbledhja e vektorëve, shumëzimi i vektorit me
scalar, produkti scalar i dy vektorëve dhe lidhja e vektorit me çiftin e
renditur të pikave. Objektet themelore në lidhje me relacionet themelore e
plotësojnë kërkesën e 16 aksiomave që ndahen në katër grupe:
I Aksiomat e hapësirës vektoriale (8),
II Aksiomat e dimenzionit të hapësirës (2),
III Aksiomat e produktit scalar (4),
V Aksiomat e lidhjes (2).

I Aksiomat e hapësirës vektoriale


1. Për  x,y  X, x + y = y + x .
2. Për  x,y z  X, (x + y) + z = x + (y + z) .
3.  0  X, ashtu që x + 0 = x, për  x  X .
4. Për  x  X,  y  X, ashtu që x + y = 0 .
5. 1 x = x, ku 1 është elementi njësh i fushës F .
6. a (b x) = (ab) x, për  a, b  F dhe  x  X .
7. a (x + y) = a x + b x , për  a  F dhe  x, y  X .
8. (a + b) x = a x + b x , për  a, b  F dhe  x  X .
18 I Hyrje

II Aksiomat e dimenzionit (bëhet fjalë për hapësirën vektoriale


tridimensionale)
1. Ekziston një treshe linearisht e pavarur vektorësh.
2. Çdo katër vektorë janë linearisht të varur.

III Aksiomat e produktit scalar


Produkti ◦ : X  X  F i plotëson këto aksioma:
1. x ◦ y = y ◦ x , për  x, y  X .
2. (x + y) ◦ z = x ◦ z + y ◦ z , për  x, y, z  X .
3. (a x) ◦ y = a (x ◦ y), për  a  F dhe  x, y  X .
4. a ◦ a > 0 . a ◦ a = 0  a = 0 .

IV Aksiomat e lidhjes
1. Për çdo pikë A dhe çdo vektor x, ekziston vetëm një pikë B, ashtu që
vektori AB te jete i barabartë me vektorin x.
2. Për çdo tri pika A, B, C vlen relacioni AB + BC + CA = 0 .

Bashkësia e të gjitha rrjedhimeve të sistemit aksiomatik


S = { I1-8, II1-2, III1-4, IV1-2}
e quajmë Gjeometri euklidiane.

(ii) Ndërtimi i gjeometrisë në bazë të teorisë së grupit


Themelues i kësaj metode është matematicienti gjerman Felix Klein
(1849-1925). Për ta ndëtuar gjeometrinë sipas kësaj metode merret një
bashkësi joboshe S e cila quhet hapësirë, kurse elementet e saj pika. Çdo
nënbashkësi e S quhet figurë.Shikohet një nëngrup  i grupit të
bijeksioneve Aut S të bashkësisë S. Në bashkësinë  = { F| F  S } të të
gjitha figurave të hapësirës S përkufizohet një relacion binar  në këtë
mënyrë:
F  F’  në qoftë se    , F = F’ .
I Hyrje 19

Pohimi 1. Relacioni  është rekuivalencë.


Vërtetim. Me qenë se grupi  e përmban identitetin 1 = idS të
bashkësisë S dhe (F)1 = F, për F, rrjedh se relacioni  është
refleksiv. Po të jetë F  F’, atëherë   , ashtu që F=F’. Me që
edhe -1  dhe -1 : F’  F rrjedh F’  F. Së fundi, po të jetë F  F’
& F’  F’’, atëherë do të ekzistojnë ,  , ashtu që F=F’ & F’=F’’.
Me qenë se  dhe (F) = (F’) = F’’  F  F’’ . □
Veti gjeometrike të grupit  quhen të gjitha veti të figurave të
bashkësisë S që mbeten të pandryshuara (invariante) ndaj të gjithë
elementeve të grupit . Bashkësia e të gjitha pohimeve mbi vetitë
gjeometrike të grupit  quhet gjeometri e grupit . Simbolikisht këtë
gjeometri do ta shënojmë me G(  ). Gjeometrria euklidiane është teoria e
invariantëve të grupit të izometrive te hapësires S.
Sipas këti koncepcioni do të kemi po aq gjeometri të hapësirës S sa edhe
grupe transformimesh të asaj hapësire. Le të jenë  dhe ’ dy grupe
transformimesh të hapësirës S, ndërsa G(  ), G( ’ ) gjeometritë
përkatëse të tyre. Vlen ky pohim:
Pohimi 2. Në qoftë se  < ’, atëherë G( ’ )  G(  ) .
Vërtetim. Po të jetë p  G( ’ ) cili do pohim i gjeometrisë të grupit ’,
atëherë p = p për   ’ (lexo: p është invariant ndaj të gjithë
elementeve të grupit ’). Me qenë se  < ’, çdo elemente  i grupit 
është element edhe i grupit ’, prandaj p = p, rrjedhimisht p  G(  ).

Kështu vërtetuam se sa më i “vogël” të jetë grupi i trnsformimeve, aq më
e pasur do të jetë gjeometria përkatëse e tij, dhe anasjelltas. Kështu, psh.
po të shënojmë me , A , me rradhë, grupin e transformimeve izometrike
dhe grupin e transformimeve afine të hapësirës S, atëherë  < A , prej ku
vimë në përfundim se gjeometria euklidiane G(  ) e përmban
gjeometrine afine G(A ).

(iii) Ndërtimi sintetik i gjeometrisë


Themelues i kësaj metode është Hilberti, sipas të cilit, siç e thamë më
sipër merren tri kategori objektesh themelore: pika, drejtëza dhe rrafshi,
dhe tri kategori relacionesh themelore: Incident, ndërmjet dhe
congruent. Përkufizimi më i plotë i tyre bëhet me sistemin e 20
20 I Hyrje

aksiomave, të cilat, varësisht nga relacioni që e përcaktojnë, ndahen në


pesë grupe:
I Aksiomat e incidencës (8),
II Aksiomat e renditjes (4),
III Aksiomat e kongruencës (5),
IV Aksiomat e vazhdueshmërisë (2),
V Aksioma e paralelshmërisë (1).
Bashkësia e kuptimeve fillestare është
K={pikë, drejtëz, rrafsh, incident, ndërmjet, kongruent},
kurse sistemin e aksiomave do ta shënojmë me
AE = { I1-8, II1-4, III1-5, IV1-2, V1}.
Po ta shënojmë me SE =( K, AE ) bërthamën e sistemit aksiomatik, atëherë
bashkësinë G( SE) të të gjitha rrjedhimeve të sistemit SE e quajmë
Gjeometri euklidiane, te cilën shkurt do ta shënojmë me GE. Bashkësia e
pikave, drjtëzave dhe rrafsheve që në lidhje me relacionet themelore
incident, ndërmjet dhe kongruen e plotësojnë kërkesën e aksiomave AE
quhet Hapësirë euklidiane. Po të marrim vetëm bashkësinë
AA= AE – {V1} e katër grupeve të para të aksiomave, atëherë bashkësinë
e rrjedhimeve të sistemit SA = (K, AA) e quajmë gjeometri absolute.

Alfabet quhet çdo bashkësi e fundme elementesh(simbolesh).Vargje të


fundme të alfbetit quhen fjalë të atij alfabeti.
Fjalët që kanë kuptime të caktuara në një disiplinë shkencore quhen terma.
Terma gjeometrike janë, psh. Drejtëza, rrafshi, këndi, rrethi, elipsa,kubi,
sfera, etj.
Shumicën e këtyre termave mund ta përkufizojmë me terma paraprake.
P.sh. Sfera është bashkësi e të gjitha pikave në hapësirë që kanë distancë
të njëjtë prej një pikë të saj.
Termi sferë është dhënë me anë të termave: bashkësi, pikë, hapësirë,
distancë që janë përkufizuar më parë. Por edhe këta terma janë përkufizuar
me terma paraprake. Po të vazhdojmë në këtë mënyrë do të arrijmë në një
numër të caktuar temash që ose duhet ti marrim pa përkufizim ose duhet
të sillemi në rreth vicioz me përkufizime. Me qenë se nuk është e kuptimtë
të sillemi në rreth, atëherë një numër të termave i marrim pa përkufizim.
Termat të tillë quhen terma fillestare ( terma themelore, primitive, të para,
të papërkufizuara).
I Hyrje 21

Thënjet (fjalitë) e kuptimta deklarative që kanë një vlerë saktësie (ose janë
të sakta ose janë të pasakta) quhen gjykime ose propozime. Gjykimet e
sakta quhen pohime.
Shuma e këndeve të brendshme të trekëndëshit është 2d, është pohim i
Gjeomerisë euklidiane,
Këndi periferik mbi diametrin e rrethit është i ngushtë është gjykim jo i
saktë në Gjeometrinë elementare, kurse gjykim i saktë në Gjeometrinë
hiperbolike.

Disiplina shkencore matematike mund të konsiderohet si bashkësi


e pohimeve që i karakterizojnë termat e saj. Të bazohet një disiplinë
matematike do të thotë që të gjitha termat dhe të gjitha pohimet e saj duhet
të sistematizohen në një varg logjik, ashtu që çdo term i ri të përkufizohet
dhe çdo pohim i ri të vërtetohet. Vërtetim i një pohimi quhet bashkësia e
të gjitha thënjeve që e arsyetojnë saktësinë e tij. Pohimet saktësia e të
cilëve vërtetohet quhen teorema (thënje të vërtetuara apo njohuri të
vërtetuara)
Çdo toeoremëmund të shprehet simbolikisht si p  q , ku gjykimi p quhet
hypotezë (premisë) e teoremës, kurse q quhet përfundim (konkludim) i
teoremës.
Vërtetimin e një teoreme e bëjmë me pohime të vërtetuara më parë, këto
gjithashtu me pohime para tyre.Duke vazhduar në këtë mënyrë do të vimë
në një numër të konsideruar pohimesh qoë ose do t’i vërtetojmë me pohime
që janë të varura prej tyre, ose do t’i pranojmë sit ë sakta pa vërtetim. Në
rastin e parë sillemi në rreth magjik, prandaj marrim pa vërtetim një numër
të konsiderueshëm të pohimeve. Pohimet e tilla i i quajmë pohime
fillestare ( pohime themelore, primitive, të para, aksioma apo postulate).
Aksiomat e përbëjnë bazën e një discipline shkencore.
22 I Hyrje

II Sitemi aksiomatik i Hilbertit. Gjeometria


absolute.

Me lindjen e Gjeometrisë joeuklidiane, përpara më shumë


se një shekull e gjysmë, u ndie nevoja që edhe gjeometria euklidiane të
bazohet në sisteme më të rrepta logjike. Përpjekjet e para në këtë drejtim
nuk munguan. Matematicientët e gjysmës së dytë të shek.19, si Pasch,
Peano, Pieri, etj. u munduan të përmirësojnë mangësinë e përkufizimeve
gjeometrike nga koha e Euklidit dhe Arhimedit dhe të bëjnë plotësimin e
sistemit aksiomatik. Kështu, Pasch-i futi edhe katër aksioma për relacionin
“ndërmjet” me të cilin trajtohet gjeometria e renditjes. Për parimin e
qëndrueshmërisë të sistemit aksiomatik nuk bëhej fjale fare. Më 1899
botohet vepra e njohur Die grundlagen der Geometrie e Hilbertit, në të
cilin, për herë të parë jepet një sistem aksiomatik i plotë, i qëndrueshëm
dhe i pavarur për ndërtimin e gjeometrisë. Kjo vepër është botuar dhe
ribotuar në shumë gjuhë të botës dhe akoma edhe sot, pas më shumë se
100 vjetëve të botimit të saj, gjeometria elementare kryesisht bazohet sipas
këti sistemi apo ndonjë sistemi ekuivalent me të. Edhe ne në këtë tekst do
ta ngremë ndërtesën e gjeometrisë sipas këti sistemi.
Për këtë qëllim do të shikojmë një bashkësi P , dy klase L
, R nënbashkësish të P dhe dy relacione  dhe , me rradhë të gjatësisë tre
dhe katër, në bashkësinë P. Bashkësinë P do ta quajmë hapësirë, kurse
elementet e saj pika dhe do ti shënojmë me A, B, C,… . Elementet e klasës
L do ti quajmë drejtëza dhe do ti shënojmë me a, b, c, … . Elementet e
klasës R do ti quajmë rrafshe dhe do ti shënojmë me germa të vogla të
alfabetit grek , , , … .
Relacionin  do ta quajmë relacion ndërmjet apo relacion të renditjes
të pikave në drejtëz. Formulën (A,B,C) (apo (A,B,C) ) do ta lexojmë:
Pika B ndodhet ndërmjet pikave A dhe C.
Relacionin  do ta quajmë relacion i kongruencës, kurse
formulën (A,B,C,D) (apo (A,B,C,D) )do ta lexojmë: Çifti i pikave
(A,B) është congruent me çiftin e pikave (C,D) dhe në vend të (A,B,C,D)
do të shkruajmë (A,B) (C,D).
Bashkësia e kuptimeve fillestare është K={ pikë, drejtëz, rrafsh,
ndërmjet, kongruent}.
I Hyrje 23

Çdo nënbashkësi F të P-së do ta quajmë figurë. Figurat F, F’ do ti


quajmë të njëjta dhe do ti shënojmë F = F’, në qoftë se bashkësitë F, F’
janë të barabarta. Përndryshe do të themi se F, F’ janë figura të ndryshme
dhe do ti shënojmë F  F’. Për figurat që kanë prerje joboshe do të themi
se priten. Figurën F do ta quajmë planare (të rrafshtë), në qoftë se ekziston
rrafshi  i tillë që F  . Në qoftë se nuk ekziston një rrafsh i tillë figurën
do ta quajmë hapësinore. Figurën F do ta quajmë lineare, në qoftë se
ekziston drejtëza a, e tillë që F  a .
Pohimet fillestare do ti ndajmë në pesë grupe. Grupin e parë
do ta quajmë aksiomat e incidencës, sepse ne të karakterizohet relacioni i
përkatësisë dhe relacioni i përfshirjes në bashkësinë e pikave, drejtëzave
dhe rrafsheve. Në grupin e dytë, aksiomat e renditjes, përkufizohen vetitë
themelore të relacionit ndërmjet, ndërsa në grupin e tretë, aksiomat e
kongruencës karakterizohet relacioni themelor kongruent. Në grupin e
katërt do të marrim dy aksioma të vazhdueshmërisë, atë të Arhimedit dhe
atë të Kantorit, ndërsa në grupin e pestë do të marrim aksiomën e
paraleleve.
Do të shënojmë me Aa = { I, II, III, IV} dhe Sa = (K, Aa). Bashkësinë
T(Sa) të të gjitha rrjedhimeve të sistemit Sa do ta quajmë gjeometri
absolute dhe do ta shënojmë me Ga. Në Ga vërtetohet pohimi: Nëpër pikën
A  a, në rrafshin e tyre ekziston drejtëza b që kalon nëpër A dhe nuk e
pret drejtëzën a. Pohimet si: drejtëza e b është e vetme; dhe drejtëza b
nuk është e vetme nuk janë rrjedhime të gjeometrisë absolute.
Në qoftë se si aksiomë të grupit të pestë e marrim pohimin e
Pleyferit Ve: Drejtëza b është e vetme, atëherë bashkësinë e rrjedhimeve
T(Se), ku Se = (K, Ae) dhe Ae = Aa {Ve } do ta quajmë gjeometri
euklidiane dhe do ta shënojmë me Ge.
Në qoftë se sistemit Aa ia shtojmë aksiomën e Llobaçevskit Vh:
Drejtëza b nuk është e vetme, atëherë atëherë bashkësinë e rrjedhimeve
T(Sh), ku Sh = (K, Ah) dhe Ah = Aa  {Vh} do ta quajmë gjeometri
hiperbolike apo gjeometri joeuklidiane të Bolyai-Llobaçevskit dhe do
ta shënojmë me Gh.
Siç shihet, Aa = Ae  Ah , prandaj edhe Ga = Ge  Gh . (figura…)

2.1. Aksiomat e incidencës dhe rrjedhimet e para

Me këtë grup të aksiomave karakterizohet relacioni i përkatësisë dhe i


përfshirjes në bashkësinë e pikave,drejtëzave dhe rrafsheve. Të dy këta
24 I Hyrje

relacione në gjeometri do ti shprehim me të njëjtin emër: relacion i


incidencës. Formulat

A  a ( A  a ), A (Aa);

a (a  )
në gjeometri do ti lexojmë edhe: Pika A ndodhet (nuk ndodhet) në
drejtëzën a (rrafshin ), apo drejtëza a (rrafshi ) kalon (nuk kalon) nëpër
pikën A; drejtëza a ndodhet (nuk ndodhet) në rrafshin , ose rrafshi 
kalon nëpër drejtëzën a. Si sinonime për këto relacione do të përdorim
edhe: pika është incidente me drejtëzën (rrafshin), drejtëza është incidente
me rrafshin, ose drejtëza (rrafshi) është incidente me pikën, apo rrafshi
është incident me drejtëzën.

Për tri e më shumë pika incidente me një drejtëz do të themi se janë


pika kolineare. Përndryshe do te themi se ato janë jokolineare. Për katër e
më shumë pika që janë incidente me një rrafsh do të themi se janë pika
koplanarë. Përndryshe do të themi se ato janë jokoplanare.

Drejtëzat jokoplanare do ti quajmë drejtëza të kithta. Dy apo


mëshumë drejtëza që janë incident me të njëjtën pikë do ti quajmë drejtëza
(rrafshe) konkurente. Dy apo më shumë rrafshe që janë incidentë me të
njëjtën drejtëz do ti quajmë rrafshe koaksialë.

Aksiomat e incidencdencës do ti formulojmë kështu:

I1. Klasët L dhe R janë joboshe.

I2. Drejtëzat dhe rrafshet janë nënbashkësi të mirëfillta të P.

I3. Çdo drejtëz është incidente të paktën me dy pika.

Nga aksiomat I1-3 rrjedh se P përmban të paktën tri pika jokolineare:


Dy pika të ndonjë drejtëze a (një drejtëz e tille nga I1 ekziston) dhe një
pikë që nuk i takon asaj drejtëze (sepse drejtëza nuk i përmban të gjitha
pikat e P).

I4. Për çdo dy pika të ndryshme ekziston një dhe vetëm një drejtëz
incidente me secilën prej tyre.
I Hyrje 25

Drejtëzën të përcaktuar nga pikat A,B do ta shënojmë AB , ose a(AB)


, në qofttë se dëshirojmë të theksojmë se a është drejtëza që i përmban
pikat A,B.

I5. Për çdo tri pika jokolineare ekziston një dhe vetëm një rrafsh incident
me secilën prej tyre.

Rrafshin e përcaktuar nga pikat jokolineare A,B,C do ta shënojmë ABC


ose (ABC) kur dëshirojmë të themi se  është rrafshi i pikave A, B, C.

Nga aksiomat I1-5 rrjedh se hapësira P ka të paktën katër pika


jokoplanare: Konstatuam se P kishte tri pika jokolineare të cilat, nga I5, e
përcaktojnë një rrafsh  jashtë të cilit ekziston të paktën një pikë (sepse 
nuk i përmban të gjitha pikat e P).

Me përkufizim do të themi se drejtëza a është incidente me rrafshin , në


qoftë se çdo pikë e drejtëzës a është incidente me rrafshin .

I6. Në qoftë se dy pika të ndryshme të një drejtëzë janë incidente me një


rrafsh, atëherë ajo drejtëz është incidente me atë rrafsh.

I7. Në qoftë se dy rrafshe janë incidentë me një pikë, atëherë ata janë
incidentë edhe me një pikë.

Bashkësia e të gjitha rrjedhimeve të këti grupi aksiomash quhet gjeometri


e incidencës, të cilën do ta shënojmë me GI . Aksiomat I1-4 quhen aksioma
të gjeometrisë në rrafsh (planimetrisë).

Po i japim tani pohimet më kryesore të GI .

P.1. Në qoftë se tri pika janë jokolinere, çdo dy prej tyre janë të ndryshme.

Vërtetim. Le të jenë A,B,C pika jokolineare. Çdo dy prej tyre janë të


ndryshme, sepse sikur dy prej tyre, psh. pikat A,B të ishin të njëjta, atëherë
B = A  a(AC) . □

P.2. Në qoftë se katër pika janë jokoplanare, çdo dy prej tyre janë të
ndryshme. □

P.3. Në qoftë se A,B  a dhe C  a, atëherë A, B, C janë jokolineare.


26 I Hyrje

Vërtetim. Sikur të ekzitonte ndonjë drejtëzë b, A,B,Cb, atëherë a


=AB = b, rrjedhimisht C a. □

P.4. Në qoftë se A,B,C   dhe D , atëherë A,B,C,D janë pika


jokoplanare.□

P.5. Ekziton më së shumti një pikë incidente me dy drejtëza të ndryshme.

Vërtetim. Le të jenë a,b dy drejtëza të ndryshme.Atëherë a  b =  ose


a  b   . Po të ekzistojë pika A  a, b , atëherë ajo është e vetme, se
po të ekzistonte edhe pika B  a, b, B  A, atëherë a = AB = b, që nuk
është e mundur. □

P.6. Pika dhe drejtëza joincidente me të përcaktojnë një dhe vetëm një
rrafsh.

Vërtetim. Le të jetë Aa. Nga I3,  B,Ca, kurse nga P.3. pikat A,B,C
janë jokolineare. Nga aksioma I5 ato përcaktojnë një dhe vetëm një rrafsh
(A,B,C), të cilit nga I6 i takon drejtëza a(BC). □

P.7. Dy drejtëza që priten përcaktojnë një dhe vetëm një rrafsh

Vërtetim. Le të jenë a,b drejtëza që priten në pikën M. Me qenë se çdo


drejtëz ka të paktën dy pika, ekzistojnë pikat A  a dhe B  b, A, B 
M . Pikat A,B,M janë jokolineare, sepse sikur të ekzistonte drejtëza c 
A,B,M , atëherë do të ishte a =c(AM) = c(BM) = b. Po të shënojmë me
(ABM), atëherë nga I6, a,b  . Po të ekzistonte edhe ndonjë rrafsh tjetër
 që i përmban drejtëzat a,b , atëherë  do ti përmbante edhe pikat A,B,M,
prandaj do të ishte i njëjtë me rrafshin . □

Rrafshi të përcaktuar nga drejtëzat a,b do ta shënojmë rr(ab).

P.8. Ekziston të paktën një rrafsh incident me drejtëzën e dhënë a.

Vërtetim. Me qenë se drejtëza a nuk i përmban të gjitha pikat, ekziston të


paktën një pikë A  a dhe nga P.6. ekziton rrëfshi (aA) që e përmban a-
në. □

P.9. Në qoftë se katër pika A,B,C,D janë jokoplanare, çdo tri prej tyre janë
jokolineare.
I Hyrje 27

Vërtetim. Sikur pikat A,B,C,D të ishin kolineare, nga P.8., ato do te ishin
koplanare, që nuk janë. Po të ishin tri prej tyre, psh. pikat A,B,C kolineare,
atëherë drejtëza që i përmban ato dhe pika D do te përcaktonin një rrafsh
 incident me pikat A,B,C,D, që nuk është e mundur. □

P.10. Dy rrafshe të ndryeshëm ,  ose s’kanë pika të përbashkëta, ose


kanë vetëm një drejtëz të përbashkët.

Vërtetim. Rrafshet ,  ose janë disjunkt ose kanë prerje joboshe. Po të


jetë A  , , nga I7, do të ekzistojë edhe B  , . Por atëherë edhe
drejtëza a(AB)  , . Po të ekzistonte edhe ndonjë pikë tjetër C
 , e tillë që Ca, atëherë  = ABC = , që s’është e mundur, sepse ,
janë të ndyshëm. □

Rrafshet disjunkt do ti quajmë rrafshe joprerës, kurse për rrafshet që


kanë drejtëz të përbashkët do të themi se priten.

P.11. Drejtëza a joincidente me rrafshin  ka më së shumti një pikë të


përbashkët me të.

Vërtetim. Me qenë se a  , atëherë a   është boshe ose joboshe. Në


rastin e dytë sikur të ekzistonin dy pika A,B  a  , atëherë a = AB 
, që nuk është e mundur.□

Në rastin kur a   =  themi se drejtëza a nuk e prêt rrafshin . Në


qoftë se a   = {A} themi që drejtëzza a e depërton rrafshin . Pika A
quhet pike e depërtimit të drejtëzës a në rrëfshin .

P.12.( Teorema themelore) Ekzistojnë të paktën katër pika të ndryshme,


gjashtë drejtëza të ndryshme dhe katër rrafshe të ndryshme.

Vërtetim. Nga I1-5 rrjedh se ekzistojnë katër pika A,B,C,D, çdo tri prej të
cilave janë jokolineare. Nga P.9 rrjedh se A,B,C,D janë katër pika të
ndryshme. Por atëherë nga I4 dhe I5 ekzistojnë edhe drejtëzat AB, AC,
AD, BC, BD, CD dhe rrafshet ABC, ABD, ACD, BCD. Në qoftë se dy
prej këtyre drejtëzave dhe rrafsheve do të ishin të njëjtë, atëherë pikat
A,B,C, D do të ishin koplanare, që nuk janë. □

P.13. Në qoftë se A,B,C,D janë katër pika jokoplanare dhe E pikë e


ndryshme prej tyre,atëherë pika E i takon më së shumti dy prej rrafsheve
ABC,ABD,ACD,BCD.
28 I Hyrje

Vërtetim. Supozojmë se E i takon tre rrafsheve (ABC), (ABD),


(ACD). Nga se E i takon dy rrafsheve të parë rrjedh se pikat A,B,E janë
kolineare, kurse përshkak se E i takon (ABC) dhe Y(ACD) rrjedh se
pikat A,C,E janë kolineare. Pikat B,C i takojnë drejtëzës AE,
rrjedhimisht A,B,C, D do të jene koplanare, që nuk është e mundur. □

P.14. Çdo rrafsh  ka të paktën tri pika jokolineare.

Vërtetim. Nga I2 Ekziston të paktën një pikë A   dhe të paktën një


pikë B  . Shënojmë me a(AB). Nga I2 C  a, kurse nga I5 !
(ABC)  . Me qenë se A  ,   ekziston edhe një pikë D  , 
dhe D  A. Prap tani nga I2,  pika E   dhe (ABE)  . Rrafshet , 
janë incident me pikën A  ata janë incident edhe me një pikë F, F  A.
Pikat F, D janë të ndryshme, sepse sikur F = D do të kishim  = ABD =
ABE = , që s’është . Pikat A,D,E janë jokolineare, sepse sikur A,D,E 
b, prap do të kishim  = Bb = . Pra, në  ekzistojnë të paktën tri pika
jokolineare A,D,F. □

2.2. Disa modele të gjeometrisë të incidencës

1. Le të jetë S = {1,2,3,4} një bashkësi me katër elemente. Do të


quajmë “pikë“ çdo elemente të bashkësisë S, do të quajmë “drejtëzë” çdo
nënbashkësi dyelementëshe të S dhe “rrafsh” çdo nënbashkësi
trielemntëshe të S. “Incidenca” shprehet në kuptimin e zakonshëm në
teorinë e bashkësive. Pra,

P = S, L = {{1,2}, {1,3}, {1,4}, {2,3}, {2,4}, {3,4}} dhe R =


{{1,2,3}, {1,2,4}, {1,3,4}, {2,3,4}}.

Detyrë. Provoni se objektet themelore pikë, drejtëz, rrafsh dhe incidenca


të interpretuara në këtë mënyre i plotësojnë aksiomat I1, …, I7 .
I Hyrje 29

Këto katër pika, gjashtë drejtëza dhe katër rrafshe, sebashku me


relacionin e incidencës formojnë nje model të gjeometrisë të incidencës.
Në gjeometrinë e incidencës nuk mund të vërtetojmë ekzistencën e pesë
pikave të ndryshme, shtatë drejtëzave të ndryshme dhe pesë rrafsheve të
ndryshëm.

2. Konsiderojmë një tetraedër T1T2T3T4 . (figura). Do të


quajmë pikë- çdo kulm të tetraedrit, drejtëz- çdo teh të tij dhe rrafsh- çdo
faqe të tetraedrit. Incidenca: Pika është incidente me drejtëzën  në se
kulmi i takon tehut, pika është incidente me rrafshin  në se kulmi i
takon faqes. Formuloni aksiomat I1-7 dhe verifikoni saktësinë e tyre.

3. Në figurën e tetraedrit do të ndërtojmë një model tjetër të


gjeometrisë së incidencës. Do të quajmë pika-faqet e tetraedrit, drejtëza-
brinjët e tetraedrit dhe rrafshe- kulmet e tetraedrit. Incidenca: Pika është
incidente me drejtëzën  në se faqja e përmban tehun; pika është
incidente me rrafshin  në se faqja e përmban kulmin. Formuloni
aksiomat I1-7 dhe verifikoni saktësinë e tyre.

4. Objektet pikë, drejtëz, rrafsh le të jenë si në modelin 2, kurse


incidencën do ta interpretojmë kështu: Pika është incidente me drejtëzën
 në se kulmi nuk i takon tehut; pika është incidente me rrafshin  në
se kulmi i takon faqes; drejtëza është incidente me rrafshin  në se tehu
nuk i takon faqes. Verifikoni saktësinë e aksiomave I1-7.

5. Modeli arithmetik apo modeli dekartian.

Së fundi do të ndërtojmë një model të gjeometrisë të incidencës ne


arithmetikë. Për thjeshtësi do ta ndërtojmë vetëm modelin arithmetik te
gjeometrisë në rrafsh. Për këtë qëllim, pikë do të quajmë çiftin e renditur
(x,y) të numrave realë, kurse drejtëz raportin e një tresheje të renditur
(A:B:C) të numrave reale me kusht që A2  B 2  0 . Pika (x,y) është
incidente me drejtëzën (A:B:C), në qoftë se plotësohet barazimi

Ax  By  C  0 .

Vërtetojmë tani aksiomën: Dy pika të ndryshme ( x1 , y1 ) dhe ( x2 , y2 )


përcaktojnë një dhe vetëm një drejtëz incidente me secilën prej tyre. Me
qenë se pikat janë të ndryshme, së paku njëri nga numrat
30 I Hyrje

A  y1  y2 , B  x2  x1 është i ndryshueshëm nga zero, prandaj kushti


A2  B 2  0 plotësohet. Po të shënojmë me C  x1 y2  x2 y1 , atëherë

Ax1  By1  C  ( y1  y2 ) x1  ( x2  x1 ) y1  x1 y2  x2 y1  0

Ax2  By2  C  ( y1  y2 ) x2  ( x2  x1 ) y2  x1 y2  x2 y1  0 ,

Pra, për pikat e dhëna ( x1 , y1 ) dhe ( x2 , y2 ) ekziston drejtëza (A:B:C)


incidente me secilën prej tyre. Po të ekzistoje edhe ndonjë drejtëz tjetër
(A’:B’:C’) incidente me pikat ( x1 , y1 ) dhe ( x2 , y2 ) , atëherë për çdo pikë
(x, y) që i takon drejtëzave në fjalë kemi

Ax  By  C  0

A' x  B ' y  C '  0 .

Me qenë se sistemi i mësipërm ka dy zgjidhje ( x1 , y1 ) dhe ( x2 , y2 ) , nga


algjebra lineare gjemë se koeficientët A, B, C dhe A ', B ', C ' janë
proporcionalë, prandaj (A:B:C) dhe (A’:B’:C’) është e njëjta drejtëz.

Të vërtetojmë tani aksiomën: Çdo drejtëz është incidente sëpaku me dy


pika. Ekzistojnë të paktën tri pika joincidente me një drejtëz. Vërtet, po të
jetë (A:B:C) një drejtëz e dhënë, atëherë ekuacioni Ax  By  C  0 ka
pambarimisht shumë zgjidhje, që do të thotë se drejtëza (A:B:C) është
incidente jo vetëm me dy pika, por me pambarimisht shumë pika. Pikat
(0,0), (0,1) dhe (1,0) janë jokolineare, sepse sikur të ekzistonte drejtëza
(A:B:C) incidente me secilën prej tyre, do të kishim

A0  B 0  C  0
A  0  B 1  C  0
A 1  B  0  C  0

që plotësohet vetëm për A  B  C  0 . Por në këtë rast (A:B:C) nuk


paraqet drejtëz, sepse nuk plotësohet kushti A2  B 2  0 .
I Hyrje 31

GJEOMETRIA E RENDITJES

Renditja në gjeometri futet në shekullin 19 pas vërejtjeve të Gaussit se për


ndërtimin rigoroz të gjeometrisë disa veti më të thjeshta të renditjes së
pikave në drejtëz duhen të merren si aksioma. Siç e dimë te Elementet e
Euklidit ky kuptim merret si evident duke u mbështetur në krahasimin e
segmenteve. Për here të pare aksiomat e renditjes në gjeometri i fut
matematicienti gjerman M. Pasch më 1882, në veprën ”Leksione të
gjeometrisë së re”, kurse më vonë, Peano dhe Hilberti e përmirësojnë
sistemin aksiomatik të tij. Renditjen e pikave në drejtëz Hilberti e bën me
anë të relacionit themelor “ndërmjet” për tri pika të ndryshme të një
drejtëze. Faktin se pika B ndodhet ndërmjet pikave A dhe C do ta shënojmë
B (A,B,C).

1. Aksiomat e renditjes dhe rrjedhimet e para

Me aksiomat e renditjes karakterizohet kuptimi themelor ”ndërmjet” si


relacion ternar në bashkësinë e pikave kolineare.

Aksiom II1. Në qoftë se B (A,B,C), atëherë A, B, C janë tri pika të


ndryshme kolineare dhe gjithashtu B (C,B,A).
Aksioma II2. Në qoftë se B (A,B,C), atëherë nuk është B (A,C,B).
Aksioma II3. Për çdo dy pika A dhe B ekziston pika C ashtu që B
(A,B,C).
Aksioma II4. Në qoftë se A, B, C janë tri pika të ndryshme kolineare,
atëherë B (A,B,C) ose B (B,C,A) ose B (C,A,B).
Aksioma II5. (M. Pasch) Le të jenë A, B, C tri pika jokolineare dhe p
drejtëz në rrafshin e tyre, joincidente me pikën A. Në qoftë se ekziston
pika Dp, B (B,D,C), atëherë ekziston edhe pika Ep, ashtu që B
(A,E,B) ose B (A,E,C).

Katër aksiomat e para janë pohime të gjeometrise në drejtëz, prandaj dhe


quhen aksioma lineare të renditjes, ndërsa aksioma II5 është pohim i
gjeometrisë në rrafsh. Aksiomat lineare nuk janë të mjaftueshme për
gjeometrinë e renditjes në drejtëz, por duhet të merren edhe aksioma të
tjera të gjeometrisë lineare. Por me qenë se këto janë rrjedhime të aksiomës
së Pasch-it, nuk i marrim si aksioma.
32 I Hyrje

Për tri pika kolineare A, B, C aksioma II4 siguron ekzistencën e së paku


njërit prej relacioneve B (A,B,C), B (B,C,A), B (C,A,B). Do të vërtetojmë
se vlen pikërisht njëri prej tyre.

Pohimi 1. Nëse A, B, C janë tri pika të ndryshme kolineare, atëherë vlen


vetëm njëri nga relacionet :
B (A,B,C), B (B,C,A), B (C,A,B).

Vërtetim. Pa e cënuar përgjithësimin e vërtetimit, mund të supozojmë se B


(A,B,C).
Atëherë, nga II2, nuk është B (A,C,B) dhe nga II1, nuk është B (B,C,A).
Nga B (A,B,C) dhe II1 kemi B (C,B,A) dhe përsëri nga II2 nuk është B
(C,A,B). □

Si rrjedhim të pohimit 1 tani bashkësinë e pikave të një drejtëze do ta


karakterizojmë me një pohim që në literaturë njhet si pohimi i parë i
Peanos.
Pohimi 2. (Pohimi i parë i Peanos) Nëse A,B janë dy pika të drejtëzës p,
atëherë drejtëza p përputhet me bashkësinë p’ që përbëhet nga pikat A,B
dhe të gjitha pikat Xp që e plotësojnë ndonjërin nga relacionet

B(A,X,B), B(X,A,B), B(A,B,X). (1)

Vërtetim. Nga përkufizimi i bashkësisë p’ kemi p’  p . Le të jetë Xp


cila do një pikë. Nëse X=A ose X=B, atëherë Xp’. Nëse X  A & X  B
, atëherë A, B, X do të jenë tri pika të ndryshme të p, dhe nga pohimi 1,
do të ketë vend vetëm njëri nga relacionet (1), prandaj X  p ' dhe për
rrjedhim p  p ' . □

Aksioma e Paschit pohon se drejtëza p e pret së paku njërën prej drejtëzave


AB dhe AC. Do të vërtetojmë tani se p e pret vetëm njërën prej tyre.

Pohimi 3. Në qoftë se A,B,C janë tri pika jokolineare, kurse X,Y,Z pika të
tilla që
B (B,X,C), B (C,Y,A), B (A,Z,B), atëherë X,Y,Z janë pika jokolineare.

oD

C o
B
o o o
A X
oE
I Hyrje 33

oA

Y
o
Z o
o o o
B X C

Vërtetim. Nga B (C,Y,A), B (A,Z,B) dhe II1 rrjedh se Y, Z janë pika të


ndryshme nga pika A dhe me që ato ndodhen në drejtëza të ndryshme, ato
janë të ndryshme edhe ndërmjet veti. Pokështu vërtetohet se pika X është
e ndryshme edhe nga Y edhe nga Z. Sikur X,Y,Z të ishin kolineare, nga
pohimi 2 do të kishte vend vetëm njëri nga relacionet
B (X,Y,Z), B (Y,Z,X), B (Z,X,Y).
Pa e cënuar përgjithësimin e vërtetimit, mund të supozojmë se B (Z,X,Y).
Drejtëza BC i takon rrafshit të pikave A,Y,Z , nuk kalon nëpër pikën A, e
pret drejtëzën BC në pikën X ashtu që B (Z,X,Y), kurse drejtëzat AY dhe
AZ në pikat C dhe B, të tilla që nuk është B(Y,C,A) dhe nuk është
B(A,B,Z), që është në kundërshtim me aksiomën e Pasch-it. Pra, X , Y , Z
janë pika jokolineare. □

Nga Pohimi 3 rrjedh se pika E për të cilën bën fjalë aksioma e Pasch-it e
plotëson vetëm njërin prej relacioneve B (A,E,B) ose B (A,E,C).

Pohimi 4. Për çdo dy pika të ndryshme A, B ekziston pika X, ashtu që B


(A,X,B).

Vërtetim. Le të jetë C pikë joincidente me drejtëzën AB (ekzistencën e saj


e sigurojnë aksiomat e incidencës). Nga II3, ekzistojnë pikat D dhe E, ashtu
që B (A,C,D) dhe B (D,B,E). Nga aksiomat e incidencës rrjedh se pikat D
dhe E i takojnë rrafshit të pikave A, B, C . Me qenë se A, B, D janë tri pika
jokolineare dhe drejtëza CE, që nuk kalon nëpër A, e pret drejtëzën AD në
pikën C, ashtu që B (A,C,D) dhe drejtëzën BD në pikën E, ashtu që B
(D,B,E), nga Aksioma e Pasch-it rrjedh se CE e pret edhe drejtëzën AB në
një pikë X, ashtu që B (A,X,B).□
34 I Hyrje

Për bashkësinë e fundme të pikave kolineare A1 , A2 ,..., An , n  3 , themi


se është linearisht e renditur dhe e shënojmë B ( A1 , A2 ,..., An ), në qoftë se
B ( Ai , Aj , Ak ) , atëherë dhe vetëm atëherë kur 1  i  j  k  n .

Po të jetë
B ( A1 , A2 ,..., Ai , Ai 1 ,..., An )
atëherë, nga aksiomat e renditjes dhe nga përkufizimi i bashkësisë
linearisht të renditur, do të jetë edhe B ( An , An1 ,..., Ai 1 , Ai ,..., A1 ) ,
kurse nuk do të jetë B ( A1 , A2 ,..., Ai 1 , Ai ,..., An ) .

Në vazhdim do ti vërtetojmë disa teorema që kanë të bëjnë me renditjen e


katër pikave të një drejtëze.

Pohimi 5. Nëse B (A,B,C) dhe B (B,C,D), atëherë vlen B (A,B,C,D).

oF

o E
Ho
oG

o o o o
A B C D

Vërtetim. Ekzistojnë pikat E, F joincidente me drejtëzën e pikave A,B,C,D


ashtu që B (D,E,F) . Nga se B (B,C,D) dhe B (D,E,F) dhe aksioma II5 (për
pikat jokolineare B,D,E dhe drejtëzën CF) rrjedh B (B,G,E). Me që
B (A,B,C) dhe B (B,G,E) dhe II5 (për A,B,E dhe CF) kemi B (A,H,E). Tani
nga se B (A,H,E) dhe B (D,E,F) dhe aksioma II5 (për pikat A,D,E dhe
drejtëzën CF) kemi B (A,C,D).
Me qenë se B (B,C,D) dhe B (A,B,C), nga II1, kemi B (D,C,B) dhe B
(C,B,A) prej ku rrjedh B (D,B,A), gjegjesisht B (A,B,D). Pra vlen B
(A,B,C,D). □
I Hyrje 35

Pohimi 6. Nëse është B (A,B,C) dhe B (A,C,D), atëherë vlen B


(A,B,C,D).
Vërtetim. Le të jenë E, F pika jashtë drejtëzës të pikave A,B,C,D ashtu që
B (D,E,F). Nga se B (A,C,D) dhe B (D,E,F) dhe aksioma II5 (për pikat
A,D,E dhe drejtëzën CF) kemi B (A,H,E). Tani nga B (A,B,C) dhe B
(A,H,E) dhe II5 (për pikat A,B,E dhe CF ) kemi B (B,G,E). Dhe së fundi po
ta zbatojmë aksiomën II5 për pikat B,D,E dhe drejtëzën CF, nga B (B,G,E)
dhe B (D,E,F) rrjedh B (B,C,D).
Nga B (A,B,C) dhe B (B,C,D) dhe Pohimi 5 kemi B (A,B,C,D).□

Pohimi 7. Po të jetë B (A,B,C) dhe B(A,B,D) dhe C pikë e ndryshme nga


D, atëherë ose B (A,B,C,D) ose B (A,B,D,C).

Vërtetim. Me qenë se A,C,D janë tri pika të ndryshme jokolineare, atëherë


vlen vetëm njëri nga relacionet: B (A,C,D), B (C,D,A), B (D,A,C).
Nëse është B (A,C,D), nga B (A,B,C) dhe Pohimi 6 rrjedh B (A,B,C,D).
Nëse është B (C,D,A), atëherë është edhe B (A,D,C) dhe meqë është B
(A,B,D), prap nga pohimi 6 kemi B (A,B,D,C). Në fund le të jetë B
(D,A,C). Nga B (D,B,A) dhe B (D,A,C) dhe pohimi 6 kemi B
(D,B,A,C), pra B (B,A,C), që është në kundërshtim me supozimin e bërë.□

Pohimi 8. Po të jetë B (A,C,B) dhe B (A,D,B) dhe C,D pika të ndryshme,


atëherë vlen vetëm njëri nga relacionet: B (A,D,C,B) ose B (A,C,D.B).

Vërtetim. Si në vërtetimin e pohimit 6, për pikat A,C,D vlen vetëm njëri


nga relacionet: B (A,D,C), B (D,C,A), B (C,A,D). Nëse është B (A,D,C),
nga B (A,D,C) dhe B (A,C,B) kemi B (A,D,C,B). Po të jetë B (C,A,D), me
që vlen dhe B (B,C,A) , nga pohimi 5, rrjedh B (B,A,D), që kundërshton
supozimin. Dhe në fund, nëse është B (D,C,A), do të jetë edhe B (A,C,D)
dhe nga B (A,D,B) e pohimi 6 rrjedh B (A,C,D.B).□

2. Segmenti dhe vija poligonale


36 I Hyrje

Përkufizim. Le të jenë A,B dy pika të ndryshme. Bashkësinë e të gjitha


pikave ndërmjet pikave A dhe B do ta quajmë segment i hapur AB dhe do
ta shënojmë (AB). Pikat A dhe B do ti quajmë skaje apo anë të tij. (AB)
{ A, B} do ta quajmë segment i mbyllur dhe do ta shënojmë [AB].
Unionin e segmentit të hapur AB dhe njërit prej skajeve të tij do ta quajmë
segment gjysmë i hapur dhe do ta shënojmë [AB) apo (AB] varësisht nga
fakti se ai e përmban skajin A apo skajin B. Nëse nuk theksohet veçmas,
ne do ta konsiderojmë segmentin AB të mbyllur dhe thjesht do ta shënojmë
me AB.

Shënim. Te Euklidi kuptimi i segmentit nuk është përkufizuar për shkak


se në gjeometrinë e atëhershme, siç shihet edhe nga postulati i dytë,
drejtëza dhe segmenti e kishin të njëjtin funksion.

Nga pohimi 4 i pikës 1 rrjedh se segmenti i hapur është bashkësi jo boshe.


Do të vërtetojmë tani këtë pohim:

Pohimi 1. Segmenti i hapur AB është bashkësi e numërueshme pikash.

Vërtetim. Nga pohimi 4, ekziston pika C1 , B (A, C1 ,B), rrjedhimisht C1


(AB). Nga i njëjti pohim, ekziston pika C2 , B (A, C2 , C1 ).
Nga B (A, C2 , C1 ) dhe B (A, C1 ,B) dhe pohimi 6 rrjedh B (A, C2 ,B), pra
edhe C2 (AB). Kështu mund të gjeme sa të duam pika të segmentit hapur
AB. □

Po të jenë A, B dy pika të një drejtëze a, atëherë nga pohimi1 [AB]  a .


Prandaj, ka vend ky pohim:

Pohimi 2. Drejtëza, rrafshfshi dhe hapësira janë bashkësi të numërueshme


pikash. □

Si rrjedhim të pohimit 8 të pikës 1 tani e kemi këtë pohim:

Pohimi 3. Le të jetë C pikë e segmentit të hapur AB. Pika X e ndryshme


nga pika C i takon segmentit të hapur AB, atëherë dhe vetëm atëherë nëse
ajo i takon njërit prej segmenteve të hapur AC dhe BC. □

Pohimi 4. Le të jenë A,B,C tri pika jokolineare. Prerja e segmenteve të


hapur (AB) dhe (BC) është boshe, atëherë dhe vetëm atëherë nëse
I Hyrje 37

B (A,B,C).

Vërtetim. Nëse B (A,B,C), atëherë nga pohimi 2 kemi ( AB)  ( BC )   .


Po të jetë B (B,C,A), përsëri nga pohimi 2 kemi ( BC )  ( BA) , prandaj
( AB)  ( BC )  ( BC ) . Po kështu, nga B (C,A,B) rrjedh ( AB)  ( BC ) ,
gjegjësisht ( AB)  ( BC )  ( AB) . □

Pohimi 5. Le të jetë B ( A1 , A2 ,..., An ). Pika X e ndryshme nga pikat


A1 , A2 ,..., An i takon segmentit të hapur A1 An , atëherë dhe vetëm atëherë në
qoftë se i takon vetëm njërit prej segmenteve të hapur A1 A2 ,..., An1 An .

Vërtetimi bëhet duke e zbatuar pohimin 3 një numër të fundëm herësh. □

Nga pohimi 5 shohim se po të jetë B ( A1 , A2 ,..., An ), atëherë


( Ai Ai 1 ), i  1,2,..., n 1 është varg i fundëm i segmenteve të hapur që i
takojnë një drejtëze. Por vlen edhe pohimi i anasjelltë:

Pohimi 6. Në qoftë se ( Ai Ai 1 ), i  1,2,..., n  1 , n  2 , është varg i fundëm


i segmenteve të hapur të një drejtëze, atëherë
B ( A1 , A2 ,..., An ).

Vërtetim. Vërtetimi bëhet me metodën e induksionit matematik. Për n=3,


vërtetimi rrjedh nga pohimet 3 dhe 4. Supozojmë se pohimi është i saktë
për të gjithë numrat natyralë më të vegjël se n. Nga ky supozim kemi
B ( A1 , A2 ,..., An1 ) dhe B ( A2 , A3 ,..., An ).

Vërtetojmeë se B ( A1 , Ai , An ) për çdi i, 2  i  n  1 . Vërtet, nga


B ( A1 , Ai , An1 ) dhe B ( Ai , An1 , An ) dhe pohimi 5 i pikës 1 kemi
B ( A1 , Ai , An ), prej ku rrjedh B ( A1 , A2 ,..., An ).□

Të vërtetojmë tani këtë pohim:


38 I Hyrje

Pohimi 7. Çdo bashkësi e fundme S e pikave kolineare, S  3 , mund të


renditet linearisht në dy mënyra.

Vërteim. Vërtetimin e bëjmë me induksion matematik. Për S  3 vërtetimi


rrjedh nga pohimi1 i pikës së më parshme dhe aksioma 1. Supozojmë se
pohimi është i saktë për 3  S  m dhe vërtetojmë se ai është i saktë edhe
për S  {A1 , A2 ,..., Am1} . Le të jetë X  S dhe F  S  { X } . Nga hapi i
induksionit kemi se bashkësia F  {F1 , F2 ,..., Fm } mund të renditet
linearisht, pra vlen B ( F1 , F2 ,..., Fm ) .
Për pikat X , F1 , Fm vlen vetëm njëri nga relacionet: B ( X , F1 , Fm ) , B
( F1 , Fm , X ) , B ( F1 , X , Fm ) .
a) Në qoftë se B ( X , F1 , Fm ) , nga pohimi 4, kemi ( XF1 )  ( F1Fm )  
. Me qenë se ( XF1 ),( F1F2 ),...,( Fm1Fm ) janë segmente disjunkte, nga
pohimi 5 rrjedh B ( X , F1 ,..., Fm ) .
b) Në qoftë se B ( F1 , Fm , X ) , atëherë B ( X , Fm , F1 ) dhe nga rasti a) kemi B
( X , Fm ,..., F1 ) .
c)Në qoftë se B ( F1 , X , Fm ) , atëherë, nga pohimi 5, X i takon vetëm njërit
prej segmenteve të hapur ( F1F2 ),...,( Fm1Fm ) . Nëse X  ( Fi Fi 1 ) , atëherë
kemi B ( F1 ,..., Fi , X , Fi 1 ,..., Fm ) .
Kështu kemi vërtetuar se për S bashkësinë S  { A1 , A2 ,..., Am1} vlen B
( A1 , A2 ,..., Am1 ) , a prej kësaj edhe B ( Am1 , Am ,..., A1 ) .
Do të vërtetojmë tani se këto dy mënyrë janë të vetme. Le të jetë
p1 , p2 ,..., pm1 një permutacion i numrave 1,2,…,m+1.
Në qoftë se
B ( Ap1 , Ap2 ,..., Apm1 )
atëherë, meqë A1 , Am1 janë pikat e vetme të bashkësisë
S  { A1 , A2 ,..., Am1} ndërmjet të cilave nuk ndodhen pika të tjera, kemi
{ Ap1 , Apm1 }  { A1 , Am1} .
I Hyrje 39

Mund të supozojmë, pa e cënuar përgjithësimin e vërtetimit, se p1=1 dhe


pm 1 = m  1 . Me qenë se A2 është pika e vetme e S , e tillë që ndërmjet A1
dhe A2 ska pika të tjera nga S , atëherë p2  2 . Duke vazhduar në këtë
mënyrë vimë në përfundim se pi  i, 2  i  m . Kështu vërtetuam se S
mund të renditet linearisht në dy dhe vetëm dy mënyra. □

Përkufizim. Figura F quhet konvekse, në qoftë se për çdo dy pika


A, B  F  ( AB)  F .

Nga renditja e katër pikave në drejtëz rrjedh se

Çdo segment është figurë konvekse;


Prerja e çfarëdo familje të figurave konvekse është figurë konvekse.

Poligoni. Le të jetë A1 , A2 ,..., An1 një bashkësi e renditur pikash, ashtu që


çdo tri pika të njëpasnjëshme janë jokolineare .
Bashkësinë
A1 A2  A2 A3  An An1
do ta quajmë vijët të thyer apo vijë poligonale I dhe do ta shënojmë
A1 A2 ... An1 . Pikat A1 , A2 ,..., An1 quhen kulme, kurse segmentet
A1 A2 , A2 A3 ,..., An An1 quhen brinjë të vijës poligonale. Kulmet të një brinje
i quajmë kulme fqinjë, ndërsa brinjët incidente me të njëjtin kulm, brinjë
fqinje. Për vijën poligonale A1 , A2 ,..., An1 themi se i lidh apo bashkon pikat
A1 dhe An1 .
Vija poligonale quhet e hapur apo e mbyllur, varësisht nga fakti se kulmet
A1 , An1 janë pika të ndryshme apo përputhen.
Vija poligonale e mbyllur quhet poligon ose shumëkëndësh dhe shënohet
me A1 A2 ... An . Poligoni me 3,4,5,...,n brinjë quhet trekëndësh,
katërkëndësh, pesëkëndësh,..., n-këndësh. Poligoni quhet i rrafshtë, në
qoftë se të gjithë kulmet e tij i takojnë një rrafshi. Përndryshe poligoni
quhet hapësinor.
n-këndëshi A1 A2 ... An quhet i thjeshtë, në qoftë se çdo dy brinjët të tij skanë
pika të përbashkëta, përvëç që dy brinjë fqinje kanë kulm të përbashkët.
Përndryshe poligoni quhet jo i thjeshtë apo i përbërë.
Segmenti i përcaktuar nga dy kulme jofqinje të poligonit quhet diagonale
e tij.
40 I Hyrje

Përkufizim. Për dy pika A, B të figurës F themi se janë të lidhura, në


qoftë se ekziston vija poligonale (apo segmenti) që i bashkon pikat A, B
dhe i takon figurës F . Figura F quhet e lidhur, në qoftë se çdo dy pika të
saj janë të lidhura. Figura e lidhur quhet zonë.

Nga përkufizimi rrjedh se : Çdo figurë konvekse është e lidhur.

Përkufizim. Le të jenë F1 , F2 ,..., Fn figura që përmbahen në figurën F .


Për figurat F1 , F2 ,..., Fn themi se e zbërthejnë figurën F në figura (zona)
Z1 , Z 2 ,..., Z m , (m  1) , në qoftë se relacioni i lidhmërisë në figurën
F  ( F1  F2  ...  Fn ) është relacion ekuivalence, ndërsa Z1 , Z 2 ,..., Z m
janë klasët e ekuivalencës të atij relacioni.

Si rrjedhime të përkufizimit të më sipërm dhe pohimeve 3 dhe 5 kemi:

Pohimi 8. Çdo pikë C e segmentit të hapur AB e zbërthen AB në dy


segmente të hapur AC dhe CB.□

Pohimi 9. Në qoftë se B ( A1 , A2 ,..., An ) , atëherë pikat A2 , A3 ,..., An1 e


zbërthejnë segmentin ( A1 An ) në n  1 segmente ( A1 A2 ),..., ( An1 An ) .□

3. Gjysmëdrejtëza, gjysmërrafshi dhe gjysmëhapësira

Gjysmëdrejtëza

Në bashkësinë e pikave të një drejtëze do të përkufizojmë një relacion


ekuivalence dhe do të tregojmë se faktorbashkësia në lidhje me të ka dy
dhe vetëm dy klasë, secilën prej të cilave do ta quajmë gjysmëdrejtëz.

Le të jetë l cila do një drejtëz dhe A cila do një pikë e saj.

Përkufizim. Për pikat P, Q  l  { A} themi se ndodhen nga njëra anë e


pikës A, dhe shënojmë  A ( P, Q) , në qoftë se nuk është B ( P, A, Q) . Në të
kundërtën, në se B ( P, A, Q) , atëherë themi se pikat P, Q ndodhen në anë
të ndryshme të pikës A dhe shënojmë P, Q  A .
I Hyrje 41

Pohimi 1.  A është relacion ekuivalence në l  { A} dhe e zbërthen këtë


bashkësi në dy klasë të ekuivalencës.

Vërtetim. Për çdo Pl-{A}kemi  B ( P, A, P) , sepse po të ishte B ( P, A, P)


, nga aksioma II1, P, A, P do të ishin tri pika të ndryshme, gjë që nuk janë.
Pra, vlen  A (P,P) për çdo Pl-{A}.
Nëse  A ( P, Q) , atëherë  B ( P, A, Q)   B (Q, A, P)   A (Q, P) .
Le të jetë  A ( P, Q) dhe  A (Q, R) . Vërtetojmë  A ( P, R) . Supozojmë të
kundërtën, se P, R  A , gjegjësisht B ( P, A, R) .
Nga  A ( P, Q)   B ( P, A, Q) , atëherë ose B ( P, Q, A) ose B (Q, P, A) .
Tani, nga B ( P, Q, A) & B ( P, A, R)  B (Q, A, R) (pohimi 6), kurse nga B
(Q, P, A) dhe B ( P, A, R)  B (Q, A, R ) (pohoimi 5). Në të dy rastet kemi
Q, R  A , që është në kundërshtim me supozimin e bërë. Pra  A është
tranzitiv.
Vërtetojmë tani se (l  { A}) ka dy dhe vetëm dy klasë. Vërtet, meqë
A
çdo drejtëz ka së paku dy pika, Pl, P  A . Nga aksioma II4 ekziston
Ql, B ( P, A, Q) , rrjedhimisht P, Q  A , prandaj numri i klasëve është së
paku dy. Po të ishte numri i klasëve më i madh se 2, do të ekzistonte ndonjë
pikë R, e tillë që P, R  A dhe Q, R  A . Atëherë do të kishim B ( R, A, P)
dhe B ( R, A, Q) , prej ku, në bazë të pohimit 7, nuk do të ishte B ( P, A, Q) ,
që e kundërshton supozimin. Pra, numri i klasëve nuk është më i madh se
dy. □

Po ti shënojmë me l1 , l2 klasët e ekuivalencës për të cilat bën fjalë pohimi


i më sipërm, atëherë
l  {A}  l1  l2 , gjegjësisht l  {A}  l1  l2 .
Pasi që numri i klasëve është dy, secilën prej tyre do ta quajmë plotësuese
të klasës tjetër. Po ta shënojmë njërën prej tyre, psh. l1  a , atëherë
plotësuesen e saj l2 do ta shënojmë a ' , pra
l  { A}  a  a ' .
Përkufizim. Secilën prej klasëve të ekuivalencës të relacionit  A në
drejtëzën l do ta quajmë gjysmëdrejtëz të hapur të drejtëzës l. Pikën A do
ta quajmë pikë fillestare apo kufi të gjysmëdrejtëzës. Unionin e
gjysmëdrejtëzës së hapur dhe pikës fillestare të saj do ta quajmë
42 I Hyrje

gjysmëdrejtëz të mbyllur. Nëse nuk thuhet ndryshe, ne gjysmëdrejtëzën do


ta konsiderojmë të mbyllur.
Nga sa u tha më sipër, gjysmëdrejtëza përcaktohet nga pika fillestare dhe
cila do një pikë e saj. Për B  l  { A} po ta shënojmë me a  CB klasën e
ekuivalencës të pikës B, atëherë me
AB   { A}  a , AB   l  a
do ti shënojmë gjysmëdrejtëzat plotësuese të drejtëzës l me fillim në pikën
A.

Për çdo dy pika M , N  AB  dhe X  ( MN ) , nga pohimi 2.6 rrjedh B


( A, M , X , N ) ose B ( A, N , X , M ) , pra ( M , A, X ) që do të thotë se
X  AB . Kështu kemi vërtetuar se

Çdo gjysmëdrejtëz është figurë konvekse.

Nga pohimi1 dhe përkufizimi i lidhmërisë së dy pikave rrjedh se relacioni


”nga njëra anë e pikës A” është relacion i lidhmërisë në figurën l  { A} ,
prandaj pika A e zbërthen drejtëzën l në dy gjysmëdrejtëza të hapura që
janë plotësuese të njëra tjetrës.

Kështu vërtetuam pohimin:

Çdo pikë Al e zbërthen drejtëzën l në dy gjysmëdrejtëza të hapura,


plotësuese të njëra tjetrës.

Le të jenë A, B  l dy pika të fiksuara dhe X cila do një pikë e saj. Nga


pohimi i parë i Peanos rrjedh se X i takon vetëm njërës prej bashkësive
( AB), AB  , BA dhe me qenë se secila prej tyre është figurë e lidhur kemi

Çdo dy pika A,B të drejtëzës l e zbërthejnë drejtëzën në tri zona


( AB), AB , BA .

Ngjajshëm vërtetohet se

Çdo n pika A1 , A2 ,..., An të një drejtëzee zbërthejnë atë drejtëz në n+1


I Hyrje 43

zona, prej të cilave n-1 janë segmente të hapur dhe dy gjysmëdrejtëza të


hapura.

Gjysmërrafshi.

Le të jetë p drejtëz e një rrafshi .

Përkufizim. Për dy pika P, Q    p themi se ndodhen nga njëra anë e


drejtëzës p, dhe shënojmë  p ( P, Q ) , në qoftë se PQ  p   .
Në qoftë se p e pret segmentin PQ, atëherë themi se pikat P,Q ndodhen
në anë të ndryshme të drejtëzës p dhe shënojmë P, Q  p ose B ( P, p, Q) .

Pohimi 2. Relacioni  p është ekuivalencë dhe   p


 p =2.
Vërtetim. Vetia refleksive dhe simetrike janë rrjedhime direkte të
përkufizimit. Vërtetojmë se nga  p ( P, Q ) dhe  p (Q, R ) rrjedh  p ( P, R) .
a) Nëse P, Q, R janë pika kolineare dhe nëse drejtëza PQ nuk e pret
drejtëzën p, vërtetimi është i qartë. Nëse drejtëza PQ e pret p në ndonjë
pikë A, atëherë do të kemi  A ( P, Q) dhe  A (Q, R) , dhe meqë  A është
tranzitiv, kemi  A ( P, R) , prej ku  p ( P, R) .
b) Le të jenë P, Q, R pika jokolineare. Nëse supozojmë se P, R  p ,
atëherë, nga aksioma e Pasch-it, drejtëza p do ta priste edhe njërin prej
segmenteve PQ, QR , që është në kundërshtim me supozimin. Pra vlen
 p ( P, R ) .
Nga aksiomat e incidencës Xp dhe Ap, kurse nga II4, Y, B
( X , A, Y ) . Pasi që p  XY  { A} kemi që X , Y  p , rrjedhimisht numri i
klasëve është së paku dy. Nëse numri i klasëve është më i madh se dy, do
të ekzistojë edhe një pikë Z e tillë që X , Z  p dhe Y , Z  p . Atëherë
drejtëza p do ta presë secilin prej segmenteve XY , YZ , XZ , që për X , Y , Z
jokolineare, është në kundërshtim me pohimin1.3, kurse për X , Y , Z
kolineare e kundërshton pohimin 1.1. Pra ekzistojnë vetëm dy klasë të
ekuivalencës.□

Po ti shënojmë me  1 ,  2 klasët e ekuivalencës për të cilat bën fjalë pohimi


2, atëherë
44 I Hyrje

  p  1  2 , gjegjësisht   p 1  2 .
Pasi që numri i klasëve është dy, secilën prej tyre do ta quajmë plotësuese
të klasës tjetër. Le ta shënojmë njërën prej tyre, psh. 1   , atëherë
plotësuesen e saj  2 do ta shënojmë  ' , pra

  p   ' .

Përkufizim. Secilën prej klasëve të ekuivalencës të relacionit  p në


rrafshin  do ta quajmë gjysmërrafsh të hapur në rrafshin . Drejtëzën p
do ta quajmë teh apo kufi të gjysmërrafshit. Unionin e gjysmërrafshit të
hapur dhe tehut të tij do ta quajmë gjysmërrafsh të mbyllur.
Nëse nuk thuhet ndryshe, gjysmërrafshin do ta konsiderojmë të mbyllur.

Nga sa u tha më sipër, gjysmërrafshi përcaktohet nga tehu i tij dhe cila do
një pikë që nuk i takon tehut. Për A    p po të shënojmë me   CA
klasën e ekuivalencës të pikës A, atëherë me
pA  p   , pA    
do ti shënojmë gjysmërrafshet plotësuese të rrafshit  me teh të përbashkët
p. Me ABX  do ta shënojmë gjysmërrafshin që ka për teh drejtëzën AB
dhe e përmban pikën X, kurse me ABX  gjysmërrafshin plotësues të tij.

Për çdo dy pika M , N  pA dhe X  ( MN ) , lehtë konstatohet se


X  pA , gjegjësisht

Çdo gjysmërrafsh është figurë konvekse, pra edhe figurë e lidhur apo zonë.

Pohimi 3. Nëse pikat A, B ndodhen në anë të ndryshme të drejtëzës p,


atëherë çdo vijë poligonale që i bashkon pikat A,B e pret drejtëzën p.
Vërtetim. Sikur të ekzistonte ndonjë vijë poligonale , e tillë që p=,
atëherë të gjitha kulmet, pra, edhe të gjitha pikat e saj do të ishin nga ajo
anë e p nga është edhe pika A edhe pika B, që është në kundërshtim me
pohimin 2.

Nga pohimet 2 dhe 3 dhe përkufizimi i lidhmërisë së çiftit të pikave rrjedh


se relacioni ”nga njëra anë e drejtëzës p” është relacion i lidhmërisë në
figurën -p, prandaj drejtëza p e zbërthen rrafshin  në dy gjysmërrafshe
të hapur, plotësues të njëri tjetrit.
I Hyrje 45

Pra ka vend pohimi:

Çdo drejtëz p e rrafshit  e zbërthen rrafshin në dy gjysmërrafshe të hapur,


plotësues të njëri tjetrit.

Gjysmëhapësira.

Përkufizim. Le të jetë  cili do një rrafsh dhe P, Q  P -.


Për pikat P, Q themi se ndodhen nga njëra anë e rrafshit  dhe shënojmë
 ( P, Q) , në qoftë se   PQ = . Nëse  PQ   themi që P,Q
ndodhen në anë të ndryshme të rrafshit  dhe shënojmë P, Q   ose B
( P,  , Q ) .

Ngjashëm si në rrafsh vërtetohet pohimi

Pohimi 4.  është relacion ekuivalence që e zbërthen hapësirën P në dy


klasë të ekuivalencës.□

Po ti shënojmë me P 1, P 2 klasët e ekuivalencës për të cilat bën fjalë


pohimi 4, atëherë
P -  = P 1  P 2, gjegjësisht P =   P 1  P 2 .
Pasi që numri i klasëve është dy, secilën prej tyre do ta quajmë plotësuese
të klasës tjetër. Nëse njërën prej tyre e shënojmë, psh. P 1= , atëherë
plotësuesen e saj P 2 do ta shënojmë me ’ , pra
P =     ’ .
Përkufizim. Secilën prej klasëve të ekuivalencës të relacionit  në
hapësirën P do ta quajmë gjysmëhapësirë të hapur. Rrafshin  do ta
quajmë teh apo kufi të gjysmëhapësirës. Unionin e gjysmëhapësirës të
hapur dhe tehut të saj do ta quajmë gjysmëhapësirë të mbyllur.
Nëse nuk thuhet ndryshe, gjysmëhapësirën do ta konsiderojmë të mbyllur.
Nga sa u tha më sipër, gjysmëhapësira përcaktohet nga tehu i saj dhe cila
do një pikë që nuk i takon tehut. Për B P - , po të shënojmë me
= CB klasën e ekuivalencës të pikës B, atëherë me
 B     ,  B   P - 
46 I Hyrje

do ti shënojmë gjysmëhapësirat plotësuese me teh të përbashkët . Me


ABCX  do të shënojmë gjysmëhapësirën që ka për teh rrafshin ABC dhe
e përmban pikën X, kurse me ABCX  gjysmëhapësirën plotësuese të saj.

Gjithashtu, vërtetohet se

Çdo gjysmëhapësirë është figurë konvekse, pra edhe figurë e lidhur apo
zonë.

Pohimi 5. Nëse pikat A, B ndodhen në anë të ndryshme të rrafshit ,


atëherë çdo vijë poligonale që i bashkon pikat A,B e pret rrafshin . □

Nga pohimet 4 dhe 5 dhe perkufizimit të lidhmërisë së dy pikave kemi se


relacioni ”nga njëra anë e rrafshit ” është relacion i lidhmërisë në figurën
P - , prandaj rrafshi  e zbërthen hapësirën P në dy gjysmëhapësira të
hapura, plotësuese të njëratjetrës.

4. Vija këndore dhe këndi

Përkufizimi 1. Unionin e dy gjysmëdrejtëzave p, q me fillim të


përbashkët O do ta quajmë vijë këndore pq dhe do ta shënojmë pq .
Gjysmëdrejtëzat p, q do ti quajmë krahë, ndërsa pikën O kulm të vijës
këndore. Nëse A, B janë pika, me rradhë, nga krahët p, q , të ndryshme
nga kulmi O, vijën këndore pq do ta shënojme edhe AOB .
Nëse q  p ' , pp ' do ta quajmë vijë këndore të shtrirë.

Le të jetë pq   . Do të përkufizojmë tani relacionin  pq (nga njëra


anë e vijës këndorë pq) në bashkësinë  - pq .

Përkufizimi 2. Për pikat A, B    pq themi se janë nga njëra anë e


pq dhe i shënojmë  pq ( A, B ) , nëse A, B përputhen apo nëse ekziston
vija e thjeshtë poligonale (ose segment)  në  që i lidh pikat A, B dhe
ska pika të përbashkëta me pq .
Në qoftë se një vijë poligonale e tillë nuk ekziston, themi që pikat A,B
ndodhen në anë të ndryshme të vijës poligonale pq dhe shënojmë
A, B  pq .
I Hyrje 47

Pohimi 1. Relacioni  pq është ekvivalencë që bashkësinë - pq e ndan


në dy klasë të ekuivalencës.
Vërtetim. Vetia refleksive dhe simetrike janë rrjedhime direkte të
përkufizimit 2. Nëse  pq ( A, B ) dhe  pq ( B, C ) , ekzistojnë vijat
poligonale 1 dhe 2 në  që i bashkojnë pikat A,B dhe B,C, dhe
i  pq = , i=1,2.
Por atëherë edhe   pq = , ku  = 1  2 është vijë poligonale që
i bashkon pikat A,C. Pra,  pq është relacion ekuivalence.

Vërtetojmë se (  pq )
pq  2 .
Le të jenë a ', b ' , me rradhë, plotësimet e krahëve a,b, O kulmi i pq dhe
A,B pika të krahëve a,b.
Shënojmë me p  a  a ' {O}, q  b  b ' {O} .
Le të jenë   pA ,   qB gjysmërrafshet me tehe p,q që i përmbajnë
pikat A,B, kurse  ',  ' gjysmërrafshet plotësuese të tyre.
Për pikën P    ekziston pika Q, ashtu që B ( P, B, Q) . Meqë
ab  PQ  {B} rrjedh se pikat P, Q ndodhen në klasë të ndryshme të
relacionit ab .
Vërtetojmë tani se numri i klasëve nuk është më i madh se dy. Le të jetë
R    ab, R  P dhe R  Q .
Në qoftë se R    , atëherë PR ab   , prej ku rrjedh se pikat P
dhe R ndodhen në të njëjtën klasë.
Në qoftë se R    , atëherë R mund të jetë në njërën prej këtyre
bashkësive:
R  a '; R  b '; R    '; R  '  ; R  '  ' .
Në secilin prej këtyre rasteve, ekziston vija e thjeshtë poligonale  në
rrafshin , me skaje në pikat Q dhe R (psh.  = QM  MR , M a ' ), ashtu
që   ab = . Kështu, kur R    , pikat R dhe Q ndodhen në te
njëjtën klasë të ekuivalencës te relacionit ab . □
Përkufizimi 3. Secilën prej klasave të ekuivalencës të relacionit ab do
ta quajmë kënd i hapur ab dhe do ta shënojme me (ab) . Unionin e këndit
të hapur ab dhe vijës këndore ab do ta quajmë kënd i mbyllur ab dhe do
ta shënojmë me [ ab] .
Neve këtu me kuptimin kënd ab do të konsiderojmë këndin e mbyllur ab
dhe kur nuk ekziston rreziku për ngatërrimin e kuptimeve, do ta shënojmë
48 I Hyrje

me ab - njësoj si vijën këndore. Krahët dhe kulmin e vijës këndore do ti


quajmë, me rradhë, krahë të këndit dhe kulm i këndit . Këndet do ti
shënojmë edhe me germat ,  ,… të alfabetit grek.

Nga pohimi 1 dhe kuptimi i lidhshmërisë se dy pikave rrjedh se relacioni


nga njëra anë e vijës këndore pq është relacion i lidhshmërisë në figurën
- ab , prandaj vija këndore ab e ndan rrafshin  në dy kënde të hapur
(zona) që do ti quajmë plotësues të njëri-tjetrit. Nga vërtetimi i pohimit 1
rrjedh se    dhe  '  ' janë zonat e këndit ab. Zona    është
konvekse si prerje e bashkësive konvekse.

Pohimi 2.  '  ' është zonë jokonvekse.


Vërtetim. Le të jenë A,B pika përkatësisht të krahëve a,b të ab me kulm
në pikën O, kurse P,Q pika ashtu që B ( P, A, B, Q) . Pikat P, Q  '  ' ,
por PQ nuk i takon  '  ' sepse pika APQ, dhe A  '  ' . □

Nëse krahët e ab përputhen, këndi i hapur ab përbëhet prej të gjitha


pikave të rrafshit joincidente me a=b. Këndin e tillë do ta quajmë kend i
plotë. Nëse krahët e këndit janë gjysmëdrejtëza plotësuese të njëra-tjetrës,
këndin e tillë do ta quajmë kënd i shtrirë.
Dy kënde të një rrafshi që kanë një krah të përbashkët dhe nuk kanë pika
të tjera të përbashkëta quhen këndë fqinjë. Dy kënde fqinjë, krahët jo të
përbashkët të të cilëve janë gjysmëdrejtëza plotësuese quhen kënde të
përbrinjëshëm apo të bashkëmbështetur. Dy kënde konvekse, krahët e të
cilëve janë plotësime të njëri-tjetrit, quhen kënde kryqëzuese.
Në vazhdim, nëse nuk theksohet ndryshe, me fjalën kënd do të kuptojmë
këndin konveks. Po të jetë  një kënd konveks, me  ' do ta shënojmë
këndin plotësues të tij.

Sipërfaqja diedrike dhe diedri.

Përkufizimi 4. Unioni e dy gjysmërrafsheve  ,  me teh të përbashkët o


quhet sipërfaqe diedrike. Gjysmërrafshet ,  quhen faqe, kurse tehu i
përbashkët i tyre quhet teh i sipërfaqes diedrike. Sipërfaqen diedrike me
faqe ,  do ta shënojmë  .
I Hyrje 49

Nëse A, B janë pika, me rradhë, në faqet ,, sipërfaqen diedrike do ta


shënojmë edhe AoB .

Njësoj si në rrafsh e përkufizojmë kuptimin nga njëra anë e sipërfaqes


diedrike .

Përkufizimi 5. Për pikat P,Q P -  themi se janë nga njëra anë e
 dhe shënojmë   ( P, Q ) , nëse ekziston vija e thjeshtë
poligonale (ose segmenti)  që i lidh pikat P, Q dhe ska pika të
përbashkëta me  .
Në qoftë se një vijë poligonale e tillë nuk ekziston, themi që pikat P,Q
ndodhen në anë të ndryshme të sipërfaqes diedrike  dhe shënojmë
P, Q   .
Pohimi 3. Relacioni   është ekvivalencë që e ndan bashkësinë P -
 në dy klasë të ekuivalencës.
Vërtetimi bëhet në mënyrë analoge si në pohimin1□

Përkufizimi 6. Secilën prej klasave të ekuivalencës të relacionit   do


ta quajmë diedër i hapur  dhe do ta shënojme me ( ) . Unionin e
diedrit të hapur  dhe sipërfaqes diedrike  do ta quajmë diedër i
mbyllur ab dhe do ta shënojmë me [ ] .
Diedrin do të konsiderojmë të mbyllur dhe gjithmonë kur nuk ekziston
rreziku për ngatërrimin e kuptimeve, do ta shënojmë  - njësoj si
sipërfaqen diedrike. Faqet dhe tehun të sipërfaqes diedrike do ti quajmë,
me rradhë, faqe dhe teh të diedrit.

Nga pohimi 3 dhe kuptimi i lidhshmërisë se dy pikave rrjedh se relacioni


nga njëra anë e sipërfaqes diedrike është relacion i lidhshmërisë në
figurën P -  , prandaj sipërfaqja diedrike e ndan hapësirën P në dy
diedra të hapur, plotësues të njëri-tjetrit.
Kuptimi i diedrit të plotë, diedrit të shtrirë, diedrave fqinjë, diedrave të
përbrinjshëm dhe diedrave kryqëzues jepet në mënyrë analoge me
kuptimet përkatse për këndet.
50 I Hyrje

Trekëndëshi

Le të jenë A, B, C tri pika jokolineare. Unioni i segmenteve AB, BC , CA


quhet trekëndësh dhe simbolikisht shënohet ABC .
Pikat A, B, C quhen kulme, segmentet AB, BC , CA quhen brinjë, kurse
A  BAC , B  ABC , C  ACB quhen kënde të trekëndëshit
ABC .

Le të jetë  rrafshi i pikave A, B, C dhe P, Q   ABC .

Përkufizimi 1. Për pikat P, Q themi se ndodhen nga njëra anë e ABC


dhe simbolikisht shënojmë  ABC ( P, Q) , atëherë dhe vetëm atëherë nëse
ekziston vija poligonale P me skaje P, Q , ashtu që

P  ABC   .

Teorema 1.  ABC ( P, Q) është relacion ekuivalence në bashkësinë


  ABC .
Vërtetimi bëhet njësoj si te relacioni nga njëra anë e këndit.

Teorema 2.  ABC ( P, Q) e zbërthen bashkësinë   ABC në dy dhe


vetëm dy klasë të ekuivalencës:
 A  B C dhe  *A   B*   C*
ku  A  BCA ,  B  ACB  ,  C  ABC  dhe  *A ,  B* ,  C* plotësimet e
tyre.

Vërtetimi bëhet njësoj sikur për këndin.

 A  B C quhet zonë e brendshme e trekëndëshit dhe shënohet


( ABC ) , kurse  *A   B*   C* quhet zonë e jashtme e trekëndëshit dhe
shënohet ( ABC )* .

Teorema 3. Zona e brendshme e trekëndëshit është konvekse, kurse zona


e jashtme jokonvekse.
I Hyrje 51

Vërtetim. Zona e brendshme është konvekse si prerje e zonave konvekse,


kurse zona e jashtme është jokonvekse si union i zonave jokonvekse.□

Teorema 4. Zona e brendshme e ABC përmbahet në zonën e brendshme


të çdo këndi të tij.
Vërtetim.  A  B C   A   B ,  B  C ,  A C .□

Teorema 5. Në qoftë se P i takon zonës së brendshme të ABC , atëherë


gjysmëdrejtëza AP  e pret brinjën BC.

Vërtetim. Po të jetë M pikë e tillë që (B-A-M), atëherë (MC) i takon zonës


së jashtme të  A . Me qenë se AP  i takon zonës së brendshme të  A
rrjedh që
AP   (MC )   …….(1)
Nga ana tjetër, (MC)   C , dhe AP    C* , prandaj
AP   ( MC )   ……(2)
Nga (1) & (2) kemi
d ( AP)  ( MC )   .
Tani për pikat B, C , M dhe drejtëzën AP plotësohen konditat e aksiomës
II4, prandaj D  AP  ,( B  D  C ) , gjegjësisht D  ( BC ) .
Me qenë se ( BC ) i takon zonës së brendshme të  A , rrjedh që D i takon
asaj gjysmëdrejtëze të drejtëzës AP që ndodhet në  A , pra D i takon AP 
.□

5. Sipërfaqja trekëndëshe dhe trekendëshi


52 I Hyrje

III AKSIOMAT E KONGRUENCËS

3.1. Aksiomat e kongruencës dhe rrjedhimiet e para.

Aksiomat e kongruencës, sipas Hilbertit janë pesë dhe në mënyrë implicite


e përkufizojnë relacionin themelor është congruent në bashkësinë e
segmenteve dhe këndeve. Faktin se segmenti  AB  (këndi ab ) është
kongruent me segmentin CD (këndin cd )e shënojmë:
 AB  CD ( ab  cd ).

III1. Për çdo segment  AB  dhe për çdo gjysmëdrejtëz a ' me fillim në
pikën A ' , B '  a ' , e tillë që  AB  CD . Çdo segment është kongruent
me vetvehten.

III2 . Dy segmente kongruentë me një të tretë janë kongruentë edhe ndërjet


veti.
( A ' B '   AB   A '' B ''   AB)  A ' B '   A '' B " .

Nga III1, III2 rrjedh kjo teoremë:


I Hyrje 53

T.1. Relacioni  është ekuivalencë në bashkësinë e segmenteve.

Vërtetim. (r) Vetia refleksive rrjedh nga III1.


(s) Le të jetë  AB   A ' B ' . Nga  A ' B '   A ' B ' &  AB   A ' B ' dhe
III2 rrjedh  A ' B '   AB .
(t) Le të jetë  AB   A ' B ' &  A ' B '   A '' B '' . Nga  AB   A ' B ' &
 A '' B ''   A ' B ' dhe III2 rrjedh  AB   A '' B '' . //
Ekuivalenca  e copton bashkësinë S të segmenteve në klasë të
ekuivalencës:
S   s  CAB : AB  S .

III3. Le të jetë ( A  B  C ) & ( A ' B ' C ') .


Në qoftë se  AB   A ' B ' ,  BC    B ' C ' , atëherë  AC    A ' C ' .
III4. Për çdo ab , për çdo gjysmëdrejtëz a '  O ' P  dhe për çdo
gjysmërrafsh O ' PQ  , ekziston gjysmëdrejtëza e vetme b '  O ' PQ me
fillim në O ' e tillë që ab  a ' b ' . Çdo kënd është kongruent me
vetvehten.

Shënim. Ndryshimi i III1 dhe III4 qëndron në faktin se në III4 kërkohet që


gjysmëdrejtëza b ' të jetë e vetme.

III5. Le të jenë ABC , A ' B ' C ' dy trekëndësha.


Në qoftë se  AB   A ' B ' ,  AC    A ' C ' , A  A ' , atëherë edhe
B  B ', C  C ' .

Tani dotë vërtetojmë këtë teoremë:

T.2. Pika B ' për të cilën bën fjalë III1 është e vetme.

Vërtetim. Supozojmë se në gjysmëdrejtëzën a ' , ekziston edhe pika B ''


 B ' , e tillë që  AB   A ' B '' . Nga  AB   A ' B ' dhe  AB   A ' B ''
  A ' B '   A ' B '' .
Le të jetë C '  d ( A ' B ').
54 I Hyrje

Nga  A ' B '   A ' B '' ,  A ' C '   A ' C ' , B ' A ' C '  B '' A ' C ' dhe
aksioma III5 kemi
A ' C ' B '  A ' C ' B ''.
Por me qenë se A ' C ' B '  A ' C ' B ' , atëherë në të njëjtin gjysmërrafsh
A ' C ' B ' ekzistojnë dy gjysmëdrejtëza të ndryshme C ' B ' , C ' B '' që i
plotësojnë relacionet e mësipërme, gjë që është në kundërshtim me III4.
Pra, supozimi se ekziston edhe ndonjë pikë tjetër është kontradiktor,
rrjedhimisht, B ' është e vetme. //

T.2. është plotësim i III1. Atë do t’a zbatojmë për vërtetimin e kësaj
teoreme:

Teorema 3. Le të jetë ( A  C  B)  ( A ' C ' B ') . Në qoftë se


([ AB]  [ A ' B ']  [ AC ]  [ A ' C '])  [ BC ]  [ B ' C '] .

Vërtetim. Nga III1 B ''  C ' B ' , ashtu që [CB]  [C ' B ''] . Pasi që
( A  C  B)  ( A ' C ' B '') dhe ([ AC ]  [ A ' C '] , [CB]  [C ' B '']) , nga
aksioma
III3 , [ AB]  [ A ' B ''] . Me qenë se B ', B '' janë nga njëra anë e A ' dhe e
plotësojnë konditën [ AB]  [ A ' B '], [ AB]  [ A ' B ''] , nga T.2. rrjedh
B '  B '' . Tani kemi
[ BC ]  [ B '' C ']  [ B ' C '] . //

Tani do të tregojmë se edhe në bashkësinë e këndeve relacioni  është


ekuivalencë. Për këtë qëllim do t’i vërtetojmë disa pohime ndihmëse:

Lema 1. Në kushtet e aksiomës III 5 edhe [ BC ]  [ B ' C '] .

Vërtetim. Supozojmë se ([ BC ]  [ B ' C ']) . Atëherë në gjysmëdrejtëzën


B ' C ' ekziston pika C ''  C ',[ BC ]  [ B ' C ''] . Nga e dhëna e L.1. dhe III 5
kemi ABC  A ' B ' C ' .
Tani nga ([ AB]  [ A ' B '] ,[ BC ]  [ B ' C '], ABC  A ' B ' C '' ) dhe
aksioma III 5 kemi BAC  B ' A ' C '' . Me që BAC  B ' A ' C ' , në
gjysmërrafshin A ' B ' C ' ekzistojnë dy gjysmëdrejtëza të ndryshme
A ' C ' , A ' C '' që me gjysmëdrejtëzën A ' B ' formojnë kënnde kongruentë
I Hyrje 55

me BAC , që është në kundërshtim me III 4 . Supozimi ([ BC ]  [ B ' C '])


është kontradiktor, rjedhimisht [ BC ]  [ B ' C '] .//

Lema 2. Le të jetë ab  a ' b ' dhe c gjysmëdrejtëz në ab me fillim


në kulmin e tij O. Ekziston gjysmëdrejtëza e vetme c ' në a ' b ' me fillim
në kulmin e tij O ' e tillë që ac  a ' c ', bc  b ' c ' .

Vërtetim. Le të jenë A  a, B  b . Ekzistojnë pikat A '  a ', B '  b ' te tilla



OA  O ' A ' , OB  O ' B ' .
Nga AOB  A ' O ' B ' dhe III5 kemi
OAB  O ' A ' B ', OBA  O ' B ' A ' .
Ekziston pika C ashtu që  AB  c  {C}. Nga III1 C '  A ' B ' , ashtu
që [ AC ]  [ A ' C '] . Nga [ AB]  [ A ' B '] dhe [ AC ]  [ A ' C '] dhe T.3. kemi
[ BC ]  [ B ' C '] .
Duke zbatuar III5 për OAC , O ' A ' C ' fitojmë ac  a ' c ' , kurse III5
për OBC , O ' B ' C ' na jep bc  b ' c ' . Uniciteti i c ' rrjedh nga
uniciteti i pikës C ' . //

Lema e ardhëshme ka të bëjë me pohimet III3 dhe T.3. për këndet:

Lema 3. Le të jenë ab, a ' b ' kënde , përkatësisht me kulme O,O’, kurse
c, c ' gjysmëdrejtëza me fillim në O, O ' që ndodhen në këndet ab, a ' b '
. Atëherë:
(a) (ac  a ' c '  cb  c ' b ')  ab  a ' b '
(b) (ab  a ' b '  ac  a ' c ')  bc  b ' c ' .
Vërtetim. Vertetimi i Lemes 3.

(a) E bëjmë konstruktimin e vërtetimit: Në gjysmërrafshin me teh të


përcaktuar nga gjysmëdrejtëza a ' e që e përmban gjysmëdrejtëzën b '
ekziston gjysmëdrejtëza b1 me fillim në O ' ashtu që ab  a ' b1 . Në
bazë të Lemës 2 në këndin a ' b1 ekziston gjysmëdrejtëza c1  O ' e tillë
që ac  a ' c1 , bc  b1c1 . Meqë ac  a ' c ' , nga aksioma III 4
rrjedh c1  c ' . Tani nga bc  b1c ', bc  b ' c ' kemi b1  b ' . Pra,
ab  a ' b1  a ' b ' .
56 I Hyrje

Ngjashëm vërtetohet edhe (b): b1  O ' cb  c ' b1 . Me qenë se


ac  a ' c ', bc  b1c ' , nga (a) kemi ab  a ' b1 . Me që
ab  a ' b ' , nga III 4 rrjedh b1  b ' . Pra, bc  b1c '  b ' c ' . □

Lema 4. Në qoftë se te ABC , [ AC ]  [ BC ]  CAB  CBA .


Vërtetim. Shikojmë ABC , BAC . Nga

[ AC  [ BC ] 

[ BC  [ AC ]  dhe III5  ACB  BCA . □
ACB  BCA

Lema 5. Le të jenë C1 , C2 pika në anë të ndryshme të d ( AB ) . Në qoftë


se
[ AC1 ]  [ AC2 ]&[ BC1 ]  [ BC2 ] , atëherë AC1B  AC2 B.


Vërtetim. (i) Në qoftë se gjysmëdrejtëza C1C2 është në këndin AC1B
vërtetimi rrjedh nga Lema 3(a). (ii) Nëse gjysmëdrejtëza C1 A është në
këndin C2C1B , ose gjysmëdrejtëza C1 B  në këndin C2C1 A vërtetimi
rrjedh nga Lema 3(b).

Lema 6. Le të jenë ABC , A ' B ' C ' . Në qoftë se


[ AB]  [ A ' B ']  A  A ' 
 
[ BC ]  [ B ' C ']  B  B '  .
[CA]  [C ' B ']  C  C '

Vërtetim. Mjafton të vërtetojmë kongruencën A  A ' dhe të thirremi


në III5.
Le të jetë C ''  A ' B ' C ' ashtu që
C '' A ' B '  CAB,[ A ' C '']  [ AC ].
Nga Lema 1 kemi [ BC ]  [ B ' C ''] . Nga vetia tranzitive e kongruencës të
segmenteve kemi [ A ' C '']  [ A ' C ']&[ BC '']  [ B ' C '] . Tani nga Lema 5
kemi
A ' C '' B '  A ' C ' B '.

Nga III5 për A ' B ' C '', A ' B ' C ' rrjedh C '' A ' B '  C ' A ' B '. ...(1)
I Hyrje 57

Le të jetë tani C1'  A ' B ' C ' C1' A ' B '  CAB, [ A ' C1' ]  [ AC ]. Atëherë
[ A ' C1' ]  [ A ' C '']&[ B ' C1' ]  [ BC ]  [ B ' C ''] .
Duke vepruar si më sipër do fitojmë C '' A ' B '  C1' A ' B '. ....(2)
Pasi që pikat C ', C1' ndodhen në të njëjtin gjysmërrafsh me teh d ( A ' B ')
dhe i plotësojnë relacionet (1) dhe (2), nga aksioma III4 rrjedh se
A ' C '  A ' C1' dhe nga se [ A ' C1' ]  [ A ' C ']  C '  C1' . Pikën C1' e
mmuarrëm ashtu që C1' A ' B '  CAB , prandaj CAB  C ' A ' B ' . □

Lema 7. (Pohimi III2 për këndet)


Dy kënde kongruentë me një kënd të tretë janë kongruentë edhe ndërmjet
veti.

Vërtetim. Le të jenë ab, a ' b ', a '' b '' kënde me kulme, përkatësisht
O, O ', O '' të tillë që
ab  a ' b ' 

ab  a '' b ''
Vërtetojmë se a ' b '  a '' b '' .
Le të jenë A  a, A '  a ', A ''  a '', të tillë që [OA]  [O ' A ']  [O '' A ''] ;
B  b, B '  b ', B ''  b '', të tillë që [OB]  [O ' B ']  [O '' B ''] .
Nga OAB, O ' A ' B 'dhe L1 rrjedh [AB]  [A'B'] , kurse nga
OAB, O '' A '' B ''dhe L1 rrjedh [AB]  [A''B''] . Rrjedhimisht
[ A ' B ']  [ A '' B ''] .
Tani për trekëndëshat O ' A ' B ' dhe O '' A '' B '' kemi:
[O ' A ']  [O '' A ''],[O ' B ']  [O '' B ''],[ A ' B ']  [ A '' B ''] ,
dhe nga Lema 6 kemi a ' b '  a '' b '' . □

Teorema 4. Kongruenca në bashkësinë e këndëve është relacion


ekuivalence.

Vërtetim. Vetia (r) rrjedh nga III4. (s) Le të jetë ab  a ' b ' . Nga
a ' b '  a ' b '
 dhe Lema 7 rrjedh a'b'  ab .
ab  a ' b ' 
(t) Le të jetë ab  a ' b ' & a ' b '  a '' b '' . Nga
58 I Hyrje

ab  a ' b ' 


 dhe Lema 7 rrjedh ab  a''b'' □
a '' b ''  a ' b '

Le të jetë K c = K   {k  Cab : ab  K} bashkësia e klasëve të


ekuivalencës të relacionit  në bashkësine e këndeve K .

3.2. RELACIONI “është më i vogël” NË BASHKËSINË E


SEGMENTEVE

Në bashkësinë S të segmenteve dhe bashkësinë S c  S  do t’a


përkufizojm relacionin "  " dhe do të vërtetojmë se ai është renditje
rigoroze. Gjithashtu në S dhe S c do të përkufizojmë veprimin e
mbledhjes dhe do te vërtetojmë se (S c , , ) është gjysmëgrup abelian i
renditur.

Përkifizimi 1. Për segmentin [ AB ] themi se është më i vogël nga


segmenti [CD ] dhe simbolikisht shënojmë [ AB]  [CD] , në qoftë se
P, (C  P  D), ashtu qw [CP]  [ AB].

Teorema 1. Në bashkësinë S të segmenteve relacioni “<” është renditje


rigoroze.

Vërtetim. Vërtetojmë se “<” është vetëm tranzitiv, nuk është as refleksiv,


as simetrik.
Vërtet, sikur të [ AB], [ AB]  [ AB] , do të
P, ( A  P  B), ashtu qw [AP]  [ AB] , gjë që është në kundërshtim me
plotësimin e III1.
Le të jetë [ AB]  [CD] . Atëherë P, (C  P  D), ashtu qw [CP]  [ AB].
Sikur
[CD]  [ AB], atwherw do tw ekzistone pika Q, (A-Q-B), [ AQ]  [CD]
…(1)
Le të jetë R, (C  D  R), [DR]  [QB]. .. (2)
Nga relacionet (1) , (2) dhe aksioma III3 kemi [ AB]  [CR] . Në
gjysmëdrejtëzën CD  ekzistojnë dy pika të ndryshme P, R ashtu që
[CP]  [ AB],[CR]  [ AB] , që nuk është e mundur.
I Hyrje 59

(t) Le të jetë [ AB]  [CD] & [CD]  [ EF ] .


Nga [ AB]  [CD] rrjedh se P, (C  P  D), ashtu qw [CP]  [ AB] , kurse
nga [CD]  [ EF ] kemi se Q, ( E  Q  F ), ashtu qw [EQ]  [CD] . Le të
jetë R  EF  ,[ ER]  [ AB] . Me qenë se [ ER]  [ AB]  [CD]  [ EQ] rrjedh
( E  R  Q) . Në fund, nga ( E  R  Q) & ( E  Q  F )  ( E  R  F ) .
Pra, R, ( E  R  F ),[ ER]  [ AB] jep [ AB]  [ EF ] . □

Përkufizimi 2. Për segmentin [ AB ] themi se është më i madh nga


segmenti [CD ] dhe shënojmë [ AB]  [CD] , në qoftë se [CD]  [ AB] .

Teorema 2. Relacioni “<” është trihotomik, me fjalë të tjera, për çdo dy


segment [ AB],[CD] vlen vetëm njëri nga relacionet:
[ AB]  [CD] , [ AB]  [CD], [ AB]  [CD] .....(*)
Vërtetim. P  CD  , [CP]  [ AB] . Me qenë se pikat D, P janë në të
njëjtën gjysmëdrejtëz, atëherë ose
(C  P  D) ose P  D ose (C - D - P) , prandaj ka vend vetëm njëri
nga relacionet (*) . □

Relacionin < në S c do ta japim me anë të përfaqsuesëve të klasëve.


Përkufizimi 3. Le të jenë s1 , s2  S c dhe si  C[ Ai Bi ] , i  1, 2 .
s1  s2  [ A1B1 ]  [ A2 B2 ] .
Vërtetojmë se përkufizimi nuk varet nga zgjedhja e përfaqsuesëve të
klasave, me fjalë të tjera, në qoftë se [ A ' B ']  s1,[ A '' B '']  s2 , atëherë edhe
[ A ' B ']  [ A '' B ''] .
Me qenë se [ A ' B ']  [ A1B1 ]  [ A1B1 ]  [ A2 B2 ] kemi [ A ' B ']  [ A2 B2 ] . Kurse
nga
[ A ' B '] [ A2 B2 ]  [ A2 B2 ]  [ A '' B ''] kemi [ A ' B ']  [ A '' B ''] .
Me qenë se relacioni < në S c është dhënë me relacionin < në S, atëherë
ai i gëzon të njëjtat veti si në S. Pra, ka vend kjo teoremë:
Teorema 3. Në S c relacioni < është renditje rigoroze dhe gëzon vetinë e
trihotomisë.
Përkufizimi 4. Le të jenë s, s1 , s2  S c dhe s  C[ AB] , si  C[ Ai Bi ] , i  1, 2 .
Për s themi se është i barabartë me shumën e s1 , s2 dhe shënojmë
s  s1  s2 ,në qoftë se P,( A  P  B), [ AP]  [ A1B1 ], [PB]  [ A2 B2 ] .
60 I Hyrje

Akoma themi se segmenti [AB] është i barabartë me shumën e segmenteve


[ A1B1 ], [ A2 B2 ] dhe shënojmë [ AB]  [ A1B1 ]  [ A2 B2 ] .
Shohim mbledhjen e elementeve në S c e kemi dhënë me mbledhjen në S
, gjegjësisht C[ A1B1 ]  C[ A2 B2 ]  C[ A1B1 ][ A2 B2 ] .
Të vërtetojmë se edhe mbledhja nuk varet nga zgjedhja e prezentuesit të
klasë.
Le të jetë [ A ' B ']  s1,[ A '' B '']  s2 dhe le të jetë [ A ' B ']  [ A '' B '']  [CD] .
Kjo do të thotë se Q, (C  Q  D), [CQ]  [ A ' B '], [QD]  [ A '' B ''] . Me
qenë se
[ A ' B ']  [ A1B1 ], [ A '' B '']  [ A2 B2 ] , nga vetia transitive e kongruencës së
segmenteve, kemi [CQ]  [ AP]  [QD]  [ PB] dhe nga
(C  Q  D), ( A  P  B) , nga III3 kemi [CD]  [ AB] . Prandaj
C[ A ' B ']  C[ A '' B '']  C[ A ' B '][ A '' B '']  C[CD ]  C[ AB ]  C[ A1B1 ][ A2 B2 ] 
= C[ A1B1 ]  C[ A2 B2 ] .
Teorema 4. Për çdo s1 , s2  S c ,  një dhe vetëm një s  S c , ashtu që
s1  s2  s .
Vërtetim. Le të jenë si  C[ Ai Bi ] , i  1, 2 dhe a një gjysmëdrejtëz mme fillim
në A. Le të jenë P, B  a,[ AP]  [ A1B1 ],[ PB]  [ A2 B2 ] . Po të shënnojmë me
s  C[ AB ] , atëherë nga përkufizimi 4 kemi s1  s2  s . Klasa s është e
vetme, se po të ekzistoje edhe ndonjë klasë tjetër s '  C[ A ' B '] , s '  s1  s2 ,
atëherë do të P ',( A ' P ' B '), [ A ' P ']  [ A1B1 ], [ P ' B ']  [ A2 B2 ] . Nga
vetia transitive e kongruencës së segmenteve dhe aksioma III3 do të kishim
[ AB]  [ A ' B '] dhe rrjedhimisht s '  C[ A ' B ']  C[ AB ]  s . □
Nga teorema e mësipërme rrjedh se ( S c , ) është grupoid i renditur.
Teorema 5. Le të jenë s1 , s2  S c .
s1  s2   ! s  S c , s2  s1  s .
Elementi s quhet ndryshim i elementeve s2 , s1 dhe shënohet s  s2  s1 .
Vërtetim. Le të jenë si  C[ Ai Bi ] , i  1, 2 . Nga s1  s2  [ A1B1 ]  [ A2 B2 ]
rrjedh se P,( A2  P  B2 ),[ A2 P]  [ A1B1 ] . Po të shënojmë me s  C[ PB2 ]
do të kemi s2  s1  s . □
Lehtë vërtetohet se në ( S c , ) ka vend kjo teoremë:
Teorema 6. Për çdo s1 , s2 , s3 , s4  S c vlejnë këto veti:
I Hyrje 61

(i) s1  s2  s2  s1 (vetia komutative);


(ii) ( s1  s2 )  s3  s1  (s2  s3 ) (vetia asociative);
(iii) s1  s2  s1  s3  s2  s3 (pajtueshmëria e < nda mbledhjes;
(iv) (s1  s3  s2  s4 )  s1  s2  s3  s4 . □
Shënim. Nga teorema 6 rrjedh se (S c , , ) është gjysmëgrup abelian i
renditur.

3.3. Relacioni më i vogël për kënde


Ngjashëm si për segmentet, edhe për këndet përkufizohet relacioni është
më i vogël që i gëzon të njëjtat veti sikur edhe për segmentet.

Përkufizimi 1. Për ab themi se është më i vogël se cd dhe


simbolikisht e shënojmë ab  cd , në qoftë se në (cd ) ekziston
gjysmëdrejtëza p me fillim në kulmin e tij, ashtu që cp  ab . Në qoftë
se ab  cd , atëherë themi se cd është më i madh se ab dhe
shënojmë cd  ab .

Teorema 1. Në bashkësinë K të të gjithë këndeve < është relacion i


renditjes rigoroze dhe e gëzon vetinë e trihotomisë.□

Përkufizimi 2. Le të jenë k1 , k2  K c dhe ki  Caibi , i  1, 2 .


Për k1 themi se është më i vogël se k2 dhe shënojmë k1  k2 , atëherë dhe
vetëm atëherë nëse a1b1  a2b2 .

Edhe këtu tregohet se përkufizimi 2 është korrekt (nuk varet nga zgjedhja
e prezentuesit të klasës). Gjithashtu, relacioni < në K c është renditje
rigoroze dhe trihotomik.

Përkufizimi 3. Le të jenë k  Cab , ki  Caibi , (i  1, 2) tri elemente nga


Kc.
62 I Hyrje

Për k themi se është i barabartë me shumën e elementeve k1 , k2 dhe


shënojmë k  k1  k2 , në qoftë se në ab ekziston gjysmëdrejtëza p me
fillim në kulmin e tij, ashtu që ap  a1b1 , pb  a2b2 .
Akoma themi se ab është i barabartë me shumën e këndeve a1b1 , a2b2
dhe shënojmë ab  a1b1  a2b2 .
Si për segmentet, vërtetohet se përkufizimi 3 nuk varet nga zgjedhja e
përfaqësuesit të klasës.
Tani do të vërtetojmë se veprimi + në K dhe K c është pjesërisht i
përkufizuar. Për këtë qëllim, përkufizojmë kkuptimin e këndeve të
bashkëmbështetur(të përbrinjëshëm).

Përkufizimi 4. a ' b quhet kënd i përbrinjëshëm me ab , ku a ' është


plotësi i gjysmëdrejtëzës a.

Teorema 2. Këndet e përbrinjëshëm e këndeve kongruentë janë


kongruentë.

Vërtetim. Le të jenë ab  a1b1 , kurse me rradhe, këndet e


përbrinjëshëm të tyre. Vërtetojmë se a ' b  a1b1 . '

Le të jenë O, O1 kulmet e ab, a1b1 dhe A  a, B  B, C  a ' . Ekzistojnë


pikat A1 , B1 , C1 , përkatësisht, në a1 , b1 , a1' ,ashtu që
[OA]  [O1 A1 ],[OB]  [O1B1 ],[OC]  [O1C1 ]...(*) .
Nga aksioma III5 dhe Lema 1 për ABO dhe A1B1O1 kemi
OAB  O1 A1B1 , [ AB]  [ A1B1 ] ,
kurse nga ( A  O  C)&( A1  O1  C1 ) dhe relacionet (*) rrjedh
[ AC]  [ AC1 1] .
Tani III5 dhe Lema 1 e zbatuar për ABC, A1B1C1 na jep
OCB  O1C1B1 , [BC]  [B1C1 ] .
Në fund nga OBC, O1B1C1 dhe III5 gjejmë BOC  B1O1C1 ,
gjegjësisht, a ' b  a1' b1 . □

Për çdo element k  Cab  K c , nga teorema që sapo e vërtetuam, ekziston


vetëm një element k '  Ca 'b  K c .

Tani mund ta vërtetojmë këtë teoremë:


I Hyrje 63

Teorema 4. Le të jenë k1 , k2  K c . Shuma k1  k2 ekziston, atëherë dhe


vetëm atëherë, nëse k2  k1' .
Vërtetim. Le të jenë ki  Caibi , (i  1, 2) .
: Nëse  k1  k2  k  Cab , atëherë nga përkufizimi 3, ekziston
gjysmëdrejtëza p në ab , ashtu që ap të përfaqësojë k1 , kurse pb k2
-në. Pasi që p është në ab , atëherë b ndodhet në a ' p , rrjedhimisht,
k2  k1' .
: Nëse k2  k1' , ekziston gjysmëdrejtëza p në a1' b1 ashtu që
b1 p  a2b2 . k1  k2 = Ca1 p  K c .□

Nga Teorema 4 shohim që veprimi + në K c është pjesërisht i përkufizuar,


prandaj
Kc nuk është grupoid në lidhje me veprimin + .

Në K c mund të vërtetohet kjo teoremë:

Teorema 5. (i) Për çdo k1 , k2  K c , nëse ekziston k1  k2 , atëherë


ekziston edhe k2  k1 dhe
k1  k2 = k2  k1

(ii) Për çdo k1 , k2 , k3  K c nëse ekziston (k1  k2 )  k3 , atëherë ekziston


edhe k1  (k2  k3 ) dhe
(k1  k2 )  k3 = k1  (k2  k3 ) .

3.4. Teorema karakteristike të Gjeometrisë


absolute

Përkufizimi 1. a ' b ' quhet kënd kryqëzues (i kundërt në kulm) me ab


.

Teorema 1. Këndet kryqëzues janë kënde kongruentë.

Vërtetim. a ' b '  ab si kënde të përbrinjëshëm me ba ' .


64 I Hyrje

Përkufizimi 2. Këndi i përbrinjëshëm me një kënd të trekëndëshit quhet


kënd i jashtëm i atij trekëndëshi.
Këndet e jashtme në kulmet A, B, C do t’i shënojmë A ', B ', C ' .

Teorema 2. Këndi i jashtëm i trekëndëshit është më i madh se çdo kënd i


brendshëm, jo i përbrinjëshëm me të.

Vërtetim. Vërtetojmë, psh. se B ' është më i madh se A, C .


Nga vetia e trihotomisë vlen vetëm njëri nga relacionet:

B '  C , B '  C , B '  C .....(1)

(a) Supozojmë se B '  C . Caktojmë pikën D  BA , ashtu që


BD  AC . Nga ACB, DBC dhe III5 kemi ABC  DCB . Nga ana
tjetër, ABC  BCE , si kënde të përbrinjëshëm të këndeve kongruentë
B '  C . Kështu, në të njëjtin gjysmërrafsh BCD  kemi dy
gjysmëdrejtëza CD  , CE  që me gjysmëdrejtëzën CB  formojnë kënde
kongruentë me ABC , që është në kundërshtim me III4.
(b) Po të jetë B '  C , atëherë do të ekzistojë gjysmëdrejtëza CA ' , e
tillë që këndi A ' CB  B ' . Në këtë rast do të ekzistonte A ' CB , këndi
i jashtëm B ' i të cilit është i kongruent me këndin e brendshëm A ' CB
, që nuk mundet nga rasti (a).
Me qenë se dy relacionet e fundit në (1) janë kontradiktorre rrjedh se
B '  C .□

Nga Teorema 2 rrjedh: Shuma e dy këndeve të trekëndëshit është më e


vogël se këndi i shtrirë. (A  B  B ' B  2d )

Do të formulojmë tani një teoremë të rëndësishme të Gjeometrisë absolute


që tregon ekzistencën e drejtëzave joprerëse .

Drejtëzat a , b të prera nga një drejtëz e tretë c (që quhet edhe transverzalë)
formojnë tetë kënde α, β, γ, δ, α’, β’, γ’, δ’ të cilët dy nga dy janë kënde
përgjegjëse, shndërruese (alternative) dhe të kundërta.
I Hyrje 65

Teorema 3. Në qoftë se dy drejtëza të prea nga një e tretë formojnë kënde


përgjegjëse (shndëruese) kongruente, apo shuma e këndeve të kundërta
është kënd i shtrirë, atëherë ato dy drejtëza nuk priten.
Vërtetim. Le të jetë a  c  { A} , b  c  {B} dhe    (fig….).
a  b   , sepse sikur a  b  {C} , këndi i jashtëm i ABC do të ishte
congruent me këndin e brendshëm jo të përbrinjshëm me të, që nuk
mundet.□

3.4. Teorema karakteristike të Gjeometrisë


absolute

Përkufizimi 1. a ' b ' quhet kënd kryqëzues (i kundërt në kulm) me ab


.

Teorema 1. Këndet kryqëzues janë kënde kongruentë.

Vërtetim. a ' b '  ab si kënde të përbrinjëshëm me ba ' .

Përkufizimi 2. Këndi i përbrinjëshëm me një kënd të trekëndëshit quhet


kënd i jashtëm i atij trekëndëshi.
Këndet e jashtme në kulmet A, B, C do t’i shënojmë A ', B ', C ' .

Teorema 2. Këndi i jashtëm i trekëndëshit është më i madh se çdo kënd i


brendshëm, jo i përbrinjëshëm me të.

Vërtetim. Vërtetojmë, psh. se B ' është më i madh se A, C .


Nga vetia e trihotomisë vlen vetëm njëri nga relacionet:

B '  C , B '  C , B '  C .....(1)

(a) Supozojmë se B '  C . Caktojmë pikën D  BA , ashtu që


BD  AC . Nga ACB, DBC dhe III5 kemi ABC  DCB . Nga ana
tjetër, ABC  BCE , si kënde të përbrinjëshëm të këndeve kongruentë
B '  C . Kështu, në të njëjtin gjysmërrafsh BCD  kemi dy
gjysmëdrejtëza CD  , CE  që me gjysmëdrejtëzën CB  formojnë kënde
kongruentë me ABC , që është në kundërshtim me III4.
66 I Hyrje

(b) Po të jetë B '  C , atëherë do të ekzistojë gjysmëdrejtëza CA ' , e


tillë që këndi A ' CB  B ' . Në këtë rast do të ekzistonte A ' CB , këndi
i jashtëm B ' i të cilit është i kongruent me këndin e brendshëm A ' CB
, që nuk mundet nga rasti (a).
Me qenë se dy relacionet e fundit në (1) janë kontradiktore, rrjedh se
B '  C . Ngjashwm vwrtetohet se B '  A □

Nga Teorema 2 rrjedh: Shuma e dy këndeve të trekëndëshit është më e


vogël se këndi i shtrirë. (A  B  B ' B  2d )

Do të formulojmë tani një teoremë të rëndësishme të Gjeometrisë absolute


që tregon ekzistencën e drejtëzave joprerëse .

Drejtëzat a , b të prera nga një drejtëz e tretë c (që quhet edhe transverzalë)
formojnë tetë kënde α, β, γ, δ, α’, β’, γ’, δ’ të cilët dy nga dy janë kënde
përgjegjëse, shndërruese (alternative) dhe të kundërta.
( ( ,  ') janë kënde përgjegjëse, ( ,  ') janë kënde alternative, ( ,  ')
janë kënde të kundërta.)

Teorema 3. Në qoftë se dy drejtëza të prea nga një e tretë formojnë kënde


përgjegjëse (shndëruese) kongruente, apo shuma e këndeve të kundërta
është kënd i shtrirë, atëherë ato dy drejtëza nuk priten.
Vërtetim. Le të jetë a  c  { A} , b  c  {B} dhe    (fig….).
a  b   , sepse sikur a  b  {C} , këndi i jashtëm i ABC do të ishte
congruent me këndin e brendshëm jo të përbrinjshëm me të, që nuk
mundet.□

Teorema 4. Në qoftë se A a , ekziston drejtëza


b  rr ( A, a), b  A, b  a   .

Vërtetim.Le të jetë M  a , cila do. Shënojmë a  {M }  a1  a1' , dhe


  (a1 , MA ) . Në gjysmërrafshin me teh d ( AM ) , që e përmban a1 ,
ekziston gjysmëdrejtëza b1 me fillim në A , ashtu që ( AM  , b1 )   .
Drejtëza b e përcaktuar nga gjysmëdrejtëza b1 nuk e pret drejtëzën a .□

Shënim. Në qoftë se në a marrim një pikë tjetër M '  M dhe veprojmë


si në vërtetimin e teoremës 4 do të gjejmë një drejtëz tjetër
b '  A, b ' a   . Në Gjeometrinë absolute nuk mund të vërtetojmë se
I Hyrje 67

b '  b apo b '  b . Pra, ekzistenca e drejtëzës nëpër një pikë jashtë
drejtëzës së dhënë, që nuk e pret drejtëzën e dhënë është rrjedhim i
Gjeometrisë absolute, kurse fakti se drejtëza e tillë është e vetme apo nuk
është e vetme nuk mund të vërtetohet në gjeometrinë absolute.

Për ekzistencën e pikës së mesme të segmentit e rëndësishme është kjo


teoremë:

Teorema 5. Në qoftë se te ABC  A ' B ' C '


A  A ' 

C  C '  , atëherë AC  A ' C ' .
[ AB]  [ A ' B ']

Vërtetim. Nga vetia e trihotomise vlen vetëm njëri nga relacionet:

[ AC ]  [ A ' C '],[ AC ]  [ A ' C '],[ AC ]  [ A ' C '] .

Nëse [ AC]  [ A ' C '], C1,( A ' C1  C '),[ AC]  [ A ' C1 ] . Nga III5 kemi kemi
C  A ' C1B ' , gjegjësisht, C '  A ' C1B ' , që nuk mundet sepse
A ' C1B ' është kënd i jashtëm i B ' C ' C1 . Ngjashëm vërtetohet se edhe
supozimi [ AC ]  [ A ' C '] është kontradiktor.

Teorema 6. Për çdo segment [ AB], ! M , ( A  M  B),[ AM ]  [ MB] .

Vërtetim. Le të jenë C, C1 pika në anë të ndryshme të d ( AB ) , ashtu që


BAC  ABC1 , [ AC]  [BC1 ] ….(1)
Me qenë se C ÷ C1 , M , {M }  [CC1 ]  d ( AB) . Vërtetojmë se
( A  M  B) .
Sikur M  A do të kishim ( A  M  B) (C  A  C1 ) dhe BAC  ABC1
, që s’është e mundur sepse BAC është kënd i jashtëm i ABC1 . Po
kështu vërtetohet se nuk mund të jete M  B . Pra, A, M , B janë tri pika
të ndryshme për të cilat vlen vetëm njëri nga relacionet:
( A  M  B), ( A  B  M ), ( M  A  B) . Sikur të ishte ( A  B  M ) do të
kishim ABC1  BMC1  BAC , që është në kundërshtim me (1). Po
kështu vërtetohet se ( M  A  B) është kontradiktor. Pra mbetet
( A  M  B) .
68 I Hyrje

Për AMC, BMC1 plotësohen konditat e teoremës 4, prandaj kemi


[ AM ]  [ BM ] .
Në fund vërtetojmë se pika M është e vetme. Supozojmë se ekziston edhe
një pikë M1 , ( A  M1  B),[ AM1 ]  [M1B] . Me qenë se M , M1 janë nga
njëra anë e A, mund të jetë ( A  M1  M )apo( A  M  M1 ) . Nëse
( A  M1  M ) atëherë (M1  M  B) , prandaj
[ AM ]  [ AM1 ]  [M1B]  [MB] , që është në kundërshtim me
[ AM ]  [ MB] . Po kështu tregohet se edhe rasti ( A  M  M1 ) është
kontradiktor. Pra M është e vetme. □

Përkufizimi 3. Pika M për të cilën bëhet fjalë në teoremën e më sipërme


quhet mes apo qendër e segmentit [ AB ] , kurse secili nga segmentet
[ AM ],[ BM ] quhet gjysma e tij.

Teorema 7. Për çdo trekëndësh vlejnë pohimet:


(i) Përballë brinjës më të madhë ndodhet këndi më i madh, dhe
anasjelltas.
(ii) Çdo brinjë e trekëndëshit është më e vogël nga shuma dhe më
e madhe nga ndryshimi i dy brinjëve të tjera.

Vërtetim. (i) Le të jetë te ABC , [ BC ]  [ AC ] . Vërtetojmë se


BAC  ABC . Nga [ BC]  [ AC]  B1,( B  B1  C),[B1C]  [ AC] .

Atëherë AB1C  B1 AC . Me qenë se ( B  B1  C) , gjysmëdrejtëza AB1
ndodhet në BAC , prandaj
BAC  B1 AC  AB1C  ABC .
Anasjelltas, le të jetë BAC  ABC . [ BC ]  [ AC ] , sepse sikur
[ BC ]  [ AC ] , atëherë BAC  ABC , që është në kundërshtim me
supozimin.
(ii) Le të jetë [ BC ]  [ AB],[ AC ] . Vërtetojmë se
[ BC ]  [ AB]  [ AC ] .
E caktojmë pikën A1 ,(C  A  A1 ),[ AA1 ]  [ AB] . Kemi BA1 A  A1BA .
Te A1BC :
BAC 1  BA1 A  A1BA  A1BC , rrjedhimisht,
[ BC]  [ AC
1 ]  [ A1 A]  [ AC ]  [ AB]  [ AC ] .
I Hyrje 69

Le të jetë [ AB]  [ BC ],[ AC ] . Vërtetojmë se [ AB]  [ BC ]  [ AC ] . Po t’a


caktojmë pikën B1 ,( B  B1  C),[ B1C]  [ AC] , atëherë AB1C  B1 AC .
Te ABB1 kemi:
AB1B  B1 AC  AB1C  BAB1 , prandaj
[ AB]  [ BB1 ]  [ BC]  [ B1C]  [ BC]  [ AC] . □

3.5. KËNDI I DREJTË. DREJTËZAT NORMALE

Përkufizimi 1. Këndi kongruent me këndin e vet të përbrinjëshëm quhet


kënd i drejte.
Këndin e drejtë do t’a shënojmë me germën d.
Le të jetë O prerja e drejtëzave p, q dhe
p  {O}  a  a ', q  {O}  b  b ' .
Në qoftë se njëri prej këndeve ab, a ' b, ab ', a ' b ' është i drejtë,
atëherë, nga përkufizimi1 edhe tre këndet të tjerë do të jenë të drejtë:
[ ab  d  ab  a ' b . Nga se a ' b  ab  a ' b  d . Me qenë se
a ' b  d  a ' b  a ' b ', etj. ]

Përkufizimi 2. Dy drejtëza që priten dhe formojnë një kënd të drejtë quhen


drejtëza ortogonale apo drejtëza normale.

Faktin se drejtëza p është ortogonale në drejtëzën q do ta shënojmë


pq .
Vërtetojmë tani këtë teoremë:

Teorema 1. Për çdo rrafsh  , çdo drejtëz p   dhe çdo pikë


A  p, !n   , e tillë që n  A dhe n  p .

Vërtetim. Le të jenë a, a ' gjysmëdrejtëzat që i formon pika A në a dhe 1


njëri prej gjysmërrafsheve me teh p në  . Le të jetë b  1 gjysmëdrejtëz
me fillim në A. Në qoftë se ab  a ' b , atëherë drejtëza n e përcaktuar
nga b është drejtëza e kërkuar.
70 I Hyrje

Në qoftë se (ab  a ' b) , !b1  1 , gjysmëdrejtëz me fillim në A, e


tillë që a ' b1  ab . Le të jenë B  b, B1  b1 ashtu që [ AB]  [ AB1 ] dhe
M mesi i [ AB ] . Trekëndëshat ABM , AB1M i kanë brinjët kongruente,
rrjedhimisht i kanë edhe këndet kongruente, prandaj MAB  MAB1 . Po
t’a shënojmë me s  AM  , atëherë nga as  a ' s rrjedh se drejtëza n e
përcaktuar nga s është gjysmëdrejtëza e kërkuar.
Vërtetojmë tani se n është e vetme. Po të ekzistojë edhe n1  A, n1  p ,
atëherë në 1 do të ekzistojë gjysmëdrejtëza me fillim në A dhe s1  s , e
tillë që as1  a ' s1 . Pasi që s, s1 janë gjysmëdrejtëza të 1 , me fillim
në A, s1 është në as , apo s1 në a ' s .
Nëse s1 është në as , atëherë s është në a ' s1 , prandaj:
as  as1  a ' s1  a ' s , që s’është e mundur, sepse as  a ' s . □

Teorema 2. Këndi kongruent me këndin e drejtë është i drejtë.

Vërtetim. Le të jetë a1b1  ab dhe ab  a ' b . Atëherë a1b1  a ' b
dhe a ' b  a1' b1' (si këndë të përbrinjshëm të këndeve kongruentë),
prandaj a1b1  a1' b1' , rrjedhimisht a1b1  d . □

Teorema 3. Këndet e drejtë janë kongruent.

Vërtetim. Le të jenë ab, a1b1 kënde të drejtë dhe O, O1 kulmet e tyre. Le


të jetë p drejtëza e përcaktuar nga a1 , kurse 1 gjysmërrafshi me teh p që
e përmban gjysmëdrejtëzën b1 . !b1'  1 me fillim në pikën O1 , e tillë që
a1b1'  ab . Nga Teorema 2, a1b1'  d , kurse nga Teorema 1, b1'  b1 .
Pra, ab  a1b1'  a1b1 . □

Teorema 4. Për çdo drejtëz p dhe çdo pikë A  p, !n  rr ( Ap) , e tilë
që n  A, n  p .

Vërtetim. Le të jetë P  p cila do dhe p  {P}  a  a ' . Shënojmë me


  pA , b  PA . Caktojmë b1   'dhe A1  b1 ashtu që
ab1  ab  [ PA1 ]  [ PA] .
Vërtetojmë se d ( AA1 ) është drejtëza e kërkuar.
I Hyrje 71

a) Nëse A, P, A1 janë kolineare, nga ab1  ab dhe ab1 , ab të


përbrinjshëm, rrjedh se ab  d , prandaj n  d ( A, A1 )  p .
b) Nëse A, P, A1 janë jokolineare, nga A ÷ A1 të p , rrjedh se p e pret
d ( AA1 ) në ndonjë pikë, ta zëmë B. Nga APB  A1PB dhe aksioma III5
kemi ABP  A1BP dhe pasi që këta janë kënde edhe të përbrinjshëm,
janë të drejtë, prandaj n  d ( AA1 )  p .
Drejtëza n është e vetme, se po të ekzistonte edhe ndonjë drejtëz tjetër n1
, do të ekzistonte ABB1 te i cili një kënd i jashtëm është congruent me
këndin e brendshëm, jo të përbrinjshëm me të. □

Si rrjedhim të Teoremës 4 kemi

Teorema 5. Dy drejtëza normale në një të tretë janë drejtëza joprerëse.

Vërtetim. Le të jenë b, b '  a . Drejtëzat b, b ' nuk priten, sepse sikur


b  b '  {M } , nëpër pikën M do të kalonin dy normale në drejtëzën e
dhënë a. □

3.6. DREJTËZA NORMALE NË RRAFSH

Përkufizimi 1. Për drejtëzën n themi se është ortogonale (normale) në


rrafshin α , në qoftë se n    {N } dhe nëse n  a, për çdo a   , a  N
. Gjithashtu themi se rrafshi  është normal në drejtëzën n dhe shënojmë
n  .

Teorema 1. (Teorema Caushy) Në qoftë se drejtëza që e pret rrafshin është


normale në dy drejtëza të atij rrafshi, incidente me pikën e depërtimit,
atëherë ajo drejtëz është normale edhe ndaj rrafshit.

Vërtetim. Le të jetë p    {P} ; a, b   , a, b  P, dhe p  a, p  b .


Duhet të vërtetojmë se p është normale në çdo drejtëz c të  , c  P .
Le të jenë M , N  p , ( M  P  N ),[ MP]  [ NP] , kurse A  a, B  b , të
ndryshme nga P .
Nga AMP, ANP  L.3.1.1
[ AM ]  [ AN ] ,
72 I Hyrje

kurse nga BMP, BNP  L.3.1.1


[ BM ]  [ BN ] .
Tani nga ABM , ABN BAM  BAN ] .
L.3.1.6.

Pa e cënuar përgjithësimin e vërtetimit, mund të supozojmë se c ndodhet


në APB . Shënojmë me {C}  c  [ AB] .
Nga ACM , ACN  L.3.1.1.
[CM ]  [CN ] ,
Kurse nga CMP, CNP CPM  CPN dhe me
L.3.1.6

qenë se këta janë edhe kënde të përbrinjshëm, rrjedh se janë kënde të


drejtë, pra p  c . □

Teorema 2. (Teorema mbi tri normalet) Në qoftë se n   , n    {N }


dhe P është projeksioni normal i pikës N në një drejtëz a   , atëherë
d ( SP)  a , për S  p .

Vërtetim. Le të jetë A  a,[ AP]  [ SN ] . Nga SNP, APN dhe L3.1.1


kemi [ SP]  [ AN ] , kurse nga SNA, APS dhe L3.1.6 kemi
APS  SNA . Me qenë se n    SNA  d , prandaj APS  d . □

Teorema 3. Për çdo pikë A dhe çdo drejtëz a , !  A,   a .

Vërtetim. Do të dallojmë dy raste: (i) A  a ; (ii) A  a .


(i) Le të jenë  ,  '  a,    ' . !n   , n  A, n  a . Gjithashtu,
!n '   ', n '  A, n '  a . Shënojmë me   rr (n, n ') . Nga Teorema
Caushy, a   . Rrafshi  është i vetëm, se po të ekzistonte edhe
 '  A,  '  a ,  '   , atëherë rrafshi  ' do t’i priste rrafshet  ,  ' në
dy drejtëza, prej të cilave së paku njëra do të ishte e ndryshme nga drejtëzat
n, n ' , kështu që në njërin prej rrafsheve  ,  ' , nëpër pikën A do të Kalonin
dy drejtëza normale në drejtëzën a.
(ii) Nëse A  a , shënojmë me   rr ( Aa) , kurseme  ' cilin do një rrafsh
tjetër që e përmban drejtëzën a. Vërtetimi bëhet në mënyrë të ngjashme si
në (i). □

Si rrjedhim të teoremës 2 është kjo teoremë:

Teorema 3. Të gjitha normalet e një drejtëze në një pikë të saj i takojnë


një rrafshi që është normal në atë drejtëz.
Vërtetim. Le të jetë a një drejtëz dhe A një pikë e saj. Le të jenë
n1 , n2 , n3  A, ni  a, i  1, 2,3. Shënojmë me   rr (n1n2 ),  '  rr (n1n3 )
I Hyrje 73

.Nga teorema 1, a   ,  ' , ndërsa nga teorema 2,    ' , pra,


n1 , n2 , n3   . □

Teorema 4. Për çdo rrafsh  dhe çdo pikë A , !n  A, n   .

Vërtetim. Sërish do t’i dallojmë rastet: (i) A  ; (ii) A  .


(i) Nëse A  , le të jetë a   , a  A . Nga teorema 2, !   A,   a .
Shënojmë me p     . Në  , !n  A, n  p . Nga a    a  n . Me
qenë se n  a, p  n   (ekzistenca). Drejtëza n është e vetme, se po të
ekzistonte edhe një normale tjetër n '  A , në rrafshin   rr (nn ') do të
ekzistonin dy drejtëza n, n ' nëpër A, normale në drejtëzën s    ,
 .
(ii) Le të jetë A  dhe a   cila do një drejtëz. Në
rr ( Aa), !d ( AP)  a .
Në rrafshin  , !b  P, b  a . Dhe në fund, në rr ( Ab), !n  A, n  b .
Vërtetojmë se n   .
Le të jetë {B}  n  b , kurse A ', ( A  B  A '),[ A ' B]  [ AB] . Nga
ABP, A ' BP dhe L.3.1.1 kemi [ AP]  [ A ' P] . Le të jetë C  a e
ndryshme nga P. Nga teorema mbi tri normalet, d ( A ' P)  a . Tani nga
ACP, A ' CP dhe L.3.1.1 rrjedh [ AC ]  [ A ' C ] . Dhe më në fund, nga
ABC , A ' BC dhe L.3.1.6 rrjedh ABC  A ' BC dhe me që këta janë
edhe të përbrinjshëm, janë të drejtë. Drejtëza n është normale në dy
drejtëza b dhe d ( BC ) të rrafshit  , prandaj n   . Drejtëza n është e
vetme, se po të ekzistonte edhe n '  A, n '   , në rrafshi e tyre
  rr (nn ') , nëpër A do të kalonin dy drejtëza n, n ' normale në drejtëzën
s    ,  . □

3.7. RRAFSHET NORMALE

Përkufizimi 1. Për rrafshin α themi se është normal (orthogonal) në


rrafshin β, dhe shënojmë α  β, nëse  a   , a   .

Teorema 1. Në qoftë se        .
74 I Hyrje

Vërtetim. Nga α  β ,  a   , a   . Shënojmë { A}  a   , p     .


Atëherë A  p . Ekziston b   , b  A, b  p . Nga a    a  b. Por
nga b  a, p  b   .
Me qenë se  përmban një normale b të rrafshit α,rrjedh se β  α .

Teorema 2. Le të jenë    . Drejtëza në njërin prej tyre që është


normale në drejtëzën     p është normale edhe në rrafshin tjetër.
Vërtetim. Le të jetë a   , a  p . Vërtetojmë se a   .
Le të jenë M , M1  a,(M  A  M1 ), ku {A}  a  p}dhe[ AM ]  [ AM1 ] .
Me qenë se edhe    , b   , b   (fig...).
Nga ABM , ABM1 dhe Lema 1 kemi [ BM ]  [ BM1 ] . Nga e njëjta lemë
dhe trekëndëshat BB1M , BB1M1 kemi [ B1M ]  [ B1M1 ] . Trekëndëshat
ABM , ABM1 i kanë brinjët përkatëse kongruente, prandaj, nga lema 6,
ata i kanë edhe këndet përkatëse kongruente, rrjedhimisht,
MAB1  M1 AB1 , dhe me qenë se ata janë këndë të përbrinjëshëm, ata
janë edhe të drejtë, pra a  d ( AB1 ) . Nga a  p, d ( AB1 )  a   .□

Teorema 3. Le të jenë 1 , 2 rrafshe normale në rrafshin β. Në qoftë se


ekziston prerja e tyre p  1 2 , atëherë p   .
Vërtetim. Shënojmë me ai   i   , i  1, 2. Supozojmë se ( p   ) .
Atëherë ( p  ai ) (sikur p  ai , me që p  i dhe ai  i   , nga
teorema 2,  p  ai ). Le të jetë P  p dhe ni  P, ni  i , ni  ai .
Nga ni  i , ni  ai  i    ni   , i  1, 2 , që nuk është e mundur,
sepse nëpër pikën e dhënë kalon vetëm një drejtëz normale në rrafshin e
dhënë. Supozoimi ( p   ) është kontradiktor, rrjedhimisht, p   . □
Teorema 4. Dy normale të një rrafshi janë drejtëza koplanare.
Vërtetim. Le të jenë a, b   . Shënojmë
{ A}  a   ,{B}  b   , p  d ( AB) . Po të jetë   rr (ap) , atëherë
   . Le të jetë b1   , b1  B, b1  p . Nga T.2. b1   . Pra, nëpër pikën
B  kalojnë dy normale b, b1 në α, rrjedhimisht b  b1   . □

Rrjedhimi 1. Të gjitha normalet e një rrafshi, pikat depërtuese të të cilave


i takojnë një drejtëzëj janë drejtëza koplanare.□

Rrjedhimi 2. Në qoftë se (a   ),  !   a,    .


I Hyrje 75

Vërtetim. Le të jenë n, n '  , me rradhë, nëpër A, B , n, n '   . Nga T.4.


ato i takojnë një rrafshi, të themi β = rr (n, n ') , i cili e përmban a dhe është
normal në α .

Po të ekzistoje edhe ndonjë rrafsh 1  a, 1   , nga T.3. edhe


  1  a   ,  . □

Përkufizimi 2. Pika prerëse e normales nëpër A në rrafshin α quhet


projeksion normal i A në α., që do ta shënojmë me prn ( A) apo A ' .

Po të jetë F një figurë, atëherë F '  {M ' : M  M } quhet projeksioni


normal i figurës F në rrafshin α.

Përkufizimi 3. Largesë apo distancë të pikës nga rrafshi α quhet segmenti


[ AA '] , ku A ' është projeksioni normali A në α .

Përkufizimi 4. Këndi më i vogël se këndi i drejtë quhet kënd i ngushtë,


ndërsa këndi më i madh se këndi i drejtë quhet kënd i gjerë.
76 I Hyrje

IV SIMETRIA

Pasqyrimi bijektiv i E në vetvehte quhet transformim gjeometrik.


Bashkësinë e transformimeve bijektive të E do ta shënojmë, si zakonisht,
me SE. Për cilin do rrafsh , me S do të shënojmë bashkësinë e të gjitha
transformimeve bijektive të .
Le të jetë f  S E . Pika XE e tillë që Xf=X quhet pikë fikse (invariante,e
dyfishtë, e pa lëvizshme) për transformimin f. Pasqyrimi f  SE që i
fikson të gjtha pikat quhet pasqyrim identik (identitet, koincidencë) dhe
zakonisht shënohet me 1 apo 1E apo idE.

Për figurën F themi se është invariante ndaj f, në qoftë se f i fikson të


gjitha pikat e M. Dihet se për çdo bashkësi M, (SM,) është grup
jokomutativ me element njësh 1M.
Prej këtu vimë në përfundim se (SE, ) dhe (S,) janë grupe joabeliane
me elemente njësh, përkatësisht 1E, dhe 1 .

4.1. Simetria boshtore në rrafsh

Përkufizimi 1. Le të jetë s  cila do një drejtëz. Pasqyrimi

s:    që XX’ i përkufizuar me:


I Hyrje 77

(i) Për Xs, X’X ;


(ii) Për Xs, d(XX’)  s & XXo  X’Xo , ku

Xo = s  d(XX’)

quhet simetri boshtore në rrafshin . Drejtëza s quhet bosht i simetrisë.

Përkufizimi 2. Pasqyrimi f :  , f  1, për të cilin f2 = 1 quhet


pasqyrim involutiv apo involucion.

Pohimi 1. Çdo involucion f është bijeksion.

Vërtetim . Le të jetë Xf =Yf , X,Y. Atëherë

X=(X)1=(X)f2=(Xf)f=(Yf)f=(Y)f2=(Y)1=Y.
Nga ana tjetër, për Y, X=Yf, ashtu që

Xf=(Yf)f=Yf2=Y1=Y. □

Nga D.1. rrjedh se s  1. Por me që s: X’ X  s2 = 1.

Kështu vërtetuam

Pohimi 2. s është transformim involutiv, rrjedhimisht s është


bijeksion.

Pohimi 3. Në qoftë se s: A,B  A’, B’ , atëherë AB  A’B’ .

Vërtetim . Në qoftë se d(AB)  s , vërtetimi rrjedh nga nga T.3.1.3. Në


qoftë se ┐(d(AB)  s ), do të shënojmë

{Ao}= d(AA’)  s , {Bo}= d(BB’)  s .

Le të jenë A,B nga njëra anë e s. Ngjashëm vërtetohet edhe rasti tjetër.
Nga ∆AAoBo, ∆A’AoBo dhe lemat e mëparshme kemi: ABoA’Bo dhe
ABoBo A’BoAo. Tani kemi që edhe ABoB  A’BoB’ .
Përfundimisht, nga ∆ABBo , ∆A’B’Bo dhe L.3.1.1 rrjedh AB A’B’. □
78 I Hyrje

Nga pohimi P.3. nuk rrjedh se [AB][A’B’]. Për këtë qëllim vërtetojmë
se

Pohimi 4. s është transormim kolinerar.

Vërtetim . Le të jene A,B,C tri pika kolineare. Me qenë se patjeter njëra


prej tyre ndodhet ndërmjet dy të tjerave, le të jetë (A-B-C). Atëherë AC
= AB + BC…(). Po të jetë s : A, B, C A’, B’, C’ do të kemi
ABA’B’, BCB’C’, ACA.C’.
Nga () kemi A’C’ = A’B’ + B’C’ prej ku vimë në përfundim se A’, B’,
C’ janë pika kolineare dhe (A’-B’-C’).

Me qenë se s e ruan renditjen, gjysmëdrejtëzën e pasqyron në


gjysmedrejtëz dhe kendin në kënd.
Pika fikse të s janë të gjitha pikat e boshtit s, rrjedhimisht boshti s është
invariant ndaj s. Nga se  s : d ( XX ')  d ( X ' X )  d ( XX ') , drejtëzat
normale në boshtin s janë fikse ndaj s, por joinvariante. Prej këtu rrjedh
se gjysmërrafshet me tehe normale në boshtin s janë fiks ndaj s.

Pohimi 5. Në qoftë se s: ab  a’b’ , atëherë : ab  a’b’ .

Vërtetim . Le të jenë O, O’ kulmet e këndeve ab , a’b’ . Atëherë


s: OO’. Le të jenë Aa, Bb, kurse A’=As , B’=Bs. Nga P.2.
kemi OAO’A’ , OBO’B’, ABA’B’. Tani nga L.1.3.6 kemi
AOBA’O’B’, gjegjësisht
aba’b’ . □

Figura …

Rrjedhim . Simetria boshtore këndin e drejtë e pasqyron në kënd të


drejtë.
I Hyrje 79

Përkufizimi 3. Drejtëza s quhet bosht simetrie apo simetrale e segmentit


AB, në qoftë se s është incidente me mesin e AB dhe s  d(AB).

Pohimi 6. Simetralja e segmentit ekziston dhe njëvleftësish është e


përcaktuar.

Vërtetimi rrjedh nga ekzistenca dhe uniciteti i mesit të segmentit dhe


normales nëpër një pikë në drejtëzën e dhënë.

Pohimi 7. s është simetrale e AB  s : AB .

Vërtetim . Në qoftë se s = sAB dhe M mesi i AB, atëherë AMBM dhe


s  d(AB), që nga përkufizimi i simetrisë boshtore rrjedh s : AB.
Anasjelltas, nga As=B  d(AB)s dhe AMBM, ku {M}= s  d(AB).□

Pohimi 8. Le të jenë A, B, P tri pika. P  s=sAB  APBP.

Vërtetim . Në qoftë se Ps  s: A,P  B, P  APBP. Anasjelltas,


po të jetë APBP dhe S mesi i AB, nga ∆ASP, ∆BSP dhe L.3.3.6. kemi
ASPBSP dhe pasi që këta janë edhe të përbrinjëshëm, rrjedh se janë
të drejtë. Pra, d(SP) është simetrale e AB. □

Përkufizimi 4. Le të jetë O kulmi i ab. Drejtëza s  O e tillë qe as=b


quhet simetrale e ab.

Në qoftë se me s1 e shënojmë gjysmëdrejtëzë e s me fillim në O që


ndodhet ne ab, atëherë nga s : a, s1  b, s1  as1  bs1.
Gjysmëdrejtëza s1 quhet përgjysmore apo bisektrisë e ab.

Pohimi 9. Për  ab ekziston dhe është e vetme simetralja e tij.


80 I Hyrje

Vërtetim . Le të jenë Aa, Bb të tilla që OAOB. Atëherë simetralja


s e AB kalon nëpër O. dhe s është simetrale e ab. Anasjelltas, po të jetë
s simetale e ab, atëherë nga ∆AOS, ∆BOS kemi ASOi dhe ASBS
 s është simetrale e AB. Pasi që simetralja e AB është e vetme 
simetralja e ab është e vetme.□

Përkufizimi 5. ABCD quhet Katërkëndësh i Saccherit, në qoftë se


ADBC dhe BADABC. AB quhet baza e sipërme, CD baza e
poshtme, kurse AD dhe BC brinjë anësore të Katërkëndëshit të
Saccherit. (Figura)

Pohimi 10. Simetralja e bazës së poshtme të katërkëndëshit të Saccherit


është simetrale edhe e bazës së sipërme. Këndet në bazën e sipërme të
katërkëndëshit të Saccherit janë kongruentë.

Vërtetim . Le të jetë s simetrale e bazës së poshtme AB të K.S. ABCD.


Pasi që s e ruan kongruencën e segmenteve dhe këndeve rrjedh
s: AB, AD+BC+. Nga se s: Di , rrjedh se s është simetrale e i.
Tani nga s : ADCBCD  ADCBCD.□ (figura)

Përkufizimi 6. Në qoftë se s : FF’, atëherë për figurat F, F’ themi


se janë simetrike ndaj drejtëzes s. Në rast të veçantë, në qoftë se s: FF,
atëherë themi se figura F është simetrike. s quhet bosht i simetrisë të saj .

Përkufizimi 7. Segmenti i përcaktuar nga meset e bazave te K. të S. quhet


vijë e mesme e K.të S.

Pohimi 11. Drejtëza e përcaktuar nga vija e mesme e K. të S. është bosht


i simetrisë për atë katërkëndësh.
Vërtetim. Nga se s: ABCD  BADC  s është bosht simetrije për
ABCD. □

4.2. Transformimet izometrike në rrafsh

Me anë të aksiomave të kongruencës do të përkufizojme një lloj të veçantë


transformimesh të rrafshit që quhen transformime izometrike.
I Hyrje 81

Përkufizimi 1. Le të jetë  cili do një rrafsh. Transformimi f  S i tillë


që për çdo dy pika A,B  ,
AfBf  AB

quhet transformim izometrik (apo izometri) e rrafshit .

Bashkësine e të gjitha izometrive të  do ta shënojmë me .

Me qenë se për  A, B  , 1: A,B A,B dhe ABAB  1  


.

Pohimi 1.  është grup (jokomutativ) në lidhje me produktin e


pasqyrimeve.

Vërtetim . Le të jenë f,g dhe f : A, B  A ', B ', g : A ', B '  A '', B ''.
Atwherë AB  A ' B '  A ' B '  A '' B '' . Me qenë se f g : AB  A '' B ''
dhe AB  A '' B '' rrjedh se f g  . Gjithashtu, nga f 1 : A ' B '  AB
dhe A ' B '  AB rrjedh f 1 . Pra,  është nëngrup i grupit S . □

Pohimi 2. Simetria boshtore është izometri. □

k
Shënojmë me ’={f f=   si kN} bashkësinë e të gjitha
1
izometrive që janë prodhime të fundme të simetrive boshtore. Prodhimi i
dy izometrive te tilla është përsëri prodhim i fundëm simetrish boshtore.
Për f   s1  s2  sk kemi
f -1 =  sk  s2  s1  ’. Pra,  është nëngrup i grupit . Elementi njësh
1=ss ’, s drejtez ne .

Do të vërtetojme tani se   ’, me fjalë të tjera, çdo izometri në 


mund të paraqitet si prodhim i fundëm i simetrive boshtore. Për këtë
qëllim po japim disa pohime ndihmëse.

Lema 1. Në qoftë se ABA’B’, ekziston f ’, i tillë që f: A,BA’, B’.


Vërtetim . Le të jetë s1=sAA’ simetralja e AB. Atëherë  s1 : A, BA’, B1
dhe ABA’B1. Nga relacioni i fundit dhe supozimi i Lemës kemi
82 I Hyrje

A’B1A’B’ , prej ku A’ s2= s B1B ' . Po të shënojmë me f   s1  s2 , atëherë


f’ dhe f : A,BA’, B’.□

Lema 2. Në qoftë se për ∆ABC dhe ∆A’B’C’ vlen ABA’B’, BCB’C’,


CAC’A’ , atëherë ekziston f-prodhimi i fundëm i simetrive boshtore i
tillë që
f : A,B,C  A’, B’, C’ .
Vërtetim . Me qenë se ABA’B’ , nga L.1.  f   s1  s2 : A,BA’, B’.
Po të jetë Cf = C1, atëherë ACA’C1 dhe BCB’C1 që sëbashku me
konnditën e lemës na japin A’C’A’C1, B’C’B’C1 që do të thotë se A’,
B’  sC 'C1  s3 .
Së fundi, pasqyrimi f1=  s1  s2  s3 : A,B,C  A’, B’, C’ .

Lema 3. Izometria në rrafsh që fikson tri pika jokolineare është identitet.

Vërtetim . Vërtetojme se f : PP, për P. Vërtet, sikur të ekzistonte


ndonje P, ashtu që Pf=P’P, atëherë nga f : A, B, C, P  A, B, C, P’
kemi APAP’, BPBP’, CPCP’ prej ku del se pikat A, B, C  s PP ' , gjë
që s’është e mundur, sepse A, B, C janë pika jokolineare. Pra, izometria f
i fikson te gjitha pikat e rrafshit  , rrjedhimisht f =1. □

Pohimi 2.(teoremë) =’ .

Vërtetim . Le të jetë f   cili do një element dhe A,B,C pika


jokolineare te . Le të jetë f : A,B,C  A’, B’, C’. Nga ABA’B’,
BCB’C’, ACA’C’ dhe L.2. , f1=  s1  s2  s3 ’, ashtu që f1 : A, B, C
 A’, B’, C’. Por atëherë izometria f1f –1 pikat A,B,CA,B,C,
rrjedhimisht f1f –1= 1  f = f1  ’. Kështu vërtetuam se
 ’ dhe me që ’   kemi ’=. □

Vërtetuam se çdo izometri në  mund të paraqitet si prodhim i tri


simetrive boshtore. Me qenë se simetritë boshtore janë transformime
kolineare që e ruajne renditjen, atëherë edhe izometrite janë transformime
kolineare që e ruajnë renditjen. Prandaj, çdo izometri, drjetëzën e
pasqyron në drejtëz, gjysmëdrejtëzën në gjysmëdrejtëz, këndin në kënd
kongruent me të, dhe drejtëzat normale në drejtëza normale.
I Hyrje 83

Pohimi 3. Izometria në rrafsh e kziston dhe është e vetme me dhënjen e


dy tresheve të pikave jokolineare A,B,C dhe A’, B’, C’ që plotësojnë
konditat
ABA’B’, BCB’C’, ACA’C’.

Vërtetim . Ekzistenca rrjedh nga L.2. Po të supozojmë se ekzistojnë dy


izometri f, f ‘ : A,B,CA’,B’,C’, atëherë f ‘f –1 : A,B,C A,B,C
rrjedhimisht f ‘f –1=1 , gjegjësisht f = f ‘ . □

k
Përkufizimi 2. Le të jetë f = 
1
si .

f quhet izometri direkte apo indirekte, varësisht nga fakti se numëri k


është çift apo tek.

Le të jetë + bashkësia e të gjitha izometrive direkte ne . Lehtë shihet


se + është nëngrup i grupit . Pra, kemi +   S .

Do të vërtetojmë tani një teoremë të rendësishme për izometritë e rrafshit.

Pohimi 4. Izometria në rrafsh që fikson një gjysmëdrejtëz është ose


identitet ose simetri boshtore me bosht të përcaktuar nga ajo
gjysmëdrejtëz.

Vërtetim . Le të jetë a=AB+ gjysmëdrejtëz e tillë që af=a . Atëherë Af=A


. Vërtetojmë se Bf=B, për Ba. Vërtet, nga se Bf a dhe ABA(Bf)
kemi Bf=B.
Po të jetë s= Aaa* , pasi f është kolinear dhe e ruan renditjen , atëherë
a*f =a*.
Pra, izometria f i fikson të gjitha pikat e drejtëzës s.Le të jetë C, Cs
dhe Cf = C’. Atëherë ACAC’ dhe BAC  BAC’…(1)
(i) Në qoftë se pikat C,C’ janë nga njëra anë e s, nga (1) kemi CC’,
rrjedhimisht f=1.
(ii) Në qoftë se C,C’ janë në anë të ndryshme të s, atëherë CC’ s ={D}.
Me qenë se f : A,C,D  A,C’,D  CDC’D, ADCADC’ dhe me
qenë se këta janë edhe të përbrinjshëm, ata janë te drejtë. Nga d(CC’) s
dhe CDC’D  C’=Cs. Pra në këtë rast f = s . □

Nga pohimi i mësipër kemi këtë rrjedhim:


84 I Hyrje

Pohimi 5. Izometria indirekte në rrafsh që ka dy pika fikse është simetri


boshtore me bosht të përcaktuar nga ato dy pika, kurse izometria direkte
që ka dy pika fikse është identitet.□

Përkufizimi 3. Le të jenë F, F’ figura në rrafshin . Do të themi se figura


F është kongruente me figurën F’, dhe simbolikisht do të shënojmë F 
F’, në qoftë se  f   ashtu që Ff = F’.

Me qenë se (, ) është grup, rrjedh që kongruenca në bashkësinë e te


gjitha figurave të rrafshit  është relacion ekuivalence.

Rregullat për kongruencën e trekëndëshave


Pohimi 6. (Rregulla e III) Në qoftë se brinjët AB, BC, CA të ∆ABC janë,
përkatësisht, kongruente me brinjët A’B’, B’C’, C’A’ të ∆A’B’C’ ,
atëherë ∆ABC ∆A’B’C’ .

Vërtetim . Vërtetimi është rrjedhim i L.2. □

Pohimi 7. Në qoftë se ab  a’b’ , ekziston izometria f , e tillë që f :


ab  a’b’ .
Zona e brendshme e ab pasqyrohet në zonën e brendshme të a’b’ .

Vërtetim . Le të jenë O, O’ kulmet e këndeve në fjalë kurse s = sOO’.


Atëherë  s1 : O, a, b  O’ a1, b1 dhe ab  a1b1. Nga relacioni i
fundit dhe relacioni i dhënë kemi a’b’  a1b1. Le të jetë s2 = s a1a '
simetralja e a1a’ dhe  s2 : a1, b1a’, b2. Atëherë kemi a1b1 a’b2.
Pra,  a'b'   a'b2 . Në qoftë se b’, b2 ndodhen nga njëra anë e a’ do
të kemi b2b’ dhe produkti  s1  s2 : ab  a’b’. Në qoftë se b’, b2
ndodhen në anë të ndryshme, drejtëza s3 e përcaktuar nga a’ do të jetë
simetrale e b2b’ dhe  s3 : a’, b2  a’, b’. Kështu produkti f =  s1  s2  s3
: aba’b’. Pjesa e dytë e vërtetimit rrjedh nga fakti se izometria e
ruan renditjen.□
I Hyrje 85

Pohimi 8. (Rregulla e I) Në qoftë se për ∆ABC dhe ∆A’B’C’ vlen:


ABA’B’, ACA’C’, BACB’A’C’ ,
atëherë ∆ABC∆A’B’C’.
Vërtetim . Nga L.3.1.1.  BCB’C’ dhe tani nga P.6. rrjedh
∆ABC∆A’B’C’. □

Vërtetimin e P.8. mund ta bëjmë edhe duke u mbështetur në


P.7. Nga BACB’A’C’ rrjedh se f  , f : BACB1A’C1. Por
atëherë A’B1ABA’B’ dhe me që B’, B1 janë në të njëjtën gjysmëdrejtëz
rrjedh BB’. Po kështu vërtetohet se C1C’. Pra,

f : A,B,C  A’, B’, C’ , rrjedhimisht ∆ABC∆A’B’C’. □

Pohimi 9. Në qoftë se dy kënde dhe një brinjë të ∆ABC janë kongruente,


përkatësisht, me dy kënde dhe një brinjë të ∆A’B’C’, atëherë
∆ABC  ∆A’B’C’ .

Vërtetim . Do të dallojmë dy raste:


a) Këndet e dhënë kongruent janë mbi brinjët e dhëna kongruente
(Rregulla e II).
b) Këndet e dhënë kongruent nuk janë mbi brinjët e dhëna
kongruente(Rregulla e IV).

Le të jenë ABA’B’, A  A’, B  B’. Mjafton të vërtetojmë se


AC A’C’ se atëherë vërtetimi do të rrjedhë nga P.8.(rregulla e parë). Po
të jetë ACA’C’, mund të supozojmë se AC<A’C’. Atëherë do të D,
(A’-D-C’), ashtu që ACA’D. Tani ∆ABC∆A’B’D (I) 
A’B’DBB’, që është në kundërshtim me aksiomen III4.
Le të jenë ABA’B’,  A A’, C  C’. Nga L.3.3.4 ACA’C’,
prandaj ∆ABC∆i’ (nga I).

Pohimi 1o. (Rregulla e pestë) Në qoftë se dy brinjë dhe këndi përballë


brinjës më të madhe të ∆ABC janë, përkatësisht, kongruente me dy brinjë
dhe këndin përballë brinjës më të madhe të ∆A’B’C’, atëherë
∆ABC∆A’B’C’.
86 I Hyrje

Vërtetim . Le të jenë ABA’B’, ACA’C’, AC>AB dhe BB’ . Nga se


BB’, f , f : B’B, A’, C’ A1, C1. Nga A1BA’B’AB
 A1A, pra ∆A’B’C’∆A1BC1∆ABC1. Në qoftë se (B-C1-C), nga
AC1A’C’AC  ∆ACC1 është dybrinjënjëshëm, prandaj
ACBAC1C > ABC1ABC, prej ku rrjedh se AB > AC, që është në
kundërshtim me supozimin. Po kështu vërtetohet se nuk mund të jetë (B-
C-C1), rrjedhimisht C1C, prandaj ∆ABC∆A’B’C’. □

4.3. Zbatimi i transformimeve izometrike

Përkufizimi 1. Trekëndshi, një kënd i të cilit është i drejtë quhet


trekëndësh kënddrejt. Brinja përballë këndit të drejtë quhet hipotenuzë,
kurse dy brinjët të tjera quhen kateta.

Pohimi 1. Dy këndet të tjerë të trekëndshit kënddrejtë janë të ngushtë.


Hipotenuza është më e madhe se katetat.

Vërtetimi rrjedh nga P...(teorema per kendin e jashtem te ∆ dhe pohimit


përballe kendit më të madh te ∆ ndodhet brinja më e madhe).

Pohimi 2. Le të jetë S pikë ne ab me kulm në O kurse Sa, Sb projeksionet


normale të pkës S në krahët a, b.
Drejtëza s = d(OS) = sab  SSa = SSb.
Vërtetim. Në qoftë se s=sab atëherë s : O,S,Sa  O,S, S’a dhe SS’a
 b  S’a Sb. Me qenë se s: S, Sa  S, Sb  SSa= SSb.

Anasjelltas, në qoftë se SSa= SSb, atëherë ∆OSSa ∆OSSb (rr.e V),


prandaj f   , ashtu që f : O,S,Sa  O,S,Sb . Nga se OS+OS+  f
është identitet apo simetri boshtore. Me qenë se f : Sa  Sb, f = s , ku
s=d(OS), pra s është simetrale e ab . □
Në gjeometrinë absolute nuk mund të vërtetohet se simetralet e dy
brinjëve të trekëndshit priten, por vërtetohet ky pohim:

Pohimi 3. Le të jenë s1 , s 2 , s3 simetralet e brinjëvë AB, BC , CA të ∆ABC.


Në qoftë se simetralet s1 , s 2 priten në pikën S, atëherë edhe s3  S .
I Hyrje 87

Vërtetim. Me qenë se Ss1s2  SASB dhe SBSC, prej ku  SASC,


a kjo do të thotë se Ss3. □

Prerja e simetraleve të këndeve të trkëndëshit në gjeometrinë absolute


mund të vërtetohet.

Pohimi 4. Simetralet e këndeve të trekëndëshit priten në një pikë.

Vërtetim. Le të jenë s A , s B , sC , përkatësisht, simetralet e këndeve të


∆ABC. Shënojmë me {A1}  s A  BC . (Ekzistenca e pikës A1 rrjedh nga
T.2.9.5.). Po nga kjo teoremë rrjedh se s B  AA1  {O}. Shënojmë me
A’, B’, C’ projeksionet normale të pikës O, përkatësisht, në d(BC), d(CA),
d(AB). Pasi që OsAOB’OC’. Por OsB  OC’OA’. Dhe së fundi
nga OAOB’  OsC. □

Për vërtetimin e teoremave të mëtejshme është e rëndësishme edhe ky


pohim:

Pohimi 5. Për çdo izometri f dhe çdo drejtëz s vlen:

 sf  f 1 s f .

Vërtetim. Le të jetë X  sf cila do pikë e drejtëzë sf. Atëherë


Xf 1  s , prandaj

( Xf 1 ) s  Xf 1 , rrjedhimisht ( X ) f 1 s f  X . Me qenë se izometria


f 1 s f i fikson të gjitha pikat e drejtëzës sf , rrjedh që f 1 s f është
ose 1 ose sf . Me qenë se f 1 s f =1 jep s=1, që s’është e mundur,
rrjedh se f 1 s f =sf . □

Pohimin e mësipërm e zbatojmë në vërtetimin e keti pohimi:

Pohimi 6. Le të jene a, b, c  O. Drejtëza s është simetrale e ab


atëherë dhe vetëm atëherë në qoftë se asbs = 1.

Vërtetim. : Nga s  sab  a s  b, prej ku


 b   a s   s a s   s b   a s   a s b s  1.
88 I Hyrje

: Le të jetë  a s b s  1. Atëherë
 a   s b s   b s  a  b s  s  s ab . □

Pokështu vërtetohen edhe pohimet:

Pohimi 7. Le të jetë (A-M-B), kurse a, b, m drejtëza, përkatësisht nëpër


A, B, M dhe normale në d(AB). m  s AB   a m   m b . □

Pohimi 8. Le të jenë a, b cilat do dy drejtëza në rrafshin . Atëherë

a  b   a  b   b a . □

4.4. Rrotacioni dhe translacioni

Dimë se çdo izometri në rrafsh mund të paraqitet si prodhim i më së


shumti tri simetrive boshtore. Para se ta përkufizojmë rrotacionin dhe
translacionin vërtetojme se:

Pohimi 1. Prodhimi i dy simetrive boshtore nuk mund të jetë simetri


boshtore.

Vërtetim. Sikur  a b   c , atëherë për Cc do të kishim


C a b  C c  C

 C a  C b  C '  a  b  sCC ' , prandaj  c   a a  1 , që nuk


ështe e mundur, sepse simetria boshtore ështe e ndryshme nga identiteti.

Nga P.1. rrjedh:

Pohimi 2. Prodhimi i tri simetrive boshtore nuk mund të jetë identitet. □

Shikojmë tani pikat dhe drejtëzat fikse të prodhimit të dy simetrive


boshtore.

Pohimi 3. Le të jenë a,b dy drejtëza të ndryshme.

X a b  X  {X }= ab .
I Hyrje 89

Vërtetim . Në qoftë se X i takon edhe a edhe b, atëherë X është pikë fikse


edhe ndaj a edhe ndaj b, pra edhe ndaj produktit të tyre. Anasjelltas, le
të jetë X a b  X . Po të jetë Xa=X’, atëhere X’b=X . Me qenë se
ab rrjedh se X  X’ ( përndryshe XX’ do të kishte dy simetrale të
ndryshme a dhe b. Tani nga Xa=X dhe Xb=X  Xa,b. □

Pohimi 4. Le të jenë a,b drejtëza të ndryshme jo ortogonale.


x a b  x  x  a dhe x  b.

Vërtetim. Në qoftë se xa, b , atëherë x është fikse ndaj a, b, pra edhe
ndaj ab. Anasjelltas, nga xab = x kemi xa = xb= x’, prej ku, pasi
që a dhe b nuk janë ortogonale, rrjedh x=x’. Dhe së fundi, nga xa = x
dhe xb= x kemi x a,b. □

Përkufizimi 1. Bashkësia e të gjitha drejtëzave në rrafsh incidente me


një pikë P quhet tufë eliptike apo tufë qendrore drejtëzash. Pika P quhet
qendër e tufës. Tufën me qendër P do ta shënojmë TP. Bashkësia e të gjitha
drejtëzave të rrafshit normale në një drejtëz të dhënë p quhet tufë
hiperbolike drejtëzash. Drejtëza p quhet bazë e tufes hipëerbolike. Tufën
hiperbolike me bazë p do ta shënojmë Tp.

Pohimi 5. (Teorema mbi tri simetritë) Në qoftë se a,b,c janë drejtëza të


një tufe T, atëherë  a b c   s dhe s T.

Vërtetim . a) Le të jenë a,b,cTP dhe a1 njëra nga gjysmedrejtëzat e a


me fillim në P. Le të jetë a1 b c  a1 '. Shënojmë me s  s a1a1 ' . Atëherë
sTP dhe a1 b c s  a1 ' s  a1 . Me qenë se izometria  b c s e
fikson gjysmëdrejtëzën a1,  b c s është ose 1 ose simetri boshtore
me bosht të përcaktuar nga a1. Me qenë se prodhimi i tri simetrive
boshtore nuk mund të jetë simetri boshtore, rrjedh se  b c s = a, prej
ku del se  a b c   s . □

Vlen edhe pohimi i anasjelltë i P.5.

Pohimi 6. Në qoftë se  a b c   s , atëherë a,b,c,s janë drejtëza të një


tufe.
90 I Hyrje

Vërtetim . Supozojmë se a b={P}. Atëherë P është pika e vetme fikse


e produktit ab. Nga  a b c   s  ab=sc dhe me qenë se
(P)sc =(P)ab=P, nga P.3. rrjedh Ps,c. Në qoftë se a,b  p , atëherë
pab=p. Me qenë se (p)sc=(p)ab=p, nga P.4. pa,b. □

Pohimi 7. Le të jenë a,b,c,d drejtëza të një tufe T. Në qoftë se ekziston


izometria direkte f –prodhim i fundëm i simetrive boshtore me boshte nga
T, e tillë që f : a,b  c,d , atëherë ab=cd .

Vërtetim . Me qenë se çdo izometri mund të paraqitet si prodhim i me së


shumti tri simetrive boshtore, atëherë izometria direkte f = mn, m,nT.
Nga kondita e dhënë kemi c=amn dhe d=bmn . Tani c =
 a m n  ( m n ) 1 a ( m n )   n m a m n . Në të njëjtën mënyrë
gjemë d =  n m b m n . Dhe në fundi

cd =(  n m a m n )(  n m b m n ) =  n m a  b m n . Me qenë se
a,m,nT   n m a është simetri boshtore, rrjedhimisht  n m a =(
 n m a )-1=  a m n . Pokështu,  b m n =  n m b . Prandaj, cd =
 n m a  b m n =  a m n  n m b = ab. □

Pra, paraqitja e rrotacionit ( translacionit) si prodhim i dy simetrive


boshtore nuk është e vetme. Dy çifte drejtëzash (a,b) dhe (c,d) të tufës T
e përcaktojnë të njëjtin rrotacion (translacion) në qoftë se ekziston
izometria direkte f = mn , m,n T, e tillë që f: a,b  c,d.

Tani mund ta vërtetojmë këtë pohim:

Pohimi 8. Rrotacioni (translacioni) përcaktohet nga çifti i dy drejtezave


të tufes eliptike (hiperbolike) prej të cilave njërën mund ta zgjedhim sipas
dëshirës.

Vërtetim . Le të jetë a  b = {P} dhe  P   a b dhe c TP cila do.


Shënojmë me a1, c1 gjysmëdrejtëza, përkatesisht në a,c me fillim në pikën
P. Shënojmë me s  s a1c1 fig…). Izometria sc : a  c. Shënojmë me
d = bsc. Nga se s,c TP dhe sc : a,bc,d , atëherë nga P.7. kemi
 P   a b = cd . Pra,  P = cd , ku cTP e zgjedhur sipas dëshirës.Në
mënyrë të ngjashme bëhet vërtetimi kur kur drejtëzat a,b janë të tufës
hiperbolike.□
I Hyrje 91

Shënim. Në qoftë se me b1 shënojmë ghysmëdrejtëzën e b me fillim


në P dhe me s1  s b1c1 , atëherë  s1  c : a, b  d1 , c , prandaj  P   a b
=  d1  c . Pra,  P   a b = cd =  d1  c .

Le të jetë A ={(a,b) a,b drejtëza prerëse } bashkësinë e çifteve të


renditur të drejtëzave që priten. Në bashkësinë A do ta përkufizojmë
relacionin   A 2 në këtë mënyrë:

(a,b)  (c,d)  Në qoftë se ekziston izometria direkte f  + ashtu që


f : a,b  c,d . Me qenë se + është grup rrjedh se  është relacion
ekuivalence. Le të jetë KO = A /={=C(a,b) a,b A } bashkësia e te
gjitha klasëve të ekuivalencës të relacionit .

Përkufizimi 2. Çdo element të bashkësisë KO do ta quajmë kënd të


orientuar.

Edhe këndet e orientuar simbolikisht do ti shënojmë me shkronjat greke


, , etj. Pra,

 = C(a,b) = {(x,y) A  (x,y)  (a,b)}.

Le të jetë  = C(a,b) dhe a  b = {P}. Po të jetë (c,d)   një


përfaqësues tjetër i këndit të orientuar  dhe c  d = {Q}, atëherë C(c,d)
= C(a,b) = . Por  P   a b   Q   c d . Pra këndi i orientuar  në çdo
pikë të rrafshit përcakton nga një rrotacion. Nga ana tjetër po të jenë m,n
drejtëza të tufës TP dhe mn : a,b  g,h, atëherë ab=gh dhe
C(g,h)=C(a,b)=. Kështu gjetëm edhe një përfaqësues (g,h) të kendit të
orientuar  që e përcakton të njëjtin rrotacion .

Shënojmë me P bashkësinë e të gjithë rrotullimeve me qendër në


P.

Pohimi 9. Ndërmjet bashkësisë P dhe bashkësisë KO të këndeve të


orientuar mund të vendoset një korespondencë bijektive.

Vërtetim .Le të jetë =C(a,b)  KO një kënd i orientuar dhe a  b ={Q}.


Në qoftë se Q  P, atëherë  P   a b është rrotullimi që do të ia
shoqëronim këndit . Në qoftë se Q  P do të shënojmë me s1=d(PQ) dhe
92 I Hyrje

me s2=sPQ. Po të shënojmë  s1  s2 : a,ba’,b’ , atëherë C(a’,b’)=C(a,b)=


dhe  P   a ' b ' është rrotullimi që ia shoqërojmë këndit të orientuar .
Shoqërimi i këtillë nuk varet nga zgjedhja e përfaqësuesit të  dhe është
reciprokish univok.□

Le të jetë  P   a b : XX’ dhe  = C(a,b). Lehte shihet se


XAX’=2a1b1, ku a1, b1 janë gjysmëdrejtëza përkatësisht në a, b me
fillim në pikën P. Këndi  = 2a1b1 quhet këndi i rrotullimit të  P dhe
shpesh  P shënohet edhe  P , apo  P , , ku  =C(a,b).

Përkufizimi 3. (Mbledhja e këndeve të orientuar) Le të jenë ,,  


KO.

(i)  =  +    P ,   P ,  P ,  ;

(ii)  = -    P,  (  P, ) 1 ;

(iii)  = 0   P ,  1 ;

(iv)  është kënd i shtrirë   P ,   a b , a  b .

Pohimi 10. Mbledhja e këndeve i gëzon këto veti:

1.  + 0 = 0 +  =  , për çdo ;

2. ( + ) +  =  + ( + ) , për çdo , , .

3.  +  =  +  , për çdo ,  ;

4.  +  =  +  , për çdo ,  ;

Vërtetim. 1. Me qenë se
 P , 0   P ,   P ,0   P ,  1   ,  1   ,   P ,0   P ,   P ,0 , atëherë
vlen  + 0 = 0 +  = . Relacioni 2. vlen përshkak të vetisë associative
të produktit të pasqyrimeve. Vërtetojmë vetinë e tretë: Le të jenë =C(a,b)
, ’=C(a’,b’ ), a  b = {O}, a’  b’ = {O’}. Ekziston drejtëza c O ashtu
I Hyrje 93

që  =C(b,c). Atëherë  O ,   a b dhe O ,    b c dhe


O ,    O , O ,    a b b c   a c ….(1)

Po ti marrim drejtëzat d,e  O ashtu që  =C(a,d) dhe =C(d,e) , atëherë


 O ,   a b   d  e ….(2) dhe O ,    a d   b c ….(3)

Shikojmë tani  O ,     O ,   O ,   a d  d  e   a e ….(4) Nga


relacioni (2) kemi  e   d a b dhe po të zëvendësojmë në (4) do të
kemi:

O ,     a e =  a d a b  (nga (2))   b c a b   a c b b   a c
…(5)

Nga (1) dhe (5) rrjedh vetia komutative për mbledhjen e këndeve të
orientuar. □

Nga konditat e mësipërme konstatojme se:

Pohimi 11. ( K0 , ) dhe ( RP , ) janw grupe komutative.

Le të jetë =C(a,b), a  b = {P} dhe  P ,   a b . Atëherë


 P,  (  P, ) 1  ( a b ) 1   b a . Kjo do të thotë se (b,a) -  -
= C(b,a).

Pra, po të jetë (a,b) përfaqësues i këndit të orientuar , atëherë (b,a)


është përfaqësues i këndit të orientuar -. Me marrveshtje do të shënojmë
-(a,b) = (b,a).

Le të jenë a1  a , b1  b dhe s simetralja e a1b1. Atëherë s:


((a,b)(b,a)= -(a,b).

Kemi thënë përpara se dy çifte (a,b) dhe (a’,b’) e përfaqësojnë të njëjtin


kënd të orientuar, në qoftë se ekziston izometria direkte që njërin çift e
pasqyron në tjetrin. Pasi që prodhimi i izometrisë direkte dhe simetrise
boshtore është izometri indirekte, atëherë vlen ky pohim:
94 I Hyrje

Pohimi 12. Le të jetë (a,b) përfaqësues i këndit të orientuar . Çifti (c,d)


do të jetë përfaqësues i këndit -, në qoftë se ekziston izometria indirekte
h që (a,b) e pasqyron në (c,d). □

4.5. Simetria qendrore

Përkufizimi 1. Rrotullimi  P   a b , ab quhet simetri qendrore.


Qendra e rrotullimit quhet qendër e simetrisë. Simetrinë qendrore me
qendër P do ta shënojmë me P.

Me qenë se rrotullimi përcaktohet nga dy drejtëza incidente me qendrën e


tij, prej të cilave njërën mund ta zgjedhim sipas dëshirës, atëherëka vend
pohimi:

Pohimi 1. Simetria qendrore përcaktohet nga cili do çift i drejtëzave


ortogonale incidente me qendrën e saj. □

Nga na tjetër kur a  b   a b   b a , atëherë P =  a b   b a , pra


çifti i drejtëzave normale që e përcakton simetrinë qendrore nuk është i
renditur.

Pohimi 2. Simetria qendrore është involucion.

Vërtetim. (P)2=PP=  a b a b =  a b b a =1. □

Pohimi 3. (i) Pika e vetme fikse e P është qendra e saj.

(ii) P : x  x  x  P .

(iii) Në qoftë se P : A  A’ , atëherë P është mesi i segmentit


AA’ .
I Hyrje 95

Vërtetim. (i) Simetria qendrore është rrotacion dhe pika e vetme fikse
e rrotacionit është qendra e tij. (ii) Le të jetë x  P. Marrim drejtëzën y 
P, y  x. Atëherë P=xy dhe xP = xxy = xy = x. Anasjelltas, në
qoftë se x = xP  PP = P. (iii) Le të jetë A cila do pikë e ndryshme nga
qendra P .Shënojmë me a =d(AP) dhe bP, b  a . Atëherë A’ = AP =
Aab = Ab, rrjedhimisht b është simetrale e segmentit AA’, pra P mesi
i AA’. □

Pohimi 4. P  a  Pa = aP .

Vërtetim. Pa  a = aP . Atëherë a =  a P   P a P prej ku


rrjedh Pa = aP . Anasjelltas, nga Pa = aP  a = PaP =  a P
 a = aP  a  P . □

Përkufizimi 2. Për dy figura F, F’ themi se janë në simetri qendrore,


Në qoftë se ekziston S ashtu që S: FF’. Në qoftë se S’ S, atëherë
themi se S është qendër e simetrisë për figurën F.

Pohimi 5. Këndet kryqëzues janë në simetri qendërore. Qendra e


simetrisë është kulmi i përbashkët i tyre.

Vërtetim. Le të jenë ab dhe a*b* kënde kryqëzues dhe p=a  a*


 {P}, q=b  b*  {P}. Shënojmë me p’, q’ drejtëzat nëpër P dhe p’ 
p , q’  q. Atëherë P = pp’ = qq’ dhe app’ = ap’ = a* ,
bqq’ = bq’ = b* . □

Për drejtëzat që nuk kalojnë nëpër qendrën e simetrisë vlen kjo veti:

Pohimi 6. Në qoftë se drejtëza a nuk kalon nëpër qendrën s


simetrisë, atëherë a dhe aP janë normale në të njëjtën drejtëz.

Vërtetim. Le të jetë n  P dhe n  a . Atëherë n = nP  aP. □

Teoremen: Për çdo kënd të orientuar  dhe për çdo pikë P ekziston
përfaqësues (a,b) me qendër në pikën P që e kemi vërtetuar më parë tani
mund ta vërtetojme edhe në këtë mënyrë:
96 I Hyrje

Le të jetë  = C(x,y) , x  y = {Q}. Në qoftë se Q  P, s’kemi çtë


vërtetojmë. Në qoftë se Q  P, shënojmë me M mesin e segmentit PQ.
Po të shënojmë me a = xP dhe b=yP , nga se P është izometri direkte,
rrjedh që C(a,b) = C(x,y) =  . □

Pohimi 7. Produkti i dy simetrive qendrore është translacion ose


identitet. Anasjelltas, çdo translacion mund të paraqitet si prodhim i dy
simetrive qendrore.

Vërtetim. Në qoftë se qendrat A,B të dy simetrive qendrore A, B


përputhen, atëherë AB është identitet, prandaj le te jetë A  B.
Shënojmë me p= d(AB), kurse me a,b drejtëzat, përkatësisht, nëpër A,B
normale ne p. Atëherë, A = ap dhe B = pb. Prandaj, AB =
appb = ab =  - translacion me bazë p. Anasjelltas, po të jetë  =
ab translacion me bazë p dhe A, B prerjet e drejtëzës p, përkatësisht, me
a, b , atëherë  = ab = appb = AB. □

Le të jetë  = ab, a, b  p translacion me bazë p dhe {A}= a  p,


{B}= b  p. Le të jenë P,Q  p , cilat do. Po të shënojmë  :
a b a b
P  P1  P' dhe Q  Q1  Q' , me qenë se  është
izometri direkte, segmented PP’, QQ’ janë congruent dhe të një orientimi.
Gjithashtu, PP’=PP1+ P1P’= 2AP1+ 2P1B= 2(AP1+ P1B)= 2AB. Pokështu
shihet se QQ’= 2AB.

Për pikat koresponduese M, M’ që nuk i takojnë bazës në Gjeometrine


absolute nuk mund të vërtetojme se MM’ = 2AB.

Në Gjeometrinë absolute ka vend ky pohim për translacionin:

Pohimi 8. Translacioni përcaktohet nga çifti i dy pikave. Dy çifte


pikash (X, X’) dhe (Y, Y’) e përcaktojnë të njëjtin translacion, në qoftë se
ata i takojnë të njëjtës drejtëz, përcaktojnë segmente kongruentë dhe të
një orientimi.

Vërtetim.Shënojmë me p =d(XX’)=d(YY’).Le të jetë xX, xp,


kurse s1=sXX’. Translacioni

x 
=  x s1 : X  X 
s1
X ' . Pokështu tregohet se ekziston
translacioni
I Hyrje 97

1=  y s2 : Y  Y ' , ku s2=sYY’. Vërtetojmë se  = 1. Shënojmë me


s = sXY. Atëherë sy:xy, kurse s1d=(s1) sy. Me qenë se izometria
direkte f =sy : x, s1  y, d rrjedh që

=  x s1   y d . Shënojmë me Y1=Y. Nga se  : X, Y  X’, Y1


rrjedh se segmented XX’ dhe YY1 janë kolinearë, kongruentë dhe të një
orientimi. Por me që e njëjta vlen (nga supozimi i teoremës) edhe për
segment XX’ dhe YY’, rrjedh se YY1 dhe YY’ janë segmente kolinearë,
kongruentë dhe të një orientimi, prandaj Y1 Y’ . Tani nga Y = Y1  Y’
kemi Yyd = Y’, gjegjësisht Yd =Y’ që do të thotë se d = sYY’. Me që
simetralja e segmentit është e vetme, kemi d = s2. Kështu kemi gjetur se
1 =  y s2   y d = . □

AKSIOMAT E VAZHDUESHMËRISË

Koncepti i vazhdueshmërisë në gjeometri është konsideruar si evident dhe


gjer nga fundi i shekullit 19 ky kuptim nuk është aksiomatizuar. Siç e dimë,
Euklidi e cakton kulmin e tretë C të trekëndëshit barabrinjës ABC si pikë
të prerjes të rrathëve l ( A, AB), l ( B, AB) duke i konsideruar rrathët si vija
të vazhdueshme. Po kështu edhe gjatë ndërtimit të normales nga një pikë
në një drejtëz të dhënë prerjet e drejtëzës me rrethin janë konsideruar si
evidente. M.Pasch në veprën ‘Gjeometria e re’ (1882) e thekson nevojën
e aksiomatizimit të vazhdueshmërisë në Gjeometri. Bile që në kohën e
Greqisë antike u konstatua se aksiomatika e Euklidit nuk është e plotë.
Vetë Arhimedi në punimin e tij ‘Mbi rruzullin dhe cilindrin’ dha edhe pesë
aksioma të tjera, prej të cilave aksioma e pestë, që në përkthim ë lire
formulohet: ‘Duke e bartur një segment të dhënë në një drejtëz një numër
të fundëm here, çdo pike e saj mund të arrihet dhe tejkalohet’ mundëson
që çdo segmenti t’i shoqërohet një numër real pozitiv. Ky pohim i
formuluar pak më ndryshe gjeti vend edhe në sistemin aksiomatik të
Hilbertit si aksiomë e vazhdueshmërisë. Me pohimin e Cantorit theksohet,
të thuash, që drejtëza është e ngjeshur me pika dhe në bazë të saj vërtetohet
prerja e drejtëzes me rrethin. Aksiomat e vazhdueshmërisë në sistemin
aksiomatik të Hilbertit (Aksioma e Arhimedit dhe Aksioma e Cantorit)
janë ekuivalente me pohimin e Dedekindit dhe bëjnë të mundur që në
gjeometri të zhvillohet teoria e matjes së figurave në rrafsh dhe hapësirë,
98 I Hyrje

teori kjo në bazë të të cilës gjeometria i përmbush detyrimet që ia


përcakton edhe emri i saj, si shkencë e matjes.

5.1. MATJA E SEGMENTEVE. AKSIOMA E ARHIMEDIT.

Me aksiomat e vazhdueshmërisë mund të zhvillohet teoria e matjes së


segmenteve dhe këndeve.
Përkufizimi 1. Shënojmë me S bashkësinë e segmenteve. Në qoftë se çdo
a  S i shoqërojmë një numër real pozitiv m( a ) , ashtu që të plotësohen
konditat:
(1) Në qoftë se a  b  m(a )  m(b) ;
(2) Në qoftë se ( A  B  C ), atëherë
m([ AC ])  m([ AB])  m([ BC ]) ;
(3) Egziston segmenti e i tillë që m(e)  1

atëherë pasqyrimi m : S  R0 , a m(a) quhet funksion i matjes të


segmenteve. Numri m( a ) quhet masë apo gjatësi e segmentit a. Segmenti
e, masa e të cilit është 1 quhet njësi e matjes.
Kushti i parë tregon se matja është invariante ndaj transformimeve
izometrike (segmentet kongruentë kanë masë të njëjtë), kurse kushti i dytë
tregon vetinë aditive të funksionit të matjes.
Me aksiomën e Arhimedit mund të ndërtohet një funksion i tillë i matjes
për të cilin tregohet se është injektiv, kurse me aksiomën e Kantorit
tregohet se funksioni i matjes është surjektiv.
Nga përkufizimi shihet lehtë se funksioni i matjes është një funksion
injektiv. Vërtetimi i këtij pohimi rrjedh nga ky:

Teorema 1. Le të jenë a, b  S .
a  b  m(a)  m(b)
Vërtetim. Nga a  b  b  a  c  m(b)  m(a)  m(c)  m(a).
Anasjelltas, le të jetë m(a)  m(b) . Atëherë a  b , sepse sikur
a  b apo a  b do të kishim m(a)  m(b) apo m(a)  m(b) , që është në
kundërshtim me supozimin m(a)  m(b) . □

Do të vërtetojmë tani këtë pohim:


I Hyrje 99

Teorema 2. Me zgjedhjen e një njësie për matje e1 , për çdo segment a,


më të vogël se e1 , numri matës i tij njëvlerësisht përcaktohet.
Vërtetim. Le të jenë A, E1 pika të një gjysmëdrejtëze p me fillim në O1 ,
të tilla që [O1E1 ]  e1, [O1 A]  a . Nga a  e1 kemi (O1  A  E1 ) . Pra,
m([O1E1 ])  m(e1 )  1, m([O1 A])  m(a) .
Shënojmë me M1 mesin e [O1E1 ] . Atëherë m([O1M1 ])  m([M1E1 ]) dhe
1  m([O1E1 ])  m([O1M1 ])  m([M1E1 ]) , prej ku kemi
1
m([O1M 1 ])  m([ M 1 E1 ])  .
2
Për pikën A vlen vetëm njëri nga relacionet:
1. A  M1 

2. (O1  A  M1 )  .
3. ( M1  A  E1 ) 
1
Në rastin 1. m(a )  m([O1 A])  m([O1M 1 ])  , është plotësisht e
2
përcaktuar;
1 1
Në rastin 2. 0  m([O1 A])  m([O1M 1 ])  , pra 0  m(a )  ,kurse
2 2
1
në rastin 3. m([O1M 1 ])  m([O1 A])  m([O1E1 ]), pra  m(a )  1 .
2
Të dy rastet 2. dhe 3. mund t’i shkruajmë
n1 n 1
 m(a)  1 , n1  0,1 .
2 2
Në të dy këto raste numri m( a ) nuk është plotësisht i përcaktuar, por
ndodhet ndërmjet dy numrave të dhënë.
Shënojmë me e2  [O2 E2 ] atë gjysmë të segmentit [O1E1 ] që e përmban
pikën A. Pra, nëse vlen 2. shënojmë O1  O2 , M1  E2 e nëse vlen 3.
shënojmë M1  O2 , E1  E2 .
1
Në të dy rastet kemi (O2  A  E2 ) dhe m([O2 E2 ])  . Tani segmentin a
2
do ta masim me segmentin e2  [O2 E2 ] .
Shënojmë me M 2 mesin e segmentit e2  [O2 E2 ] . Nga
100 I Hyrje

1
 m([O2 E2 ])  m([O2 M 2 ])  m([ M 2 E2 ]) dhe m([O2 M 2 ])  m([M 2 E2 ])
2
1
kemi m([O2 M 2 ])  m([ M 2 E2 ])  2 .
2
Përsëri për pikën A kemi
1. A  M 2 

2. (O2  A  M 2 )  .
2. (O2  A  M 2 ) 
Për A  M 2 kemi
 1
m([O2 M 2 ])  22
m(a)  m([O1 A])  m([O1M 2 ]  
m([O M ])  m([ M M ])  1  1
 1 1 1 2
2 22
që mund t’a shkruajmë shkurt
n 1
m(a)  1  2 , ku n1  {0,1} .
2 2
1 1 1 1
Në rastin 2. kemi 0  m(a)  2 ose  m(a )   2 dhe të dy rastet
2 2 2 2
n1 n1 1
i shkruajmë  m(a)   2 , kurse për rastin 3.
2 2 2
1 1 1 1
2
 m( a )  ose   m(a)  1 dhe të dy rastet i shkruajmë
2 2 2 22
n1 1 n 1
 2  m( a )  1  .
2 2 2 2
Rastet 2. & 3. i shkruajmë shkurt
n1 n2 n n 1
 2  m(a)  1  2 2 , ni  {0,1} .
2 2 2 2
Nga relacionet e fundit shohim se m( a ) nuk është përcaktuar saktësisht,
1
por rrethina e tij është zvogëluar me saktësi deri në 2 . Këtë veprim e
2
vazhdojmë.
Shënojmë me e3  [O3 E3 ] atë gjysmë të segmentit [O2 E2 ] që e përmban
pikën A dhe me M 3 e shënojmë mesin e segmentit [O3 E3 ] . Prap për pikën
A i kemi këto tri mundësi:
I Hyrje 101

1. A  M3 

2. ( ()3  A  M 3 ) 
3. ( M 3  A  E3 ) 
Potë veprojmë si më lart, për rastin e parë do të kemi
n1 n2 1
m(a)    ,
2 2 2 23
kurse për rastet 2. dhe 3:

n1 n2 n3 n n n 1
 2  3  m(a)  1  22  3 3 , ni  {0,1}, i  1,2,3.
2 2 2 2 2 2
Duke vazhduar në këtë mënyrë gjithnjë e zvogëlojmë rrethinën e numrit.
Në hapin e k-të shënojmë me M k mesin e segmentit [Ok Ek ] që e përmban
pikën A.
Tani ekzistojnë këto dy mundësi:
(i) Ose ekziston k  N e tillë që A  M k
(ii) Ose një numër i tillë k nuk ekziston.
n n n
Në rastin (i) m(a )  1  22   kk , ni  {0,1} dhe plotësisht është
2 2 2
përcaktuar, kurse në rastin (ii)
n1 n2 n n n n 1
 2   kk  m(a)  1  22   k k , ni  {0,1},
2 2 2 2 2 2
n n n n n n 1
Shënojmë me xk  1  22   kk , yk  1  22   k k dhe
2 2 2 2 2 2
vërtetojmë se:
( xk ) është varg monoton jozbritës, gjejgësisht ( xk )  ( xk 1 ) ;
( yk ) është varg monoton jorritës, gjegjësisht ( yk )  ( yk 1 ) .

nk 1
Vërtet, nga xk 1  xk   0 (sepse nk 1 {0,1} ) kemi ( xk )  ( xk 1 ) ,
2k 1
1 n 1 1 n
kurse nga yk  yk 1  k  k k1 1  k k11  0 kemi ( yk )  ( yk 1 ) .
2 2 2
1 1
Me qenë se yk  xk  k dhe lim( yk  xk )  lim k  0 ,
2 k  k  2
nga vetitë e vargjeve monotone kemi
102 I Hyrje

lim yk  lim xk  m(a) .


k  k 

Pra, me dhënjen e segmentit njësi e1 , për çdo segment a, a  e1 , numri


mates m( a ) njëvlerësisht përcaktohet. □

Për segmentin a më të madh nga segmenti njësi, numrin m( a ) nuk mund


t’a caktojmë pa aksiomat e vazhdueshmërisë.

IV1 (Aksioma e Arhimedit) Le të jenë [ AB],[CD] cilit do dy segmente


dhe A1 , A2 , pika në gjysmëdrejtëzën AB  të tilla që
( A  A1  A2 ), ( A1  A2  A3 ), dhe [ AA1 ]  [ A1 A2 ]  [ A2 A3 ]   [CD] .
Atëherë ekziston numri natyral n, ashtu që
An  B ose ( An  A  An1 ) .
Vërtetojmë tani këtë pohim:

Teorema 3. Me dhënjen e segmentit njësi, numri matës i çdo segmenti


njëvlerësisht përcaktohet.
Vërtetim. Le të jetë e, m(e)  1 , kurse a cili do një segment. Në një
gjysmëdrejtëz p me fillim O, A, [OA]  a . Le të jenë E1 , E2 ,  p , ashtu

(O  E1  E2 ),( E1  E2  E3 ), dhe [OE1 ]  [ E1E2 ]  [ E2 E3 ]   e .
Nga aksioma IV1, n  N , ashtu që En  A ose ( En  A  En1 ) . Nga
konditat e funksionit të matjes kemi m([OEn ])  n, m([OEn1 ])  n  1 .
Prandaj, në rastin e parë kemi m(a)  n , plotësisht i përcaktuar, ndërsa në
rastin tjetër
m(a)  m([OA])  m([0En ])  m([ En A])  n  m([ En A]) .
Nga se ( En  A  En1 )  [ En A]  [ En En1 ]  e , nga Teorema 2 kemi se
edhe numri m([ En A]) është njëvlerësisht i përcaktuar. □

Teorema 4. Për çdo segment a (sa do i vogël që të jetë), n  N , i tillë


që me n përgjysmime të njëpasnjëshme të segmentit njësi e, fitohet
segment [On En ] më i vogël se segmenti a.
e e
Vërtetim. Vërtet, ekziston n  N , ashtu që n  a , sepse sikur a  n
2 2
, atëherë do të kishim e  2 a  na , që është në kundërshtim me
n

aksiomën IV1.□
I Hyrje 103

Teorema 5. Në qoftë se numri matës i segmentit a jepet me


n n n
m(a)  1  22   kk 
2 2 2
atëherë duke filluar nga ndonjë k, jo të gjitha nk janë të barabarta me 1.
Vërtetim. Po të supozojmë se, duke filluar nga ndonjë k, të gjitha nk janë
të barabarta me 1, atëherë [ AEk ]  [Ok Ek ] , për çdo k, që është në
kundërshtim me teoremën 4 që pohon se duke e përgjysmuar segmentin
njësi e, segmentin [Ok Ek ] mund t’a bëjmë më të vogël se çdo segment, sa
do i vogël që të jetë ai.□

Me teoremat 2 dhe 3 ne vërtetuam se po të ekzistojë funksioni


f : S  R0 që i plotëson konditat e përkufizimit 1, numri matës i çdo
segmenti njëvlerësisht përcaktohet. Do të vërtetojmë tani ekzistencën e
një funksioni të tillë.

Teorema 6. Egziston pasqyrimi m : S  R0 që i plotëson konditat e


përkufizimit 1.
Vërtetim. Le të jetë e një segment i fiksuar dhe a cili do një segment.
Duke vepruar si në vërtetimin e teoremave 2 dhe 3, mund të gjejmë
vargjet ( xk ) dhe ( yk ) , ashtu që
( xk )e  a  ( yk )e
n1 n2 nk n n nk  1
apo (n     )e  a  (n  1  22   )e , ni  {0,1}
2 22 2 k
2 2 2k
dhe lim yk  lim xk    R .
k  k 

Kështu, për çdo a  S , !  R .


Përkufizojmë pasqyrimin
m : S  R, m(a )    lim yk  lim xk .
k  k 

Me qenë se të gjithë numrat n, n1 , , nk nuk janë njëkohësisht të barabartë


1
me zero (përndryshe do të ishte a  k e , rrjedhimisht, e  2k a  ka ),
2
n n
egziston nt  0 . Atëherë xt  tt   tt  0 dhe ( xk ) është jozbritës ka
2 2
për rrjedhim
  lim xk  0 , pra m(a)  R0 .
k 
104 I Hyrje

Vërtetojmë se funksioni m i plotëson konditat 1-3 të përkufizimit 1.


(1) Le të jenë a, b  S , a  b .
Për a, e, xk , yk , xk e  a  yk e dhe m(a )    lim yk  lim xk .
k  k 

Pokështu, për b, e, x , y '


k
'
k x e  b  y e dhe m(b)    lim yk'  lim xk'
'
k
'
k
k  k 
.
Nga xk e  a  b  yk' e  xk  yk'  lim xk  lim yk'     .
k  k 

Ngjashëm, x e  b  a  yk e  x  yk  lim x  lim yk     .


'
k
'
k
'
k
k  k 
Pra, m(a)      m(b) .
(2) Le të jenë a, b, c  S dhe c  a  b . Vërtetojmë se m(c)  m(a)  m(b)
.
Për a, b, c, e le të jetë
xk e  a  yk e dhe m(a )    lim yk  lim xk ;
k  k 

x e  b  y e dhe m(b)    lim y  lim xk' ;


'
k
'
k
'
k
k  k 

xk e  c  y k e dhe m(c)    lim y k  lim x k .


k  k 

Nga dy relacionet e para kemi


( xk  xk' )e  a  b  ( yk  yk' )e .
Tani nga
( xk  xk' )e  a  b  c  y k e rrjedh ( xk  xk' )  y k për çdo k,
rrjedhimisht lim( xk  xk' )  lim y k  m(a)  m(b)  m(c) .
k  k 

Nga ana tjetër xk e  c  a  b  ( yk  yk' )e , prandaj xk  ( yk  yk' ) ,


rrjedhimisht lim x k  lim( yk  yk' ) apo m(c)  m(a)  m(b) .
k  k 
0 0 0 1
(3) Për a  e , xk  1    k , yk  1    k dhe
2 2 2 2
m(a)  lim xk  lim yk  1 . □
k  k 

Injektiviteti i funksionit m : S  R0 , a m(a) është


rrjedhim i Teoremës 1, kurse surjektiviteti i tij nuk është
rrjedhim i tre grupeve të para të aksiomave dhe
Aksiomës së Arhimedit. Prandaj po e formulojmë
aksiomën IV2.
I Hyrje 105

IV2 (Aksioma e Cantorit) Le të jetë


[ A1B1 ],[ A2 B2 ],
varg jo i fundëm segmentesh në një drejtëz që i plotëson këto kondita:
(1) Çdo segment pasardhës përmbahet në paraardhësin;
(2) Nuk egziston segment që përmbahet në të gjithë segmentet e
vargut.
Atëherë egziston pika X që përmbahet në të gjithë segmentet e atij vargu.

Teorema 7. Pasqyrimi m : S  R0 është surjektiv.


Vërtetim. Le të jetë r  R 0 dhe e njësia e matjes.
Në qoftë se r  Q në sistemin me bazë 2 r e paraqesim si shumë të fundme
n n n
r  n  1  22   kk , ni  {0,1} .....(*) .
2 2 2
Në qoftë se r është numër iracional (r  R  Q) atëherë
n n n
r  n  1  22   kk  , ni  {0,1} ...(**)
2 2 2

Le të jenë P1 , Q1  AM  , ( A  P1  Q1 ) të tilla që [ AP1 ]  ne, . [ PQ


1 1]  e .
Shënojmë me M1 mesin e segmentit [ PQ 1 1 ] . Shikojmë shifrën n1 të
numrit r.
Në qoftë se n1  0 , shënojmë P1  P2 , M1  Q2 ;
Në qoftë se n1  1 , shënojmë M1  P2 , Q1  Q2 .
Në të dy rastet [ PQ2 2 ]  [ PQ
1 1 ] dhe është sa gjysma e tij.
Shënojmë me M 2 mesin e segmentit [ P2Q2 ] dhe e shikojmë shifrën n2 të
numrit r.
Në qoftë se n2  0 , shënojmë P2  P3 , M 2  Q3 ;
Në qoftë se n2  1 , shënojmë M 2  P3 , Q2  Q3 .
Në të dy rastet [ PQ3 3 ]  [ P2Q2 ] dhe është sa gjysma e tij.

Këtë veprim e përsërisim deri te shifra nk .


Kështu kemi:
1 n 1
m([ AP1 ])  n, m([ AM 1 ])  n  , m([ AM 2 ])  n  1  2
2 2 2
n n 1
m([ AM 3 ])  n  1  22  3 ,
2 2 2
106 I Hyrje

n1 n2 nk 1 1 n n nk
m([ AM k ])  n     k 1
 k = n  1  22   .
2 22 2 2 2 2 2k

Në qoftë se r jepet me (*), atëherë [ AM k ] është segmenti i tillë që


m([ AM k ])  r .
Në qoftë se r jepet me (**), atëherë
(***) ..... [ PQ 1 1 ],[ P2Q2 ],[ PQ 3 3 ],
është varg jo i fundëm segmentesh kolinearë të tillë që çdo segment
pasardhës përmbahet në paraardhësin dhe është sa gjysma e tij. Nuk
egziston segment që përmbahet në të gjithë segmentet e vargut, sepse sikur
të egzistonte një segment i tillë [ XY ]  [ PQ i i ], i  1, 2, , nga
1
[ Pi 1Qi 1 ]  i [ PQ
1 1 ], i  1, 2, do të kishim
2
1
[ XY ]  [ Pn 1Qn 1 ]  n [ PQ1 1 ], n , apo
2
n[ XY ]  2n [ XY ]  [ Pn 1Qn 1 ]  [ PQ1 1 ], n

gjë që është në kundërshtim me aksiomën IV1.


Pra vargu (***) i plotëson konditat e aksiomës IV2, rrjedhimisht egziston
pika B [ PnQn ], n  1, 2, që i takon çdo segmenti të vargut dhe
n n n
m([ AB])  n  1  22   kk   r. □
2 2 2
Me qenë se segmentet kongruentë kanë masë të njëjtë, atëherë
vlen
Teorema 8. Pasqyrimi f : S  R0 i dhënë me
f (C[ AB ] )  m([ AB])
është bijeksion. □

Teorema 9. Le të jetë a  OA gjysmëdrejtëz me fillim në pikën O.


Pasqyrimi
f : OA  R0 , f ( A)  m([OA]), A  a  {O} është bijektiv. □

Tani jemi në gjendje që të vendosim një korespondencë bijektive ndërmjet


pikave në drejtëz dhe numrave reale.
Le të jetë d cila do një drejtëz dhe O një pikë e saj. Pika O e ndan drejtëzën
d në dy gjysmëdrejtëza a dhe a’, prej të cilave njërën, psh.
gjysmëdrejtëzën a do ta quajmë gjysmëdrejtëz pozitive, kurse tjetrën
I Hyrje 107

gjysmëdrejtëz negative. Pra,d mund të shkruhet d  {O}  a  a ' . Për çdo


pikë P  d shënojmë:
 m([OP]), P  a

x  0, P  O
m([OP]), P  a '

Anasjelltas, për çdo numër real x, egziston pika P  d , m([OP])  x .
Pikën P do t’a marrim në a apo a’ varësisht nga fakti se x>0 apo x<0. Nëse
x=0, për P do të marrim pikën O.

Teorema 10 (Pohimi i Dedekindit për pikat në drejtëz)


Në qoftë se të gjitha pikat e një drejtëze d i ndajmë në dy bashkësi joboshe
A,B të tilla që:
(i) Çdo pikë e drejtëzës d bën pjesë vetëm në njërën klasë;
(ii) Ndërmjet çdo dy pikave të njërës klasë nuk egzistojnë pika të
klasës tjetër;
Atëherë egziston pika e vetme X në d, ashtu që të gjitha pikat të tjera të
klasës A ndodhen nga njëra anë e saj, kurse të gjitha pikat të tjera të klasës
B ndodhen nga ana tjetër e pikës X.
Vërtetim. Me qenë se bashkësitë A,B janë jo boshe, A1  A
B1  B.
Shënojmë me M1 mesin e segmentit [ A1B1 ] . Pika M1 i takon drejtëzes d,
prandaj M1  A apo M1  B.
Nëse M1  A, shënojmë M1  A2 , B1  B2 .
Nëse M1  B, shënojmë A1  A2 , M1  B2 .
Në të dy rastet A2  A, B2  B,, [ A2 B2 ]  [ A3 B3 ] dhe është sa gjysma
e tij.
Shënojmë me M 2 mesin e segmentit [ A2 B2 ] . Pika M 2 i takon drejtëzes d,
prandaj M 2  A apo M 2  B.

Nëse M 2  A,, shënojmë M 2  A3 , B2  B3 .


Nëse M1  B, shënojmë A2  A3 , M 2  B3 .
Në të dy rastet A3  A, B3  B, [ A3 B3 ]  [ A2 B2 ] dhe është sa gjysma
e tij.
Duke vazhduar në këtë mënyrë e ndërtojmë vargun
[ A1B1 ],[ A2 B2 ], ,[ An Bn ], (*)
108 I Hyrje

e pafundëm të segmenteve kolinearë, çdo segment pasardhës i të cilit


përmbahet në paraardhësin dhe është sa gjysma e tij. Nuk ekziston segment
që përmbahet në të gjithë segmentet e vargut, se përndryshe, sikur
1
[ MN ]  [ An 1 Bn 1 ]  n [ A1 B1 ] do tw kishim n[ MN ]  2n [ MN ]  [ A1B1 ] ,
2
për çdo n, që është në kundërshtim me aksiomën IV1. Në bazë të të
aksiomës së Cantorit, egziston pika e vetme X, që përmbahet në të gjithë
segmentet e vargut. Me qenë se X është pike e d, ajo i takon vetëm njërës
prej bashkësive A dhe B , psh. bashkësisë B .
Po të jetë B cila ndonjë pike e B,, atëherë ajo ndodhet nga ajo anë e X, nga
janë edhe pikat Bk , sepse sikur të ishte ( B  X  Bk ) , atëherë do të
ekzistonte k sipas dëshirës e madhe, ashtu që ( B  Ak  X ) , pra do të
ekzistonte pikë e A ndërmjet dy pikave B dhe X të B,, që nuk është e
mundur.
Po të jetë A cila ndonjë pike e A , atëherë ajo ndodhet nga ajo anë e X nga
janë edhe pikat Ak , sepse sikur ( Ak  X  A) , pika X e B, do të ishte
ndërmjet dy pikave të klasës A .
Pra, ekziston pika e vetme X, e tillë që të gjitha pikat të tjera të A ndodhen
nga njëra anë e e saj, kurse të gjitha pikat të tjera të
B ndodhen nga ana tjetër e X.□
Shënim. Për pikën X themi se bën prerjen apo ndarjen e klasëve A dhe
B.

Duhet theksuar se aksiomat e vazhdueshmërisë nga njëra anë


dhe Pohimi i Dedekindit për pikat në segment janë pohime
ekuivalente në lidhje me tre grupet e para të aksiomave. Nga tre
grupet e para të aksiomave dhe nga aksiomat e
vazhdueshmërisë e vërtetuam Pohimin e Dedekindit. Do të
vërtetojmë tani se nga I, II, III dhe Pohimi i Dedekindit rrjedhin
pohimet IV1 dhe IV2.

Teorema 11. (I, II, III, PD  IV1)


Le të jenë [ AB ],[CD] çfarë do dy segmente, A1 , A2, ,, An ,
pika të gjysmëdrejtëzën AB  , ashtu që
B( A1 , A2 ,, An ,), [ AA1 ]  [ A1 A2 ]    [CD ] .
I Hyrje 109

Supozojmë të kundërtën, se të gjitha pikat A1 , A2, ,, An ,


ndodhen në segmentin [AB] .
Bashkësinë e të gjtha pikave të [AB] e ndajmë në dy bashkësi A
dhe B: Në bashkësinë A i futim të gjitha pikat që i takojnë
segmenteve [ AA1 ),[ A1 A2 ),,[ An1 An ], , kurse në B të gjitha
pikat të tjera të segmentit [AB] .
Me qenë se A  A, B B, bashkësitë A dhe B janë joboshe.
Nga mënyra e ndarjes së pikave shihet se çdo pikë e [AB] bën
pjesë vetëm në njërën nga bashkësitë A,B dhe ndërmjet dy pikave
të njërës bashkësi nuk ndodhen pika të bashkësisë tjetër. Pra,
ndarja e këtillë e pikave në klasë i plotëson konditat e Pohimit
të Dedekindit, rrjedhimisht egziston pika X që bën ndarjen e
klasëve A,B (në njërën anë të X ndodhen pikat e A, kurse në anën
tjetër pikat e B). E caktojmë tani pikën Y në drejtëzën AB, ashtu
që [ XY ]  [CD] . (*)
Me qenë se X e bën ndarjen e klasëve dhe Y ndodhet nga ajo anë
e X nga është pika A rrjedh se Y  A, pra Y ndodhet në ndonjërin
nga segmentet [ AA1 ),[ A1 A2 ),,[ An1 An ], , p.sh. në segmentin
[ Ak 1 Ak ] , por atëherë edhe pikat Ak , Ak 1 do të jenë në segmentin
e hapur (XY) prandaj [CD ]  [ Ak Ak 1 ]  [ XY ] , që është në
kundërshtim me (*).
Supozimi se të gjitha pikat A1 , A2, ,, An , ndodhen në [AB]
është kontradiktor, rrjedhimisht vlen IV1.
110 I Hyrje

Teoremat e Legendre-Saccherit

Do te të vërtetojme disa teorema që kanë të bëjnë me shumën e


këndeve në trekendësh që në literaturë njihen si teorema te
Legendre-Saccherit. Do të shënojmë me S(∆) shumën e këndeve
, ,  të një trekëndëshi çfarëdo ∆.

P.1. (Teorema e parë e Legendre-Saccherit) Për çdo ∆, S(∆) =


+  + ≤ 2d.
Vërtetim. Supozojmë të kundërtën, se  ∆1= ∆ABC, për të cilin
vlen
S(∆1) = +  + > 2d ……(*)
(figura …)
Shënojmë me a, b, c, përkatësisht brinjët B1C1, C1A1, A1B1.
Në gjysmëdrejtëzën C1B1+ ekzistojnë pikat C2, C3, …, Cn ashtu

(C1  C2  C3  ...  Cn ), C1C2  C2C3  ...  Cn1Cn  a.
Ndërtojmë (n-1) trekëndësha
VC1C2 A2 , VC2C3 A3 ,..., VCn1Cn An kongruentë me VA1B1C1 .
Këndet te kulmet Ci të trekëndshave VAC i i Ai 1 , i  1,..., n  1 janë

2d-(+)=. Nga supozimi (*) kemi  > 2d-(+)=. Shënojmë


me d segmented kongruente AiAi+1, i=1,…,n-1. Nga se te
trekëndëshat ∆A1B1C1, ∆C1A2A1 kemi A1B1C1A2, A1C1,  > 
rrjedh se a > d, apo a’=a-d është segment. Po kështu, nga
VA1 A2C1 kemi b  c  d b  c  d  b ' - segment.
I Hyrje 111

Nga konstruktimi i vërtetimit kemi b  c  (n  1)d  na (vija e


thyer B1 A1... An  B1Cn ), rrjedhimisht,
b '  b  c  d  n(a  d )  na ' , që është në kundërshtim me
aksiomën e Arhimedit. Pra, supozimi se ekziston trekëndësh me
shumë këndesh më të madhe se 2d është kontradiktor, kështu që
S(∆)≤2d, për çdo trkëndësh ∆. □
Si rrjedhime te P1 kemi pohimet:
P.2.Këndi i jashtëm i trekëndëshit është më i madh nga shuma e
dy këndeve të brendshëm, jo të përbrinjshëm me të.
Vërtetim. +  + ≤ 2d =  + *   +  ≤ *. (figura). □
P.3. Shuma e këndeve në çdo katërkëndësh është ≤ 4d.
Vërtetim. Me diagonalen, psh. BD katërkëndëshi
ABCD ndahet në dy trekëndësha ABD dhe BCD,
shuma e këndeve të të cilëve është e barabartë me
shumën e këndeve të katërkëndëshit ABCD dhe nga P1
rrjedh P3. □
P.4. Shuma e këndeve në çdo n-këndësh konveks është ≤
(n  2)2d .□
D.1. Ndryshimi 2d-S(∆) quhet defect i trekëndëshit ∆. Do ta
shënojmë me D(∆). Pra , D(∆) = 2d – S(∆).
Në qoftë se e identifikojmë këndin me masën e tij, nga
P1 kemi D(∆) ≥ 0.

P.5. (Teorema e dytë e Legendre-Saccherit)


Në qoftë se  ∆o, S(∆o) = 2d, atëherë S(∆) =2d, për çdo
trekëndësh ∆.
Apo në gjuhën e defektit: Në qoftë se  ∆o, D(∆o) = 0, atëherë
D(∆) =0, për ∆.

Vërtetimin e këti pohimi do ta bëjmë duke u


mbështetur në disa pohime ndihmëse.
L.1. Në qoftë se D është pikë e brinjës BC të ∆ABC,
atëherë
D(∆ABC) = D(∆ABD) + D(∆ACD).
Vërtetim. Po ti shënojmë me ∆, ∆1, ∆2, me rradhe,
trekëndshat e mësipërm, atëherë nga fig… kemi:
D(∆) = 2d – ( +  + );
D(∆1) = 2d – (1+  + 1);
D(∆2) = 2d – (2+  + 2).
112 I Hyrje

Nga dy relacionet e fundit kemi


D(∆1) + D(∆2) = 4d –(1+2+ + + 1+ 2) = 4d – (
+  +  + 2d) =
= 2d – ( +  + ) = D(∆). □

Si rrjedhime të L1 kemi:
a) D(∆i) ≤ D(∆), i=1,2.
b) Në qoftë se D(∆) = 0 
D(∆i) = 0, i=1,2.

L.2. Në qoftë se B1, C1 janë me rradhë pika të brinjëve


AB, AC të ∆ABC, atëherë D(∆AB1C1) ≤ D(∆ABC).
Vërtetim. Nga L1 kemi D(∆ABC) = D(∆ABC1) +
D(∆BCC1) =
=D(∆AB1C1) + D(∆BB1C1) +
D(∆BCC1) ≥ D(∆AB1C1). □

Nga L2 dhe P1 kemi si rrjedhim


Në qoftë se D(∆ABC) = 0, atëherë edhe D(∆AB1C1).

L3. Në qoftë se defekti i një trekëndëshi kënddrejtë


është i barabartë me zero, atëherë edhe defekti i
trekëndëshit kënddrejtë, njëra katete e të cilit dyfishohet
është i barabartë me zero.
Vërtetim. Le të jetë ∆=∆ABC, =d dhe D(∆ABC) = 2d
– ( +  + ) = 0. (fig…). Atëherë  +  = d. Le të jetë
D  ACB  , S CAD   , dhe AD  BC. Nga kongruenca e
ABC dheCDA  S D  d, D(VCDA)  D(V ABC )  0.
Prej këtu gjemë se ABCD është katërkëndësh me katër
kënde te drejtë.
Le të jetë C1  CB  , CC1. Nga
VDCC1 VABC  D(VDCC1 )  0.
Si më sipër e gjemë pikën D1, ashtu që DCC1D1 të jetë
katërkëndësh me katërkënde të drejtë. Për rrjedhim
kemi se ABC1D1 është katërkëndësh me katër kënde të
drejtë. Me diagonalen AC1 ky katërkëndësh ndahet në
dy trekëndësha kënddrejtë kongruentë
V1  ABC1 dhe V2  C1D1 A me defect të barabartë. Nga se
I Hyrje 113

S(∆1) + S(∆2) = 4d  S(∆i) = 2d, i=1,2. Pra


D(∆ABC1)=0 dhe BC1  2BC. □
L.4. (Teorema e dytë e LS për trekëndëshat
kënddrejtë ) Në qoftë se  trekëndëshi kënddrejtë ∆o,
D(∆o) = 0, atëherë defekti i çdo trekëndëshi kënddrejtë ∆
është i barabartë me zero.
Vërtetim. Le të jetë ∆o=∆ABC dhe ∆ = ∆A’B’C’.
(i) Në qoftë se katetat e trekëndëshit ∆ janë më të
vogëla se katetat e trekëndëshit ∆o, atëherë  trekëndëshi
∆1=∆ A1BC1 kongruent me ∆ dhe
( A  A1  B),(C  C1  B). Pra D(∆1) = D(∆). Por , nga
L2 D(∆1) ≤ D(∆o) = 0, rrjedhimisht, D(∆) = 0.
(ii) Në qoftë se njëra apo të dy katetat e trekëndëshit ∆
janë më të mëdha se katetat e trekëndëshit ∆o, atëherë
duke zbatuar aksiomën IV1 dhe L3 ne mund të
ndërtojmë një trekëndësh kënddrejtë ∆’=∆PQB’ me
defect zero, katetat e të cilit janë më të mëdha apo të
barabarta me katetat e trekëndëshit ∆, keshtu që nga rasti
(i) rrjedh D(∆) ≤ D(∆’) = 0  D(∆) = 0. □
Vërtetimi i teoremës të dytë të Legendre-Saccherit
Me qenë se çdo trekëndësh ka të paktën dy kënde të
drejtë ekziston kulmi, këmbëza e lartësisë të të cilit në
brinjën përballë ndodhet ndërmjet dy kulmeve të tjerë.
Le të jenë Ao dhe A kulmet e trekëndëshave ∆o dhe ∆
ashtu që këmbëzat Do dhe D me rradhë të tyre ne brinjët
përballë ndodhen ndërmjet dy kulmeve të tjerë. Me pikat
Do dhe D, në trekëndëshat ∆o dhe ∆ fitohen nga dy
trekëndësha kënddrejtë ∆o1, ∆o2 dhe ∆1, ∆2 ashtu që nga
L1 kemi
D(∆o)=D(∆o1) + D(∆o2) dhe D(∆) = D(∆1) + D(∆2).
Me qenë se D(∆o) = 0  D(∆oi) = 0, i=1,2. Pasi që
ekziston trekëndëshi kënddrejtë ∆o1 me defect zero, nga
L4, defekti i çdo trekëndëshi kënddrejt do të jetë i
barabartë me zero. Prej aty, D(∆) = D(∆1) + D(∆2) = 0 .

P.6. (Teorema e tretë e Legendre-Saccherit)
Në qoftë se ekziston treknëshi ∆o, S(∆o) < 2d, atëherë
S(∆) < 2d, për ∆.
114 I Hyrje

Vërtetim. Vërtet, për çdo trekëndësh ∆, S(∆) < 2d, se


përndryshe, sikur të ekzistonte ndonjë trekëndësh ∆’,
S(∆’) = 2d, atëherë nga P5, në çdo trekëndësh, pra edhe
ne ∆o, shuma e këndeve do të ishte e barabartë me 2d, që
është në kundërshtim me supozimin e bërë. □

P.7. (Prerja e drejtëzes me rrethin) Drejtëza s e


rrafshit të një rrethi k(O,r), incidente me një pikë të
brendshme të tij, e pret rrethin në dy pika.
Vërtetim.

VI GJEOMETRIA EUKLIDIANE

6.1. Aksioma e paraleleve dhe rrjedhimet e para


Le të jetë A pikë joincidente me drejtëzën e dhënë a. Në
gjeometrinë absolute, më saktësisht, nga tre grupet e para të
aksiomave, vërtetuam se ekziston drejteza b në rrafshin (Aa)
incidente me A dhe joprerëse me a. Fakti se drejtëza e tillë b është
e vetme apo të tilla ka më shumë nuk është rrjedhim i gjeometrisë
absolute. Prandaj po e marrim si aksiomë të grupit të pestë
pohimin:
V1 (Aksioma e Playferit) Për çdo drejtëz a dhe çdo pikë A  a
ekziston jo më shumë se një drejtëz b  A joprerëse me a.

Le të jetë SE ={I1-7, II1-4, III1-5, IV1-2, V}. Bashkësia GE=<SE> e të


gjitha rrjedhimeve të sistemit aksiomatik SE quhet Gjeometri
euklidiane. SE quhet sistem aksiomatik i gjeometrisë euklidiane.
Hapësira, pikat e së cilës i plotësojnë kërkesat e sitemit SE quhet
hapësire euklidiane, kurse rrafshi pikat e të cilit i plotësojne
kërkesat e sitemit aksiomatik SE quhet rrafsh euklidian.

Përkufizimi 1. Për drejtëzën a themi se është paralele me


drejtëzën b dhe simbolikisht i shënojmë a║b, në qoftë se a, b i
takojnë një rrafshi dhe a  b ose a  b = .
I Hyrje 115

Teorema 1. Paralelizmi i drejtëzave në bashkësinë e drejtëzave


koplanare është relacion ekuivalence.
Vërtetim. Vetia refleksive dhe simetrike rrjedhin menjëherë nga
D1. Le të jetë a║b & b║c. Në qoftë se a  c, nga D1, a║c, prandaj
le të jetë a  c. Drejtëza a është paralele me c-në, se sikur a  c
={P}, nëpër pikën P do të kalonin dy drjetëza a dhe c paralele me
drejtezën b, që është në kundërshitm me kasiomën V. □

Teorema 2. Në qoftë se drejtëza c e pret njërën prej dy drejtëzave


paralele, atëherë ajo e pret edhe tjetrën. Në veçanti, në qoftë se c
është normale në njërën prej dy drejtëzave paralele, ajo është
normale edhe në drejtëzen tjetër.
Vërtetim. Le të jenë a║b dhe a  c={P}. Drejtëza c e pret edhe
b-në, se sikur të mos e priste atë, nëpër pikën P do të kalonin dy
drejtëza a, c paralele me drejtëzën b.
Le të jetë tani a  c dhe b  c ={B}. Po të jetë b’ B dhe b’ c,
atëherë edhe b’ do të jetë paralele me a dhe nga aksioma V do të
kemi b’ b, pra edhe b është normale në c.□
Teorema 3. Në qoftë se njëra prej dy drejtëzave paralele a, b e
pret rrafshin , atëherë atë e pret edhe drejtëza tjetër.
Vërtetim. Shënojmë me  = r(a,b). Në qoftë se  e pret a-në,
atëherë ,  priten. Le të jetë p =   . Drejtëza p e pret drejtëzën
a, rrjedhimisht p e pret edhe b-në, pra  e pret edhe drejtëzën b. □

Që të vërtetojme se paralelizmi i drejtëzave ne hapësirë është


relacion ekuivalence do të vërtetojmë këtë pohim:

Teorema 4. Le të jenë a, b, c tri drejtëza të ndryshme në hapësirë,


çdo dy prej të cilave janë koplanare.
(i) Në qoftë se dy prej tyre priten, edhe drejtëza e tretë kalon
nëpër atë pikë.
(ii) Në qoftë se dy prej tyre janë paralele, edhe drejtëza e tretë
është paralele me to.
Vërtetim. Shënojmë me   rr (b, c),   rr (c, a),   rr (a, b) .
(i) Supozojmë se a  b ={P}. Me qenë se P  a   dhe P
b  , atëherë P     = c.
(ii) Në qoftë se a║b, atëherë edhe c është paralele me secilën
prej tyre, se sikur c ta priste njërën prej tyre, psh. a-në,
atëherë, nga (i), edhe b-ja do të kalonte nëpër atë pikë, që
s’është e mundur sepse a║b. □
116 I Hyrje

Teorema 5. Paralelizmi në bashkësinë e drejtëzave në hapësirë


është relacion ekuivalence.
Vërtetim. Vetia refleksive dhe simetrike rrjedhin nga
përkufizimi1. Le të jetë a║b & b║c dhe le të jetë Cc (cila do).
Shënojmë me
  rr (a, C ) dhe   rr (b, c) . Nga se ,   C     = c’.
Tani, a, b, c’ janë tri drejtëza të ndryshme, çdo dy prej të cilave i
takojnë një rrafshi dhe me qenë se a║b, atëherë edhe c’║a, b. Me
që c, c’ C dhe c, c’║b, nga aksioma V rrjedh se c’ c, pra c║a.

Çdo klasë të ekuivalencës të paralelizmit të drejtëzave në hapësirë


do t’a quajmë drejtim. Drejtimin të përcaktuar nga drejtëza p do ta
shënojmë (p).
Le të jetë  cili do një rrafsh dhe p drejtëz që e pret rrafshin
. Nga aksioma V, nëpër çdo pikë A kalon vetëm një drejtëz s
(p). Me që p e pret , atëherë edhe s e pret , p.sh. në pikën A’. Me
që për pikën A, pika A’ përcaktohet në mënyrë të vetme,
pasqyrimin
: P   , A A’ e quajmë projektim paralel të hapësirës
euklidiane P në rrafshin  në drejtimin (p). Pikën A’= (A) e
quajmë projeksion paralel të pikës A në rrafshin . Po të jetë F 
P një figurë, atëherë F '  {M ' M  F} quhet projeksion parallel
i figurës F në rrafshin .

Në gjeometrinë absolute e përkufizuam projeksionin normal të


hapësirës P në një rrafsh si pasqyrim që çdo pike A ia shoqëron
këmbëzën e normales të asaj pike në rrafshin . Me qenë se në
gjeometrinë euklidiane, dy drejtëza normale në një rrafsh janë
drejtëza paralele, atëherë projektimi normal është rast i veçantë i
projektimit parallel.

Përkufizimi 2. Për drejtëzën a themi se është paralele me rrafshin


 dhe e shenojme a║, në qoftë se a është paralele me projeksionin
normal a’ të saj në rrafshin .

Teorema 6. Në qoftë se a║, atëherë


(i) a   ose a   = ;
(ii) a është paralele me pambarimisht shumë drejtëza të .
I Hyrje 117

Vërtetim. (i) Me që a║  a║a’, ku a’ është projeksioni normal


i a në . Por atëherë a  a’ ose a  a’ = , rrjedhimisht a  
ose a   = .
(ii) Për çdo drejtëz b   që është paralele me projeksionin a’ të
drejtëzës a në , nga vetia transitive e paralelizmit të drejtëzave
kemi a║b. □

Teorema 7. Në qoftë se a është paralele me një drejtëz b të ,


atëherë a║.
Vërtetim. Le të jetë a’ projeksioni normal i a në . Me që a, b, a’
janë tri drejtëza, çdo dy prej të cilave janë koplanare dhe a║b,
atëherë edhe a’║a, b rrjedhimisht a║. □
Teorema 8. Në qoftë se A a, atëherë ekzistojnë pambarimisht
shumë rrafshe incidentë me A dhe paralel me a.
Vërtetim. Le të jetë b  A, b║a. Çdo rrafsh  incident me drejtëzën
b, nga teorema 6, është paralel me a. □

Përkufizimi 3. Për dy rrafshe ,  themi se janë paralelë dhe i


shënojmë ║, në qoftë se  apo    = .
Teorema 9. Në qoftë se A,  jo më shumë se një rrafsh   A
dhe ║.
Vërtetim. Supozojmë se ekzistojnë dy rrafshe , ’  A dhe ,
’║. Shënojmë me p =   ’. Le të jetë A’ projeksioni normal
i A në  dhe  cili do një rrafsh që e përmban d(AA’). Le të jenë
   = b, ’  = b’,    = a.
Drejtëza a është projeksioni normal i b, b’ në  dhe b  b’. Me
qenë se , ’║ rrjedh që b, b’║a dhe b, b’  A, që është në
kundërshtim me V. □

Nga teorema 9 rrjedh vetia transitive e paralelizmit të rrafsheve.


Vërtet, le të jenë ║ & ║ . Po të jetë   , atëherë    = ,
se sikur    = {P}, nëpër P do të kalonin dy rrafshe , ║.

Kështu kemi vërtetuar pohimin:

Teorema 10. Paralelizmi ne bashkësinë e rrafsheve është relacion


ekuivalence. □

Vërtetojmë tani ekzistencën e rrafshit A, ║.


118 I Hyrje

Teorema 11. Në qoftë se A, të gjitha drejtëzat incidente me A


dhe paralele me  i takojnë një rrafshi që është parallel me rrafshin
.
Vërtetim. Le të jenë a,bA dhe a, b║. Shënojmë me =rr(a,b).
Rrafshet ,  janë paralelë, se sikur    = p, atëherë nga a, b║
 a, b║p. Në rrafshin , nëpër pikën A do të kalonin dy drejtëza
a, b paralele me p. Le të jetë c cila do një drejtëz nëpër A, paralele
me . Atëherë edhe rrafshi  = r(a,c) ║, rrjedhimisht  = . Pra,
të gjitha drejtëzat incidente me pikën A dhe paralele me rrafshin 
i takojnë rrafshit  që është parallel me rrafshin. □

Le të jenë a, b drejtëza aplanare dhe A, B pika përkatësisht nga a,


b. Shënojmë me b’, a’ drejtëzët, përkatësisht nëpër A, B paralele
me b, a. Nga teorema 11, =rr(a,b’) dhe =rr(a’,b) janë rrafshe
paralele. Kështu vërtetuam se drejtëzat aplanare ndodhen në
rrafshe paralele.

Do të vërtetojmë tani se

Teorema 12. Drejtëzat aplanare kanë një dhe vetëm një normale të
përbashkët.
Vërtetim. Le të jenë a, b drejtëza aplanare. Ekziston rrafshi   a
dhe ║b. Le të jetë   a,   . Çdo drejtëz c  , c║b e pret
drejtëzën a, se përndryshe, sikur c║a, do të kishim a║b, që nuk
mundet. Me që c e pret a  c e pret rrafshin , rrjedhimisht edhe
drejtëza b e pret rrafshin  p.sh. në ndonjë pikë B. Le të jetë
A=prn(B) në  dhe =rr(b,A). Po të jetë b’=   α, me që    ,
b’ është projeksioni normal i b në ,rrjedhimisht b║b’. Por d(AB)
që është normale në  është normale edhe në drejtëzat a, b’dhe b.
Pra,  n=d(AB) normale në drejtëzat aplanare a, b. Po të ekzistonte
edhe nodnjë normale tjetër n’ n e drejtëzave a, b, drejtëza n’ nuk
e përmban asnjërën prej pikave A, B , se përndryshe do të fitonom
trekëndësh me dy kënde të drejtë. Në qoftë se n’ i pret drejtëzat a,
b, përkatesisht në A’, B’, atëherë drejtëzat a, b do të ishin
koplanare. □
I Hyrje 119

6.2. Pohime ekuivalente me postulatin e peste


aksiomën e paraleleve

Të rikujtojmë se për dy pohime P1 , P2 themi se janë ekuivalentë në lidhje


me një system aksiomatik S, në qoftë se nga S , P1  P2 dhe nga
S , P2  P1 .
Do të vërtetojmë tani disa pohime që janë ekuivalente me aksiomën e
paraleleve në lidhje me sistemin aksiomatik të gjeometrisë absolute.

Teorema 1. Postulati i pestë i Euklidit (VE ) është ekuivalent me aksiomën


e paraleleve (V ) .
Vërtetim. S A ,V  VE : Le të jenë a , b dy drejtëza paralele të prera nga
një e tretë c dhe që formojnë me të kënde të brendshëm të një anëshëm
 ,  ,     2d (*) . Vërtetojmë se a, b priten. Shënojmë me A, B
prerjet e drejtëzës c me drejtëzat a, b. Le të jetë b ' drejtëza e vetme nëpër
B, paralele me a. Për këndet e kundërt  ,  ' që i formojnë drejtëzat a , b '
me transverzalen c kemi    '  2d , kështu nga (*) rrjedh se b  b ' , e
me që b ' është e vetme, drejtëza b e pret drejtëzën a.
S A ,VE  V : Le të jetë A një pikë jo incidente me drejtzen a. Në
gjeometrinë absolute egziston drejtëza b në rrafshin (Aa) incidente me A
dhe joprerëse me a. Vërtetojmë se ajo është e vetme. Po të jetë b '  A cila
do një drejtëz tjetër e këtij rrafshi, b '  b , atëherë cila do një transverzale
c  A formon kënde të kundërta me drejtëzat a, b ' , shuma e të cilëve është
më e vogël se 2d, prandaj nga VE , b ' e pret drejtëzën a. □

Le të jenë  ,  kënde përgjegjes që i formojnë dy drejtëza a , b të prera


me një transverzale c. Pohimi: Nëse     a është paralele me b
është pohim i gjeometrisë absolute. Pohimi i anasjelltë me të nuk është
rrjedhim i gjeometrisë absolute, por është pohim ekuivalent me V.

Teorema 2. Pohimi P: Dy drejtëza paralele a , b të prera me një


transverzale c formojnë kënde përgjegjëse (shëndruese)  ,  të barabarta,
është ekuivalent me aksiomën e paraleleve.
120 I Hyrje

Vërtetim. S A ,V  P Le të jenë A, B prerjet e a , b me drejtëzën c. Në


qoftë se (   ) , egziston drejtëza a '  A që me c e formon këndin  '
kongruent me këndin  . Nga gjeometra absolute rrjedh se a '  b, pra
nëpër pikën A ' do të kalonin dy drejtëza paralele me b, që është në
kundërshtim me aksiomën V.
S A , P  V Drjetëza a  A , ab është e vetme, se po të egzistonte edhe
një drejtëz tjetër a '  A ,  '  b, atëherë këndet përgjegjës  '   ,
rrjedhimisht    ' , që do të ishte në kundërshtim me aksiomën III4. □

Teorema 3. Pohimi P: Shuma e këndeve në çdo trekëndësh është e


barabartë me 2d, është ekuivalent me V.
S A ,V  P Le të jetë ABC cili do një trekëndësh, kurse B ', C ' pika në
anë të ndryshme të d ( AB ) ashtu që BAB '  B, CAC '  C . Nga kjo
rezulton që d ( AB '), d ( AC ')  d ( BC ) dhe nga aksioma V,
d ( AB ')  d ( AC ') . Me qenë se A, B ', C ' janë pika kolineare kemi
A  B  C  2d .
S A , P  V Le të jetë A  a dhe A ' projeksioni normal i A në a. Le të
jetë b  A, b  d ( AA ') . Atëherë b  a   . Vërtetojmë se çdo drejtëz
tjetër c  A , c  b , e pret drejtëzën a. Nga njëra anë e pikës A ' në a i
caktojmë pikat A1 , A2 , , An , ashtu që B ( A ', A1 , A2 , , An ) dhe
[ A ' A1 ]  [ AA '] , [ A1 A2 ]  [ AA1 ], ,[ An1 An ]  [ AAn1 ] . AA ' A1 është
trekëndësh kënddrejtë dybrinjënjishëm, prandaj, nga pohimi P, kemi se
d d d
AA1 A '  . Ngjashëm tregojmë se AA2 A1  2 , , An 1 An A  n .
2 2 2
Le të jenë B  b, C  c nga ajo anë e d ( AA ') nga janë edhe pikat Ai .
Këndet AAn A 'dhe An AB janë kongruentë si kënde komplementarë me
të njëjtin kënd A ' AAn . Nga ana tjetër, për këndet BAC dhe d , n  N
d
, ashtu që  BAC . Dhe në fund, nga
2n
d
An AB  AAn A '  An 1 An A  n  BAC rrjedh se drejtëza c ndodhet
2
në A ' AAn , prandaj c e pret [ A ' An ] , gjegjësisht drejtëzën a. □
Shënim. Në qoftë se egziston një trekëndësh , S ( )  2d , nga Teorema
e dytë e Legendre-Saccherit shuma e këndeve në çdo trekëndësh është e
I Hyrje 121

barabartë me 2d, prandaj edhe pohimi: Egziston trekëndësh, shuma e


këndeve të të cilit është e barabartë me 2d është pohim ekuivalent me
aksiomën e paraleleve. Në gjuhën e defektit këtë mund t’a formulojmë:
Pohimi: Egziston trekëndësh me defekt zero është ekuivalent me aksiomën
e paraleleve.

Nga Teorema 3 mund të vërtetohen edhe pohimet:

Teorema 4. Pohimi P: Egziston katërkëndëshi i thjeshtë, shuma e këndeve


të të cilit është e barabartë me 4d është pohim ekuivalent me V.
Vërtetim. S A ,V  P Shuma e këndeve të katërkëndëshit ABCD është
e barabartë me shumën e këndeve të trekëndëshave ABC dhe ACD dhe
me qenë se shuma e këndeve të secilit prej tyre është 2d, atëherë edhe
S ( ABCD)  2d .
S A , P  V Në qoftë se S ( ABCD)  2d , atëherë
S (ABC )  S (ACD)  S ( ABCD)  4d dhe me qenë se S (ABC )  2d
S (ACD)  2d kemi S (ABC )  S (ACD)  2d që është ekuivalent
meV.
Rrjedhime të teoremës 4 janë:
Teorema 5. Hipoteza e këndit të drejtë të katërkëndëshit të Saccherit është
pohim ekuivalent me aksiomën V. □

Teorema 6. . Hipoteza e këndit të drejtë të katërkëndëshit të Lambertit


është pohim ekuivalent me aksiomën V. □

Teorema 7. Pohimi: Egziston n-këndshi i thjeshtë, shuma e këndeve të të


cilit është e barabartë me (n  2)2d është ekuivalent me aksiomën V.
Vërtetim. S A ,V  P Nga V rrjedh se S ()  2d për çdo trkëndësh 
. Me diagonalet të një kulmi n-këndëshi ndahet në n  2 trekëndësha dhe
me qenë se shuma S n e këndeve të n-këndëshit është e barabartë me
shumën e këndeve të n-2 trekëndëshave, atëherë Sn  (n  2)2d .
S A , P  V Nga (n  2)2d  Sn  S (1 )   S ( n1 ) dhe S (i )  2d
rrjedh S ()  2d , që është pohim ekuivalent me V. □

Teorema 8. Pohimi: Nëpër çdo pikë të brendshme të këndit të ngushtë


egziston drejtëza që i prêt të dy krahët e këndit; është ekuivalent me V.
122 I Hyrje

Vërtetim. S A ,V  P Le të jetë M një pikë në këndin e ngushtë ab


me kulm O. Nga V, nëpër pikën M kalon vetëm nga një drejtëz a1 , b1
paralele, përkatësisht me drejtëzat a , b . Çdo drejtëz tjetër x  M në këndet
kryqëzues a1b1 që nuk e përmbajnë kulmin O i pret krahët e këndit ab
.
S A , P  V Në vend të V vërtetojmë pohimin S ()  2d , për çdo  .
Supozojmë të kundërtën se 0  ABC , A  d , S (0 )  2d . Shënojmë
me u  2d  S (0 ) . Atëherë në çdo  , S ()  2d .
E caktojmë pikën D në gjysmërrafshin BCA , ashtu që DCB  ABC .
Nga pohimi P, egziston drejtëza p1  D që i pret të dy krahët e këndit të
ngushtë BAC në pikat B1 , C1 . Te trekëndëshi 1  AB1C1 , S (1 )  2d
.
Shënojmë me u1  2d  S (1 ) defektin e 1 .
Vërejmë se
u1  2d  S (1 )  2d  6d  S ( 0 )  S ( ' )  S ( '' )  S ( ''' ) =

2d  S ( 0 )  2d  S ( ' )  2d  S ( '' )  2d  S ( ''' )
 u  u  u ''  u '''  2u .
Kështu ndërtuam trekëndëshin 1 , defekti u1 i të cilit është më i madh se
2u. \ngjashëm konstruktojmë trekëndëshin 2  AB2C2 , S ( 2 )  2d
me defekt u2  2d  S ( 2 )  2u1  22 u . Duke e përsëritur këtë veprim
sipas dëshirës, e kostruktojmë trekëndëshin  n  ABnCn , S ( n )  2d me
defekt
un  2d  S ( n )  2n u . Por, për këndet u dhe 2d, egziston n  N , ashtu
që 2 n u  2d , pra egziton trekëndësh  n  ABnCn me defekt
un  2n u  2d , që nuk është e mundur, sepse defekti i trkëndëshit është
më i vogël se 2d. Pra, supozimi se egziston trekëndëshi
0  ABC, S (0 )  2d është kontradiktor, rrjedhimisht S ()  2d ,
për çdo trkëndësh  , e ky pohim është ekuivalent me aksiomën e
paraleleve V. □
Ngjashëm si teorema 8 vërtetohet
Teorema 9. Pohimi P: Të gjitha normalet në njërin krah të këndit të nguhtë
e presin krahun tjetër; është pohim ekuivalent me aksiomën V. □
I Hyrje 123

Teorema 10. Pohimi: Simetralet e dy brinjëve të trekëndëshit priten në një


pikë, është ekuivalent me aksiomën V.
Vërtetim. S A ,V  P Le të jenë s, s ' me rradhë simetralet e brinjëve
AB, BC të ABC . Nga Ga , drejtëzat s, s ' ose priten ose nuk priten. Sikur
s  s '   , atëherë s s ' dhe pasi që d ( AB)  s  d ( AB)  s ' . Kështu
kemi dy drejtëza d ( AB), d ( BC ) incidente me B dhe normale në s ' , që nuk
mundet. Pra simetralet e dy brinjëve të trekëndëshit priten, a nga Ga edhe
simetralja e brinjës së tretë kalon nëpër atë pikë.
S A , P  V Le të jetë P  a . Në Ga egziston b  P, b  a  
.Vërtetojmë se çdo drejtëz tjetër c  P , c  b e pret drejtëzën a. Le të jetë
n  P, n  a dhe n  a  {M } . Le të jetë P  n , ( P  A  M ) . Shënojmë
me B  ( A) a dhe C  ( B) c . Me qenë se c  n , A, B, C janë pika
jokolineare. Drejtëzat a , c janë simetrale të ABC dhe nga pohimi P ato
priten.□

Nga Teorema 10 rrjedh se edhe pohimi: Rreth çdo trekëndëshi mund të


jashtëshkruhet rrethi, është ekuivalent me aksiomën V.

Përkufizim. Bashkësia e të gjitha pikave në rrafsh në largësi të barabartë


prej një drejtëze p të atij rrafshi quhet ekuidistantë e p.

Teorema 11. Pohimi: Ekuidistanca e drejtëzës është drejtëz, është pohim


ekuivalent me aksiomën V.
Vërtetim. S A ,V  P Le të jenë A, B, C pika të e kuidistancës e të
drejtëzës p dhe A ', B ', C ' me rradhë projeksionet normale të tyre në p.
AA ' B ' B dhe BB ' C ' C janë katërkëndësha të Saccherit, rrjedhimisht
këndet te kulmet B të tyre janë të drejtë, a prej kësaj rezulton se A, B, C
janë pika kolineare.
S A , P  V Nëse ekuidistanca e p ëahtë drejtëz, atëherë katërkëndëshi
MNM ' N ' i përcaktuar nga meset e bazave të katërkëndëshave të Sakerit
AA ' B ' B dhe BB ' C ' C është kaërkëndësh kënddrejtë, a egzistenca e një
katërkëndëshi të tillë është ekuivalente me aksiomën V. □

6.3. Rrjedhime të tjera të aksiomës V


124 I Hyrje

VII TRANSORMIMET E NGJASHMËRISË


VII TRANSORMIMET E NGJASHMËRISË

7.1. HOMOTETIA

Përkufizimi 1. Le të jetë O pikë e fiksuarnë E2 dhe kR\{0}.


Pasqyrimi  O,k : E2  E2 , ( X )  O,k  X '  OX '  kOX
quhet homoteti apo dillatacion me qendër O dhe koeficient k.
Nga përkufizimi rrjedh se homotetia njëvlerësisht përcaktohet nga qendra
dhe koeficienti i saj.
Nëse O,k : O  O '  OO '  kOO  o  O '  O , që do të thotë se
qendra e homotetisë është pikë fikse. Për k=1,  O ,1  id E2 , kurse për
k  1 ,  O ,1   O . Pra, identiteti dhe simetria qendrore janë homoteti.
Për k>0, pikat X , X ' ndodhen nga njëra anë e O, kurse për k<0 pikat
X , X ' ndodhen në anë të ndryshme të pikës O.

Teorema 1. Homotetia është pasqyrim bijektiv.


Vërtetim. Le të jenë A,B pika arbitrare dhe  O ,k : A, B  A ', B '. Ateherë
OA '  kOA , OB '  kOB .
Nga A ' B '  A ' O  OB '  k AO  kOB  k ( AO  OB )  AB
rrjedh se po të jenë A, B pika të ndryshme, atëherë edhe pikat A ', B ' do të
jenë të ndryshme.
1
Për cilën do pikë A ' e caktojme A  ( A ')  1 . Atëherë OA  OA ' 
O,
k
k
OA '  kOA  O,k : A  A '. □
I Hyrje 125

Me qenë se  O , k është bijeksion,   O1,k . Nëse  O ,k : A  A ' , atëherë


1
 O1,k : A '  A . Nga  O ,k : A  A '  OA '  kOA  OA  OA ' 
k
 1 : A '  A . Prej kësaj rrjedh se  O1,k =  1 .
O, O,
k k

Teorema 2.  O ,k është transformim kolinear.


Vërtetim. Le të jenë A,B,C pika kolineare dhe  O ,k : A,B,C A ', B ', C ' . Për
vektorët kolinearë AB, AC ,   R, AC   AB . Por atëherë nga
k AC   k AB  A ' C '   A ' B '  A ' C ', A ' B ' janë kolinearë, që do të
thotë se A ', B ', C ' janë pika kolineare.□
Shënim. a) Nëse janë dhënë dy pika A,A’ a egziston homotetia që njërën
pikë e pasqyron në pikën tjetër?
Në qoftë se  O ,k : A  A ' , atëherë OA '  kOA  O, A, A ' janë pika të
ndryshme kolineare dhe k  OA ' OA . Pra homotetia përcaktohet nga
qendra e saj O dhe një çift pikash koresponduese A, A’ ashtu që O, A, A’
janë kolineare. Çdo pike O e drejtëzës d ( AA ') është qendër e një
homotetie që pikën A e pasqyron në A ' .

Teorema 3. Le të jetë p cila do një drejtëz.


 p, O  p
p  0,k   .
 p, O  p

Vërtetim. Le të jetë O  p . Me qenë se për çdo A  p, pikat O, A dhe


A '  A 0,k janë kolineare dhe  0,k : O, A  O, A ' rrjedh se p '  p .
Anasjelltas,  0,k : p  p  p  O .
nëse Vërtet, për çdo
A  p, A '  p '  p dhe O  d ( AA ')  p .
Nëse O  p , për çdo A, B  p , dhe  0,k : A, B  A ', B ' , nga A ' B '  k AB
rrjedh se d ( A, B) d ( A ', B ') , prandaj p p ' . □

b) Në qoftë se A, B, A ', B ' janë katër pika të dhëna, a egziston homotetia


O ,k : A, B  A ', B '?
Në qoftë se një homoteti e tillë egziston, ateherë A ' B '  k AB 
d ( AB) d ( A ' B ') . Për k  1 , d ( AA ')  d ( BB ')  {O} , kurse për k=1,
126 I Hyrje

A ' B '  AB  katërkëndëshi ABB ' A ' është parallelogram, rrjedhimisht


AA '  BB '  x.
Pra, në qoftë se AB A ' B ' dhe AB  A ' B ',  O,k : A, B  A ', B ' , ku
A' B '
{O}  d ( AA ')  d ( BB '), dhe k  . Në qoftë se A ' B '  AB ,
AB
 x : A, B  A ', B '. B’
B B B’

o o o
O A A’ Oo o o A A’
A A’

Që të caktohet p ' gjejmë A ' për një A  p dhe mandej nëpër A '
konstruktojmë drejtëzën p ' paralele me p .

Detyrë. Është dhënë homotetia  me qendër O dhe një çift pikash


koresponduese A, A ' . Konstruktoni koresponduesen e ciles do një pike M.
Zgjidhja. Do dallojmë dy raste: (i) M  d (OA) ; (ii) M  d (OA) .
(i) {M '}  d (OA)  p , ku p është drejtëz nëpër A ' paralele me d ( AM ) .
(ii) Konstruktojmë së pari koresponduesen B ' të një pike B që nuk i takon
drejtëzës d (OA) mandej e gjejmë M ' sin ë (i). □

Teorema 4. Fëtyra homotetike e rrethit është rreth.

Vërtetim. Le të jetë l ( S , r ) rreth me qendër S dhe rreze r. Për çdo pikë


M l , nga  0,k : M , S  M ', S ' kemi M ' S '  kMS , gjegjësisht
M ' S '  k MS  k r . Kjo do të thotë se pika M ' i takon rrethit
l '(S ', k r ) . Anasjelltas, për çdo pikë P '  l '(S ', k r ) , po të shënojmë
P  ( P ') O, 1 , atëherë nga
k

OP  1 OP ' dhe OP  1 OP '  1 k r  r


k k k
kemi që P  l ( S , r ) . Ndërsa, nga OP '  kOP kemi P '  ( P)  O ,k . Pra,
çdo pikë e rrethit l '(S ', k r ) është fëtyrë e ndonjë pike të rrethit l ( S , r ) .
I Hyrje 127

Kështu kemi vërtetuar se rrethi l '(S ', k r ) është fëtyra homotetike e rrethit
l ( S , r ) .□

Detyrë. Janë dhënë rrathët l1 (S1 , r1 ) dhe l2 (S2 , r2 ) . A ekziston homotetia


që l1  l2 ?
Zgjidhja. a) Le të jenë l1 (S1 , r1 ) dhe l2 (S2 , r2 ) rrathë koncentrikë ( S1  S2
). Atëherë  S1 ,r2 r1 : l1 (S1 , r1 )  l2 ( S2 , r2 ) , kurse
 S ,r
1 1 r2 : l2 ( S2 , r2 )  l1 ( S1 , r1 ) .
b) Le të jetë S1  S2 dhe {P, Q}  l2  d (S1 , S2 ) . Për A  l2 , A  P, Q
shënojmë me A1 A2 atë diameter të l1 që është parallel me [ AS2 ] .
Shënojmë {O1}  d ( S1S2 )  d ( AA1 ) , {O2 }  d ( S1S2 )  d ( AA2 ) . Pika O2
gjithmonë egziston, ndërsa O1 vetëm për r1  r2 . Homotetitë
 O1 ,r2 r1 dhe  O2 , r2 r1 : l1 (S1 , r1 )  l2 ( S2 , r2 ) . Tangjentat e jashtme të
rrathëve l1 (S1 , r1 ) dhe l2 (S2 , r2 ) , nëse egzistojnë, kalojnë nëpër pikën O1 ,
ndërsa tangjentat e brendshme ,nëse egzistojnë, kalojnë nëpër O2 .
Shënim: Në qoftë se r1  r2 , atëherë d ( S1S2 ) d ( AA1 ) dhe O1 ,r2 r1   v , ku
v  S1S2 .
Përkufizim Dy figura F , F ' quhen homotetike, në qoftë se egziston
homotetia  : F  F ' .
Nga detyra e mësipërme rrjedh se çdo dy rrathë janë homotetikë. Pika O1
quhet quhet qendër e homotetisë direkte, kurse O2 qendër e homotetisë
indirekte të rrathëve l1 (S1 , r1 ) dhe l2 (S2 , r2 ) .
Teorema 5. Homotetia e ruan madhësinë e këndeve.
Vërtetim. Le të jetë   ab një kënd me kulm në P dhe
 : ab  a ' b ' , atëherë ab  a ' b ' , si kënde me krahë paralelë. □

Teorema 6. Le të jenë  i   O ,ki , (i  1, 2) homoteti me qendër O. Atëherë


1   2   O ,k k është homoteti me qendër O dhe koeficient k1k2 .
1 2

Vërtetim. Nëse 1 : A  A ',  2 : A '  A", atëherë


OA '  k1 OA, OA"  k2 OA '  k2k1 OA
rrjedhiisht, 1  2 , O ,k k : A  A " , për çdo pike A, prandaj 1  2  O ,k k . □
1 2 1 2
128 I Hyrje

Shënim. Me qenë se k1k2  k2 k1 , atëherë 1  2   2 1 .

Teorema 7. Bashkësia O  { O ,k | k  R  {0}} e të gjitha homotetive me


qendër të njëjtë është grup komutativ në lidhje me kompozimin e
pasqyrimive.
Vërtetim. Nga pohimi 6, prodhimi i dy homotetive me qendër O është
homoteti me qendër O. Për çdo  O ,k ,  O1,k   1 rrjedhimisht  O është
O,
k
nëngrup komutativ i grupit të bijeksioneve. □

Teorema 8. Le të jenë 1  O1 ,k1 ,  2  O2 ,k2 dy homoteti me qendra të


ndryshme. Atëherë
  O ,k k , nese k1k2  1

1  2   1 2 .

 , nese k1k2  1
Vërtetim. Le të jenë A, B  E2 dhe 1 : A, B  A1 , B1 , 2 : A1 , B1  A2 , B2
. Atëherë A1B1  k1 AB, A2 B2  k1 A1B1 , rrjedhimisht A2 B2  k1k2 AB .
a) Në qoftë se k1k2  1, vektorët paralelë AB, A2 B2 kanë gjatësi të
ndryshme, rrjedhimisht drejtëzat d ( AA2 ), d ( BB2 ) priten. Po të shënojmë
me {o}  d ( AA2 )  d ( BB2 ) , atëherë  O ,k1k2 : A, B  A2 , B2 , rrjedhimisht
1  2  O ,k k është homoteti.
1 2

Në këtë rast, d (O1O2 ) është invariante ndaj 1 ,  2 , prandaj edhe ndaj


produktit 1  2  O ,k1k2 , rrjedhimisht d (O1O2 )  O . Pra, O, O1 , O2 janë
pika kolineare. Vërtetojmë se
OO 1  k2
(O1O2O)  1  .
O2O k2 (k1  1)
Vërtet, nga O 1  2  O kemi O1  O21  OO2 ,1/ k2  P .
Nga O O1 ,k1  P  O1P  k1 O1O , ndërsa nga
O O2 ,1/ k2  P  O2 P  1/ k2 O2O .
Për pikat O1 , O2 , P kemi O1 P  O1O2  O2 P , apo
k1 O1O  O1O2  1/ k2 O2O . Me qenë se O1O2  O1O  OO2 kemi:
I Hyrje 129

k1 O1O  O1O  OO2  1/ k2 O2O


k 1 1  k2
 (k1  1)O1O  (1  1/ k2 )OO2  2 OO2  O2O ose
k2 k2
O1O 1  k2
 .
O2O k2 (k1  1)
b) Në qoftë se k1k2  1, atëherë A2 B2  k1k2 AB = AB  AA2  BB2  x ,
  x : A, B  A2 , B2 . Me qenë se 1 2 : A, B  A2 , B2  1  2   x . □

Nga 1  2   x  1   x  2 dhe  2  1 x , që do të thotë se ka vend


pohimi:
Teorema 9. Prodhimi i një homotetie dhe një translacioni dhe prodhimi
i një tanslacioni dhe një homotetie është homoteti. □

Teorema 10. Çdo homoteti indirekte është prodhim i një simetrie qendrore
dhe një homotetie direkte.
Vërtetim.  O ,k   O ,( 1)(  k )   O ,1  O , k   O  O , k , ku k  0 . □

7.5. Transformimet e ngashmërisë (ngjashmëria)


Izometritë janë klasë e veçantë e transformimeve më të
përgjithshme të hapësirës euklidiane En, (n=1,2,3), transformimeve të
ngjashmërisë.

D1. Le të jetë kR>0. Bijeksioni f=fk: En En që për çdo dy pika


A,B En e plotëson relacionin

AfBf
=k
AB

quhet ngjashmëri me koeficient k. Simbolikisht do ta shënojmë edhe me


fk për të thekësuar koeficientin e ngjashmërisë. Për k=1 kemi AfBf= AB,
gjegjësisht f ështe izometri. Pra, izometritë jane ngjashmëri me
koeficient 1.

Me N do të shënojmë bashkësinë e të gjitha ngjashmërive të


hapësirës En.
130 I Hyrje

P.1. (N,) ështe grup (jokomutativ).

Vërtetim.Le te jenë f1, f2 ngjashmëri, përkatësisht, me koeficent


k1, k2 dhe A,BN. Le të jetë f1: A,BA1,B1, f2: A1,B1A2,B2. Nga D.1 do
të kemi:

A1 B1 AB
 k1 , 2 2  k 2 .
AB A1 B1
A2 B2 A2 B2 A1 B1
Nga   k 2 k1 rrjedh se f1f2: A,BA2,B2 është
AB A1 B1 AB
ngjashmëri me koeficient k1k2. Nga ana tjetër, për fkN që A,BA’,B’
A' B ' AB 1
kemi =k  = …(). Me qenë se f –1: A’,B’A,B dhe
AB A' B ' k
vlen (*) kemi f = f 1 N. Kështu kemi vërtetuar se N është nëngrup i
–1

grupit te bijeksioneve të hapësirës En. Me që IN rrjedh se N eshte grup


jokomutativ.□
P.2. Ngashmëria ështe transformim kolinear dhe e ruan renditjen.
Vërtetim. Le të jetë f=fkN dhe A,B,C pika kolineare. Le të jetë
f: A, B, C  A' , B' , C '. Atëherë A' B'  kAB, B' C '  kBC, A' C '  AC. Po
të jetë ( A  B  C ), atëherë
AC  AB  BC , pra edhe kAC  kAB  kBC, apo A' C '  A' B' B' C '.
Relacioni i fundit tregon se A' , B ' , C ' janë kolineare dhe se ( A' B'C ' ) .□

P.3. Fëtyrët e segmenteve kongruentë janë segmente kongruentë.


Vërtetim. Le të jetë AB  CD dhe f k : A, B, C, D  A' , B' , C ' , D'. Ateherë
nga A' B'  kAB, C ' D'  kCD dhe AB  CD  A' B'  C ' D'.□
P.4. Homotetia  O , k është ngjashmëri me koeficient k .
  
Vërtetim. Në qoftë se  O , k : A, B  A' , B' , ateherë A' B'  kAB 
A' B' m( A' B' )
  k .□
AB m( AB)
P.5. Ngjashmëria që ka dy pika fikse është izometri.
Vërtetim. Le të jenë f : A, B  A, B dy pika fikse të ngjashmërisë f me
AB
koeficient k. Ateherë nga k   1  f është ngjashmëri me
AB
koeficient 1, rrjedhimisht f është izometri. □
I Hyrje 131

Prodhimi i një homotetije dhe një izometrije është ngjashmëri,


sepse secila prej tyre është ngjashmëri. Anasjelltas, do të vërtetojmë se
çdo ngjashmëri mund të paraqitet si prodhim i nje homotetije dhe një
izometrije. Për këtë qëllim do ti vërtetojmë disa pohime ndihmëse.

Lema1. Për çdo dy segmente AB, A' B ' ekziston ngjashmëria f , si


prodhim i një homotetije dhe një izometrije, që njërin segment e
pasqyron në tjetrin.
A' B '
Vërtetim. Shënojmë me k  . Ateherë homotetia
AB
AB1 A' B'
 O,k : A, B  A, B1 dhe k  , rrjedhimisht AB1  A' B' . Nga
AB AB
AB1  A' B'    I , : A, B1  A' , B'. Pra,  : A, B  A' , B'. □

Lema2.Për çdo dy segmente AB, A' B ' ekzistojnë dy dhe vetëm dy


ngjashmëri f1, f2 që AB  A' B' . Nëra prej tyre është prodhim i një
homotetije dhe një izometrije direkte, kurse tjetra prodhim i një
homotetije dhe një izometrije indirekte.
Vërtetim. Nga Lema1 f1=  N, f : A, B  A' , B' . Por ateherë edhe
f 2  f1 d ( A', B ') : A, B  A' , B' . Në qoftë se  është izometri direkte
(indirekte), ateherë  d ( A' B ') është izometri indirekte(direkte). Vërtetojmë
se çdo ngjashmëri tjetër f që AB  A' B' përputhet ose me f1 ose me f2.
1 1
Vërtet, nga f1 f : A' , B'  A' , B'  f 1 f është izometri dhe me qenë se
1 1 1
f 1 f e fikson gjysmëdrejtëzën A' B'  rrjedh se f 1 f =1 ose f 1 f =
 d ( A', B ') , pra f  f1 apo f  f1 d ( A', B ')  f 2 .□

Lema3. Në qoftë se ABC dhe A' B' C ' janë trekëndësha me brinjë
proporcionale, ekziston një dhe vetëm një ngjashmëri f që
A, B, C  A' , B' , C '
Vërtetim. Le të jetë A' B': AB  A' C': AC  B' C': BC  k . Nga Lema2,
ekzistojne vetëm dy ngjashmëri f1   dhe f 2  f1 d ( A'B ') , ku
   A,k , që pikat A,B i pasqyrojnë në A' , B ' . Prej tyre vetëm njëra pikën
C e pasqyron në gjysmërrafshin A' B ' C '  . Në qoftë se f1 është ajo
A' C1 A' C '
ngjashmëri, ateherë nga f1 : A, B, C  A' , B' , C1 kemi k 
AC AC
132 I Hyrje

B' C1 B' C '


, dhe k  , gjegjësisht A' C1  A' C' dhe B' C1  B' C' . Nga
BC BC
relacionet e fundit kemi C1  C ' , pra f1 : A, B, C  A' , B' , C' . □

P.6.(Teoremë) Çdo ngjashmëri në rrafsh mund të paraqitet si prodhim i


një izometrie dhe një homotetije.
Vërtetim. Le të jetë fN dhe A, B, C pika jokolineare. Nga se f është
kolinear, ateherë edhe Af , Bf , Cf jane jokolineare. Me qenë se për
trekëndëshat AfBfCf dhe ABC vlejnë relacionet
AfBf BfCf CfAf
  ( k ) , nga Lema3, !f’ N, e tillë që
AB BC CA
f ': Af , Bf , Cf  A, B, C . Por ateherë ff ': A, B, C  A, B, C ,
rrjedhimisht ff ' është izometri. Me që izometria ff ' fikson tri pika
jokolineare, ff ' =1, rrjedhimisht, f  ( f ' ) 1  (  ) 1   1  1 . Po të
jetë f= izometri, ateherë f  O ,1 . □

Si rrjedhim të teoremës të më sipërme mund ta formulojmë këtë

P.7. Çdo ngjashmëri në rrafsh ekziston dhe njëvleftësisht përcaktohet


me dhënjen e dy tresheve të pikave jokolineare proporcionale. □

Pasi që çdogjashmëri mund të paraqitet si prodhim i izometrise dhe


homotetisë dhe pasi që secila prej tyre e ruan kongruencën e këndeve,
atëherë ka vend ky pohim:

P.8. Ngjashmëria e ruan kongruencën e këndeve. □

D.2. Le të jetë fN dhe f=  . Ngjashmëria f quhet direkte(indirekte),


në qoftë se  është izometri direkte(indirekte).

P.9. Çdo ngjashmëri indirekte e ndryshme nga izometria mund të


paraqitet si prodhim i një simetrie boshtore dhe një homotetije, qendra e
së cilës i takon boshtit të simetrisë.
Pra, çdo f  N \ I , f  indirekte, f   s  O , O  s.
Vërtetim. Nga P6 f=  O, k . Me qenë se f nuk është izometri ,  k1.
Pasi që f është indirekte   është simetri boshtore apo simetri
I Hyrje 133

rrëshqitëse. a) Në qoftë se f   s  O ,k dhe Os, ateherë s '  O, s '  s .


Shënojmë me O1= s  s' .Nga se s  s'  s s '  s , prandaj
 s   s s   s ' s s '   s ' O1   s '  O1 , 1 . Tani kemi
f   s  O ,k   s '  O1 , 1  O ,k   s '  O ',k ' , ku O' d (O, O1 )  s' , k '  k  1 .
b) Le të jetë f   a , s  O ,k . Dimë se  a , s   a A   a  A, 1 , prandaj
f   a  A, 1  O ,k   a  O1 ,k1  (nga a))   s '  O ',k ' , s'  O' . □

Shënim. f  N \ I , f  indirekte, f   s  O ,k , O  s.
sf  s s  O ,k  s O ,k  s. Po kështu, Of=O.
Pra, çdo ngjashmëri indirekte në rrafsh, e ndryshme nga izometria, ka
drejtëz fikse dhe pikë fikse që i takon drejtezës fikse.

P.10. Për çdo homoteti  O , k dhe çdo izometri  vlen relacioni


  (O)   1  O ,k  .
Vërtetim. Le të jetë XE2 cila do pikë dhe X '  X O ,k rrjedhimisht)
OX '  kOX ……(1)
Le të jetë : O, X , X '  O1 , X1 , X1' , ateherë
OX  O1 X1 , OX '  O1 X1' …..(2)
Me qenë se O1 X 11 janë pika kolineare. Nga (1) dhe (2) kemi
O1 X 1'  kO1 X 1 …………(3)
Nga (3) rrjedh se O1 ,k : X1  X1' . Nga ana tjetër edhe

 1O,k : X1  X1' ……(4)


Nga (3) & (4) kemi O1 ,k   1O,k , ku O1  (O) . □

P.11.(Rrjedhim)  O ,k   O ,k  (O)  O.


Vërtetim. Nga  O ,k   O ,k kemi O,k   1O,k  (O) ,k  (O)  O .

Me qenë se ngjashmëria indirekte f, e ndryshme nga izometria paraqitet


f   s O,k , ku O  s, atwherw nga rrjedhimi i mwsipwrm kemi
f   s  O , k   O , k s .
134 I Hyrje

Paraqitja e ngjashmërisë indirekte fN \ I si prodhim i dy


transformimeve që komutojnë, prej të cilave njëra është simetri boshtore
tjetra homoteti quhet paraqitje kanonike e f. O, k, s quhen, me rradhë,
qendër, koeficient dhe bosht i ngjashmërisë f. Me qenë se f plotësisht
përcaktohet nga O,k,s atë e shënojmë edhe f = fO,k,s.
Ngjashmëria fO,k,s quhet simetri ekuiforme apo simetri homotetike.

Para se t’i shqyrtojmë ngjashmëritë direkte do ta vërtetojmë këtë


pohim:
P.12. Ngjashmëria f me koeficient k1 që ka drejtëz fikse s, ka edhe
pikë fikse S që i takon drejtëzës fikse s.
Vërtetim. Le të jetë A,Bs dhe f: A,BA’,B’. Ateherë A’,B’s dhe
A' B '
k.
AB
A' S A' B '
Me qenë se k1, ekziston Ss ashtu që k  …(1)
AS AB
SB A' S
Le të jetë B1  ( B)  S ,k . Atwherw SB1  kSB, gjegjwsisht 1  k 
SB AS
…(2)
AS A ' S
Nga (2) kemi  apo AS : ( AS  SB)  A ' S : ( A ' S  SB1 ) 
SB SB1
AS : AB  A ' S : A ' B1 apo A ' S : AS  A ' B1 : AB ….(3)
Nga (1) & (3)  A ' B '  A ' B1  B1  B ', prandaj (2) e shkruajmw si
AS A ' S
 ...(4)
SB SB '
A' S ' S ' B '
Shënojmë me S '  f ( S ) . Ateherë S '  s dhe  apo
AS SB
AS A ' S '
 …(5)
SB S ' B '
A' S A' S '
Nga (4) & (5) kemi  apo
SB ' S ' B '
A ' S : ( A ' S  SB ')  A ' S ' : ( A ' S ' S ' B ')  A ' S : A ' B '  A ' S ' : A ' B ' 
A ' S : A ' S '  S '  S . Pra S  S '  f ( S ) është pikë fikse.□

Shënim. Pika fikse S është e vetme se përndryshe f do të ishte izometri.


I Hyrje 135

P.13. Çdo ngjashmëri direkte f me koeficient k1 mund të paraqitet si


prodhim i një homotetije dhe një rrotacioni me të njejtën qendër.
Vërtetim. Me qenë se k1, eksiston çifti (A,B) i pikave, ashtu që drejtëzat
d(A,A’), d(B,B’) priten, ku f: A,BA’,B’.(Vërtet, sikur për çdo A,B
d ( A, A ') d ( B, B ') atëherë nga d ( A, A ')  f
 d ( A ', A '') dhe
d ( A, A ') d ( A ', A '') kemi d ( A, A ')  d ( A ', A '') që do të thotë se d ( A, A ')
do të jetë drejtëz fikse për f, prandaj nga P12 do të ekzistoje edhe pikë
fikse S në d(A,A’). Kështu ngjashmëria f do të kishte nga një pikë fikse në
çdo drejtëz të tufës paralele (d(A,A’)), rrjedhimisht, f do të ishte izometri,
që nuk mundet.)
Le të jetë {P}  d ( A, A ')  d ( B, B '). Rrathët 1  2  3  4 priten
apo takohen. Në rastin e parë le të jetë O pika tjetër prerëse e tyre. Nga
1  2  3  4 dhe 5  6  7  8 (fig…) rrjedh se
ABO dhe A " B " O janë trekëndësha me kënde kongruentë. Rrotacioni
 : gjd (OA)  gjd (OA '), A  A '', gjd (OB)  gjd (OB '), B  B ' dhe
O

d ( A ', B ') d ( A ", B '') . Homotetia  O ,k : A '', B ''  A ', B ' . Me qenë se
ngjashmëria direkte f njëvleftësisht përcaktohet nga dy çifte pikash
koresponduese ( A, A ')dhe( B, B ') dhe
A' B '
O ,  O ,k : A, B  A ', B ', ku k  dhe  = AOA' rrjedh që
AB
f  O ,  O ,k .
Po kështu bëhet vërtetimi kur rrathët k ( ABP) dhe k ( A ' B ' P) takohen,
shih fig…) □
Shënim. Me qenë se O : O  O  O ,  O ,k   O ,k O , . Pra, çdo
izometri direkte e ndryshme nga izometria mund të paraqitet si prodhim i
dy transformimeve që komutojnë, prej të cilave njëra është rrotacion
tjetra homoteti me të njëjtën qendër. Me qenë se f përcaktohet nga O, k,
 , ateherë atë e shënojmë si f=fO,k, . O, k,  i quajmë, me rradhë,
qendër, koeficient dhe kënd i ngjashmërisë f. Ngjashmëria f=fO,k, quhet
rrotacion ekuiform apo rrotacion homotetik.
Nga sa u tha më sipër ngjashmëritë në rrafsh mund të klasifikohen
në këtë mënyrë:
P.13. (Teorema per klasifikimin e ngjashmërive ne rrafsh).
Çdo ngjashmëri në rrafsh është një nga këto transformime:
a) Izometri;
b) homoteti;
136 I Hyrje

c) rrotacion homotetik (rrotacion ekuiform);


d) simetri homotetike (simetri ekuiforme). □

7.6. Ngjashmëria e figurave në E2

Me anë të grupit N të ngjashmërive ne rrafsh do të përkufizojmë një


relacion ekuivalence – relacionin e ngjashmërive të figurave.

D.1. Le të jenë F,F’ dy figura në E2.


Themi se F është e ngjashme me F’ dhe simbolikisht e shënojmë
FF’  fN, ashtu që f: FF’ .
Me qenë se N është grup rrjedh se  është relacion ekuivalence.
Kjo ekuivalence e ndan bashkësinë e figurave në pambarimisht shumë
klasë të ekuivalencës (klasët e trekëndshave të ngjashëm, klasët e
katërkëndshave të ngjashëm,…). Figurat e nje klase kanë formë të njëjtë.
Duhet theksuar se kushtet me të cilat konstatohet se dy figura kanë formë
të njëjtë nuk janë edhe aq të thjeshta se duhet gjetur ngjashmërinë që
njërën figurë e pasqyron në tjetrën. Kështu, psh, edhe te figurat më të
thjeshta siç janë trekëndëshat ato mund të shprehen në disa mënyra. Po i
përmendim këtu kriteret e ngjashmërisë së trekëndshave që zbatohen
shpesh gjatë vërtetimit të teoremave.
P.1. (Rregulla e parë e ngjashmërisë së trekëndshave)
ABC A ' B ' C ' , në qoftë se dy brinjë të njërit trekëndësh janë

A
o
A’
o
B”o o C”
Bo o C B’ o o C’
Fig.1
proporcionale me brinjët përkatëse të trekëndëshit tjetër, kurse këndet të
përcaktuara prej tyre janë kongruente proporcionale me brinjët përkatëse
të trekëndëshit tjetër, kurse këndet të përcaktuara prej tyre janë
kongruente.
I Hyrje 137

Vërtetim. Le të jetë A ' B ' : AB  A ' C ' : AC dhe A '  A . Homotetia


A' B '
 A,k : A, B, C  A, B ", C ", ku k  .
AB
AB " A' B '
Nga k   AB "  A ' B '. Po kështu vërtetohet se
AB AB
AC "  A ' C '.
Tani nga AB "C "  A ' B ' C ' rrjedh se I,  : A, B ", C "  A ', B ', C '.
Pasi që f   A,k : A, B, C  A ', B ', C ' dhe f  N rrjedh se
AB "C " A ' B ' C ' .□

Shënim. Kemi vërtetuar se po të jenë dy trekëndësha me brinjë


proporcionale, ateherë ekziston ngjashmëria që njërin e pasqyron në
tjetrin. Nga T.5 në 7.3 në qoftë se plotësohen konditat e P.1,
trekëndeëshat në fjalë i kanë të gjitha brinjët proporcionale, pra ata janë
të ngjashëm.

P.2. (Rregulla e dytë e ngjashmërisë së trekëndshave) Dy


trerkëndësha janë të ngjashëm, në qoftë se dy kënde të njërit trekëndësh
janë congruent me dy kënde përkatëse të trekëndëshit tjetër.
Vërtetim. Le të jenë A  A ', B  B '. Ateherë edhe C  C '.
A' B '
Homotetia  A,k : A, B, C  A, B ", C ", ku k  . Nga
AB
AB " A' B '
k   AB "  A ' B ' dhe BC B " C ". Tani kemi që
AB AB
AB "C "  A ' B ' C ' rrjedhimisht, ekziston izometria :
A, B ", C "  A ', B ', C '. Me që f=  A, k N dhe f:
A, B, C  A ', B ', C '  ABC A ' B ' C '. □

P.3. (Rregulla e tretë) Në qoftë se brinjët e një trekëndëshi janë


proporcionale me brinjët e trekëndshi tjetër, ateherë ata dy trekëndësha
janë të ngjashëm.
Vërtetimi rrjedh nga teorema per ekzistencën e ngjashmërisë. Por mund
ta vërtetojmë edhe kështu: Le të jetë
A ' B ': AB  B ' C ': BC  C ' A ': CA  k. Po të jetë
AB " AC " B " C "
 A,k : A, B, C  A, B ", C ", ateherë nga    k dhe
AB AC BC
kushtet e dhëna kemi AB "  A ' B ', AC "  A ' C ', B " C "  B ' C ' . Tani
138 I Hyrje

AB " C "  A ' B ' C '    I ,  : A, B ", C "  A ', B ', C ' . Ngjashmëria
f   A,k : A, B, C  A ', B ', C '  ABC A ' B ' C '. □

P.4. (Rregulla e katërt)Dy trekëndësha janë të ngjashëm, në qoftë se dy


brinjë të njërit janë proporcionale me dy brinjë të trekëndshit tjetër, këndet
përballë dy prej tyre janë kongruentë, ndërsa këndet përballë dy brinjëve
të tjera janë të ngushtë, të drejtë apo të gjerë.
Vërtetim. Le të jenë A ' B ' : AB  A ' C ' : AC , B '  B , kurse këndet
C , C ' janë të ngushtë, të drejtë apo të gjerë. Homotetia
A' B '
 A,k : A, B, C  A, B ", C ", ku k  . Prej aty kemi AB "  A ' B ",
AB
AC "  A ' C ', B "  B  B ', C "  C. Prej këtu rrjedh kongruenca e
trekendëshave AB " C "  A ' B ' C ', prandaj ekziston izometria :
A, B ", C "  A ', B ', C ' dhe rrjedhimisht ngjashmëria
f   A,k : A, B, C  A ', B ', C ', pra ABC A ' B ' C '. □

7.7. NGJASHMËRIA NË E3

You might also like